D) An increased end-systolic volume
E) A shift of the pressure-volume relationship to the left
157. Which of the following is correct about treatment of shock?
A) Dextran solution (if plasma is not available) is helpful as
plasma substitution in cardiogenic shock since it increases
interstitial pressure.
B) Anti-histamine agents in anaphylactic shock help to prevent
development of disseminated intra-vascular coagulation
C) Dextran solution (if whole blood or plasma are not available)
is helpful as plasma substitution since it increases intravascular hydrostatic pressure.
D) Dextran solution (if whole blood or plasma are not available)
is helpful as plasma substitution since it increases intravascular colloid osmotic pressure.
E) Glucocorticoids stimulate release of enzymes from lysosomes
that are crucial for tissue protection and to prevent cellular
deterioration
Answers
1. B) The capillary filtration coefficient is calculated as rate of net fluid
movement across a capillary wall divided by net filtration pressure. Net
filtration pressure = capillary hydrostatic pressure − plasma colloid
osmotic pressure + interstitial colloid osmotic pressure − interstitial
hydrostatic pressure. The rate of net fluid movement across the capillary
wall is 150 ml/min.
Capillary filtration coefficient (Kf
) = Filtration rate/Net filtration pressure
Kf = 150 ml/min / [Pc − Πc + Πi − PI
]
Kf = 150 ml/min / [25 − 25 + 10 − (−5)]
Kf = 150 /15 = 10 ml/min/mm Hg
TMP14 p. 197
2. A) Moving from a supine to a standing position causes an acute fall in
arterial pressure that is sensed by arterial baroreceptors located in the
carotid bifurcation and aortic arch. Activation of the arterial
baroreceptors leads to an increase in sympathetic outflow to the heart
and peripheral vasculature and a decrease in parasympathetic outflow
to the heart. The increase in sympathetic activity to peripheral vessels
results in an increase in total peripheral resistance. The increase in
sympathetic activity to the heart results in an increase in heart rate and
strength of contraction. The decrease in parasympathetic outflow to the
heart also contributes to the increase in heart rate.
TMP14 pp. 222–223
3. E) Administration of a drug that decreases the diameter of arterioles in a
muscle bed increases the vascular resistance. The increased vascular
resistance decreases vascular conductance and blood flow. The
reduction in arteriolar diameter also leads to a decrease in capillary
hydrostatic pressure and capillary filtration rate.
TMP14 pp. 174, 177, 197
4. G) Moving from a supine to a standing position causes an acute fall in
arterial pressure that is sensed by arterial baroreceptors located in the
carotid sinuses and aortic arch. Activation of the baroreceptors results in
a decrease in parasympathetic activity (or vagal tone) and an increase in
sympathetic activity, which leads to an increase in plasma renin activity
(or renin release).
TMP14 pp. 217–223
5. A) The difference between systolic pressure and diastolic pressure is the
pulse pressure. The two major factors that affect pulse pressure are the
stroke volume output of the heart and the compliance of the arterial
tree. In patients with moderate aortic regurgitation (due to incomplete
closure of aortic valve), the blood that is pumped into the aorta
immediately flows back into the left ventricle. The backflow of blood
into the left ventricle increases stroke volume and systolic pressure.The
rapid backflow of blood also results in a decrease in diastolic pressure.
Thus, patients with moderate aortic regurgitation have high systolic
pressure, low diastolic pressure, and high pulse pressure.
TMP14 pp. 184–185
6. F) The increase in local metabolism during exercise increases carbon
dioxide production and decreases tissue oxygen concentration and
tissue pH. The decrease in tissue oxygen concentration, tissue pH, and
increase in carbon dioxide production increase arteriolar diameter
(decrease arteriole resistance) and increase vascular conductance and
blood flow to skeletal muscles.
TMP14 pp. 206–208
7. B) Cognitive stimuli increase cerebral blood flow by decreasing cerebral
vascular resistance. The diameter of cerebral vessels is increased by
various metabolic factors in response to cognitive stimuli. Metabolic
factors that enhance cerebral blood flow include increases in carbon
dioxide, hydrogen ion (decreased pH), and adenosine.
TMP14 pp. 205–207
8. A) Histamine is a vasodilator that is typically released by mast cells and
basophils. Infusion of histamine into a brachial artery would decrease
arteriolar resistance and increase water permeability of the capillary
wall. The decrease in arteriolar resistance would also increase capillary
hydrostatic pressure. The increase in capillary hydrostatic pressure and
water permeability leads to an increase in capillary filtration rate,
interstitial volume, and interstitial hydrostatic pressure.
TMP14 pp. 174, 197
9. C) An increase in shear stress in blood vessels is one of the major stimuli
for the release of nitric oxide by endothelial cells. Nitric oxide increases
blood flow by increasing cyclic guanosine monophosphate.
TMP14 p. 210
10. C) Angiotensin I is formed by an enzyme (renin) acting on a substrate
called angiotensinogen. Angiotensin I is converted to angiotensin II by a
converting enzyme. Angiotensin II also has a negative feedback effect on
juxtaglomerular cells to inhibit renin secretion. Angiotensin II is a
powerful vasoconstrictor and sodium-retaining hormone that increases
arterial pressure. Administration of an ACE inhibitor would increase
plasma renin concentration, decrease angiotensin II formation, and
decrease total peripheral resistance and arterial pressure.
TMP14 pp. 236–238
11. E) A decrease in the diameter of a precapillary arteriole would increase
arteriolar resistance. The increase in arteriolar resistance would lead to a
decrease in vascular conductance, capillary blood flow, hydrostatic
pressure, and filtration rate.
TMP14 pp. 174, 197
12. A) Stenosis of one kidney results in the release of renin and the
formation of angiotensin II from the affected kidney. Angiotensin II
stimulates aldosterone production and increases total peripheral
resistance by constricting most of the blood vessels in the body.
TMP14 p. 239
13. D) Blood flow in a vessel is directly proportional to the fourth power of
the vessel radius. Increasing vessel diameter (or radius) by 100% (2 ×
control) would increase blood flow 2 to the fourth power × normal blood
flow (100 ml/min). Thus, blood flow would increase to 100 ml/min × 16,
or approximately 1600 ml/min.
TMP14 p. 177
14. A) In patent ductus arteriosus, a large quantity of the blood pumped
into the aorta by the left ventricle immediately flows backward into the
pulmonary artery and then into the lung and left atrium. The shunting
of blood from the aorta results in a low diastolic pressure, while the
increased inflow of blood into the left atrium and ventricle increases
stroke volume and systolic pressure. The combined increase in systolic
pressure and decrease in diastolic pressure results in an increase in
pulse pressure.
TMP14 pp. 184–185
15. E) A decrease in tissue oxygen tension is thought to be an important
stimulus for vascular endothelial growth factor and the growth of blood
vessels in solid tumors.
TMP14 pp. 211–213
16. A) The net movement of sodium across a capillary wall is directly
proportional to the wall permeability to sodium, wall surface area, and
concentration gradient across the capillary wall. Thus, increases in
permeability to sodium, surface area, and sodium concentration
gradient wall would all increase the net movement of sodium across the
capillary wall.
TMP14 pp. 195–196
17. D) Constriction of the carotid artery decreases blood pressure at the
level of the carotid sinus. A decrease in carotid sinus pressure leads to a
decrease in carotid sinus nerve impulses to the vasomotor center, which
in turn leads to enhanced sympathetic nervous activity and decreased
parasympathetic nerve activity. The increase in sympathetic nerve
activity results in peripheral vasoconstriction and an increase in total
peripheral resistance and heart rate. The decreased parasympathetic
nerve activity to the heart would also contribute to the increase in heart
rate.
TMP14 pp. 221–223
18. E) Pulse pressure is the difference between systolic pressure and
diastolic pressure. The two major factors that affect pulse pressure are
the stroke volume output of the heart and the compliance of the arterial
tree. An increase in stroke volume increases systolic and pulse pressure,
whereas an increase in compliance of the arterial tree decreases pulse
pressure. Moderate aortic valve stenosis results in a decrease in stroke
volume, which leads to a decrease in systolic pressure and pulse
pressure.
TMP14 pp. 184–185
19. B) A person with atherosclerosis would be expected to have decreased
arterial compliance. The decrease in arterial compliance would lead to
an increase in systolic pressure and pulse pressure.
TMP14 pp. 184–185
20. B) Constriction of the carotid artery reduces blood pressure at the
carotid bifurcation where the arterial baroreceptors are located. The
decrease in arterial pressure activates baroreceptors, which in turn leads
to an increase in sympathetic activity and a decrease in parasympathetic
activity (or vagal tone). The enhanced sympathetic activity results in
constriction of peripheral blood vessels, including the kidneys. The
enhanced sympathetic activity leads to an increase in total peripheral
resistance and a decrease in renal blood flow. The combination of
enhanced sympathetic activity and decreased vagal tone also leads to an
increase in heart rate.
TMP14 pp. 221–223
21. B) Filtration rate is the product of the filtration coefficient (Kf
) and the
net pressure across the capillary wall. The net pressure for fluid
movement across a capillary wall is promoted by increases in capillary
hydrostatic pressure and positive interstitial colloid osmotic pressure,
whereas negative plasma colloid osmotic pressure and a positive
interstitial hydrostatic pressure oppose filtration. Thus, increased
capillary hydrostatic pressure, decreased plasma colloid osmotic pressure,
and increased interstitial colloid osmotic pressure would all promote
filtration. Increased arteriolar resistance would decrease filtration by
decreasing capillary hydrostatic pressure. The filtration coefficient is the
product of capillary surface area and the capillary water permeability. A
decrease in capillary water permeability would decrease the filtration
coefficient and reduce the filtration rate.
TMP14 pp. 196–197
22. E) Solid tumors are metabolically active tissues that need increased
quantities of oxygen and other nutrients. When metabolism in a tissue is
increased for a prolonged period, the vascularity of the tissue also
increases. One of the important factors that increases growth of new
blood vessels is VEGF. Presumably, a deficiency of tissue oxygen or
other nutrients, or both, leads to the formation of VEGF.
TMP14 p. 212
23. A) A decrease in the diameter of a precapillary arteriole increases
arteriolar resistance while decreasing vascular conductance and
capillary blood flow, hydrostatic pressure, filtration rate, interstitial
volume, and interstitial hydrostatic pressure.
TMP14 pp. 174, 197
24. C) Excess secretion of aldosterone results in enhanced tubular
reabsorption of sodium and secretion of potassium. The increased
reabsorption of sodium and water leads to an increase in extracellular
fluid volume, which in turn suppresses renin release by the kidney. The
increase in potassium secretion leads to a decrease in plasma potassium
concentration, or hypokalemia.
TMP14 pp. 237–238
25. B) The two main factors that increase lymph flow are an increase in
capillary filtration rate and an increase in lymphatic pump activity. A
decrease in plasma colloid osmotic pressure increases capillary filtration
rate, interstitial volume and hydrostatic pressure, and lymph flow. In
contrast, an increase in hydraulic conductivity of the capillary wall and
capillary hydrostatic pressure increase capillary filtration rate,
interstitial volume and pressure, and lymph flow. An increase in
arteriole resistance would decrease vascular conductance, capillary
hydrostatic pressure, capillary filtration rate, interstitial volume and
pressure, and lymph flow.
TMP14 pp. 195–203
26. E) According to Poiseuille’s law, flow through a vessel increases in
proportion to the fourth power of the radius. A 4-fold increase in vessel
diameter (or radius) would increase 4 to the fourth power, or 256 times
normal. Thus, flow through the vessel after increasing the vessel 4 times
normal would increase from 100 to 25,600 ml/min.
TMP14 p. 177
27. B) Vascular resistance is equal to arterial pressure divided by blood
flow. In this example, arterial pressure is 125 mm Hg, venous pressure is
5 mm Hg, and blood flow is 1200 ml/min ( plasma flow/hematocrit).
Thus, vascular resistance is equal to 120 divided by (600/.50 or 1200), or
0.10 mm Hg/ml/min.
TMP14 p. 174
28. D) The rate of blood flow is directly proportional to the fourth power of
the vessel radius and to the pressure gradient across the vessel. In
contrast, the rate of blood flow is inversely proportional to the viscosity
of the blood. Thus, an increase in blood viscosity would decrease blood
flow in a vessel.
TMP14 pp. 177–178
29. D) The flow in a vessel is directly proportional to the pressure gradient
across the vessel and to the fourth power of the radius of the vessel. In
contrast, blood flow is inversely proportional to the viscosity of the
blood. Because blood flow is proportional to the fourth power of the
vessel radius, the vessel with the largest radius (vessel D) would have
the greatest flow.
TMP14 p. 177
30. B) The arterial baroreceptors are activated in response to a fall in
arterial pressure. During hemorrhage, the fall in arterial pressure at the
level of the baroreceptors results in enhanced sympathetic outflow from
the vasomotor center and a decrease in parasympathetic nerve activity.
The increase in sympathetic nerve activity leads to constriction of
peripheral blood vessels, increased total peripheral resistance, and
plasma renin activity, angiotensin II and a return of blood pressure
toward normal. The decrease in parasympathetic nerve activity and
sympathetic outflow would result in an increase in heart rate.
TMP14 pp. 221–223, 236–238
31. A) Activation of the baroreceptors leads to an increase in sympathetic
activity, which in turn increases heart rate, strength of cardiac
contraction, and constriction of arterioles and veins. The increase in
venous constriction results in an increase in mean circulatory filling
pressure, venous return, and cardiac output.
TMP14 pp. 221–223
32. E) The conversion of angiotensin I to angiotensin II is catalyzed by a
converting enzyme that is present in the endothelium of the lung vessels
and in the kidneys. The converting enzyme also serves as a kininase that
degrades bradykinin. Thus, a converting enzyme inhibitor not only
decreases the formation of angiotensin II but also inhibits kininases and
the breakdown of bradykinin. Angiotensin II is a vasoconstrictor and a
powerful sodium-retaining hormone. While plasma bradykinin
increases, the major cause for the decrease in arterial pressure in
response to an ACE inhibitor is the decrease in formation of angiotensin
II.
TMP14 pp. 236–238
33. G) When blood pressure falls below 80 mm Hg, carotid and aortic
chemoreceptors are activated to elicit a neural reflex to minimize the fall
in blood pressure. The chemoreceptors are chemosensitive cells that are
sensitive to oxygen lack, carbon dioxide excess, or hydrogen ion excess
(or fall in pH). The signals transmied from the chemoreceptors into the
vasomotor center excite the vasomotor center to increase arterial
pressure.
TMP14 p. 224
34. D) Although sympathetic nerves, angiotensin II, and vasopressin are
powerful vasoconstrictors, blood flow to skeletal muscles under normal
physiological conditions is mainly determined by local metabolic factors
such as tissue adenosine, oxygen, hydrogen, and carbon dioxide
concentrations. Capillary osmotic pressure plays a role in determining
fluid movement across a capillary not blood flow.
TMP14 pp. 206–208
35. E) During exercise, tissue levels of carbon dioxide and lactic acid
increase. These metabolites dilate blood vessels, decrease arteriolar
resistance, and enhance vascular conductance and blood flow.
TMP14 pp. 208–209
36. C) The velocity of blood flow within each segment of the circulatory
system is inversely proportional to the total cross-sectional area of the
segment. Because the capillaries has the highest total cross-sectional area
of all circulatory segments, it has the lowest velocity of blood flow.
TMP14 p. 172
37. B) Filtration rate is the product of the filtration coefficient (Kf
) and the
net pressure across the capillary wall. The net pressure for fluid
movement across a capillary wall = capillary hydrostatic pressure −
plasma colloid osmotic pressure + interstitial colloid osmotic pressure −
interstitial hydrostatic pressure. The net pressure in this question
calculates to be 10 mm Hg, and the Kf
is 15. Thus, the filtration rate is 15
× 10, or 150 ml/min.
TMP14 p. 197
38. A) Resistance of a vessel = pressure gradient ÷ blood flow of the vessel.
In this example, vessel A has the highest vascular resistance (100 mm
Hg/1000 ml/min, or 0.1 mm Hg/ml/min).
TMP14 pp. 173–174
39. C) The transport of oxygen across a capillary wall is proportional to the
capillary surface area, capillary wall permeability to oxygen, and oxygen
gradient across the capillary wall. Thus, a 2-fold increase in the oxygen
concentration gradient would result in the greatest increase in the
transport of oxygen across the capillary wall. A 2-fold increase in
intercellular clefts in the capillary wall would not have a significant
impact on oxygen transport because oxygen can permeate the
endothelial cell wall.
TMP14 pp. 195–196
40. D) Atrial natriuretic peptide is released from myocytes in the atria in
response to increases in atrial pressure. ANP in turn inhibits the
production of angiotensin II and aldosterone. Stretch of the atria also
results in a nervous reflex to inhibit renal sympathetic nerve activity.
TMP14 p. 224
41. C) The capillaries have the largest total cross-sectional area of all
vessels of the circulatory system. The venules also have a relatively large
total cross-sectional area but not as great as the capillaries, which
explains the large storage of blood in the venous system compared with
that in the arterial system.
TMP14 p. 172
42. E) An increase in atrial pressure increases plasma levels of atrial
natriuretic peptide, which in turn would decrease plasma levels of
angiotensin II and aldosterone and increase sodium excretion.
TMP14 p. 224
43. C) An increase in perfusion pressure to a tissue results in excessive
delivery of nutrients such as oxygen to a tissue. The increase in tissue
oxygen concentration constricts arterioles and returns blood flow and
nutrient delivery toward normal levels.
TMP14 pp. 209–210
44. C) The percentage of total blood volume in the veins is approximately
64%.
TMP14 p. 172
45. B) Constriction of the renal artery increases release of renin, formation
of angiotensin II and aldosterone, and arterial pressure. A 50%
reduction in renal artery pressure would be below the range of renal
autoregulation and would result in a decrease in the glomerular
filtration rate.
TMP14 pp. 239–240
46. B) An increase in plasma colloid osmotic pressure would reduce net
filtration pressure and capillary filtration rate. Increases in capillary
hydrostatic pressure and interstitial colloid osmotic pressure would also
favor capillary filtration. An increase in venous hydrostatic pressure and
arteriolar diameter would tend to increase capillary hydrostatic pressure
and capillary filtration rate.
TMP14 pp. 194–201
47. E) A decrease in sodium intake would result in a decrease in sodium
excretion to maintain sodium balance. Angiotensin II and aldosterone
would increase in response to a chronic decrease in sodium intake,
whereas plasma atrial natriuretic peptide levels would decrease.
TMP14 pp. 236–238
48. E) The rate of lymph flow increases in proportion to the interstitial
hydrostatic pressure and the lymphatic pump activity. An increase in
capillary hydrostatic pressure would increase filtration rate, interstitial
volume, interstitial hydrostatic pressure, and lymph flow. An increase in
plasma colloid osmotic pressure would decrease filtration rate,
interstitial volume, interstitial hydrostatic pressure, and lymph flow. A
decrease in interstitial volume would increase interstitial hydrostatic
pressure and lymph flow. A decrease in arteriolar diameter would
decrease capillary hydrostatic pressure, capillary filtration, interstitial
volume, interstitial hydrostatic pressure and lymph flow.
TMP14 pp. 197–204
49. E) Nitric oxide and prostacyclin are potent vasodilator and natriuretic
substances. Thus, increases in nitric oxide and prostacyclin production
would result in a decrease in arterial pressure. In contrast, angiotensin II
and aldosterone are anti-natriuretic and pro-hypertensive factors. An
increase in the production of these factors would increase arterial
pressure.
TMP14 p. 244
50. B) The liver endothelium contains many open pores, or fenestrae that
allow extremely large molecules such as albumin to pass in or out of the
liver tissues. Under normal conditions, very lile if any albumin cross
the capillary walls of muscle, glomerular, brain, or intestine.
TMP14 p. 195
51. D) The Cushing reaction is a special type of CNS ischemic response
that results from increased pressure of the cerebrospinal fluid around
the brain in the cranial vault. When the cerebrospinal fluid pressure
rises, it decreases the blood supply to the brain and elicits a CNS
ischemic response. The CNS ischemic response includes enhanced
sympathetic activity, decreased parasympathetic activity, and increased
heart rate, arterial pressure, and total peripheral resistance.
TMP14 p. 226
52. B) The factors that determine the net movement of glucose across a
capillary wall include the wall permeability to glucose, the glucose
concentration gradient across the wall, and the capillary wall surface
area. Thus, an increase in the concentration difference of glucose across
the wall would enhance the net movement of glucose.
TMP14 pp. 195–196
53. E) An increase in atrial pressure of 10 mm Hg would tend to decrease
venous return to the heart and increase vena cava hydrostatic pressure.
Plasma colloid osmotic pressure, interstitial colloid osmotic pressure,
arterial pressure, and cardiac output would generally be low to normal
in this patient.
TMP14 pp. 197–200
54. A) An increase in renal arterial pressure results in increases in sodium
and water excretion. Normally, glomerular filtration rate would be
normal or slightly increased in response to an increase in renal artery
pressure. However, in the absence of an intact tubuloglomerular
feedback system, an important renal autoregulatory mechanism, an
increase in renal artery pressure would result in significant increases in
glomerular filtration rate.
TMP14 p. 210
55. C) The vascular compliance is proportional to the vascular
distensibility and the vascular volume of any given segment of the
circulation. The compliance of a systemic vein is 24 times that of its
corresponding artery because it is about 8 times as distensible and has a
volume about 3 times as great.
TMP14 p. 183
56. E) The difference between systolic pressure and diastolic pressure is
called the pulse pressure. The two main factors that affect pulse
pressure are stroke volume and arterial compliance. Pulse pressure is
directly proportional to the stroke volume and inversely proportional to
the arterial compliance. Thus, increases in systolic pressure or stroke
volume would tend to increase pulse pressure.
TMP14 pp. 184–185
57. B) Moving from a supine to a standing position results in pooling of
blood in the lower extremities and a fall in blood pressure. The pooling
of blood in the legs increases venous hydrostatic pressure. The fall in
arterial pressure activates the arterial baroreceptors, which in turn
increases sympathetic nerve activity and decreases parasympathetic
nerve activity. The increase in sympathetic activity constricts renal
vessels and reduces renal blood flow. The heart rate also increases.
TMP14 pp. 221–223
58. B) Moving from a supine to a standing position would tend to cause
pooling of blood in the lower extremities and a reduction in arterial
pressure. Decreased arterial pressure activates arterial the baroreceptor
reflex which then leads to increased sympathetic nervous system
activity, decreased parasympathetic activity, increased heart rate, and
increased cardiac contractility.
TMP14 pp. 221–223
59. B) An increase in arterial pressure to a tissue leads to an increase in
tissue oxygen concentration and a decrease in tissue carbon dioxide
concentration. Both events lead to a decrease in arteriolar diameter,
increased vascular resistance, and decreased vascular conductance.
TMP14 pp. 209–210
60. B) Because oxygen is lipid soluble and can cross the capillary wall with
ease, it has the fastest rate of movement across the capillary wall. The
ability of lipid-insoluble substances such as sodium, albumin, and
glucose to move across a capillary wall depends on the permeability of
the capillary to lipid-insoluble substances. Because the capillary wall is
relatively impermeable to albumin, it has the slowest rate of net
movement across the capillary wall.
TMP14 p. 195
61. A) An increase in capillary wall permeability to water would increase
capillary filtration rate, whereas increases in arteriolar resistance,
plasma colloid osmotic pressure, and interstitial hydrostatic pressure
would all decrease filtration rate. Plasma sodium concentration would
have no effect on filtration.
TMP14 pp. 197–1201
62. D) The tendency for turbulent flow occurs at vascular sites where the
velocity of blood flow is high. The aorta has the highest velocity of blood
flow.
TMP14 pp. 175–176
63. E) Total peripheral vascular resistance = (arterial pressure − right atrial
pressure) ÷ cardiac output. In this example, total peripheral vascular
resistance = 130 mm Hg ÷ 3.5 l/min, or approximately 37 mm Hg/l/min.
TMP14 p. 177
64. C) Interstitial hydrostatic pressure in a muscle capillary bed is normally
negative (−3 mm Hg). Pumping by the lymphatic system is the basic
cause of the negative pressure.
TMP14 pp. 202–203
65. D) The two main factors that affect pulse pressure are stroke volume
and arterial compliance. Decreases in stroke volume decrease pulse
pressure, and an increase in arterial compliance decreases pulse
pressure. Hemorrhage and decreased venous return would decrease
stroke volume and pulse pressure. In patients with patent ductus, stroke
volume and pulse pressure are increased as a result of shunting of blood
from the aorta to the pulmonary artery.
TMP14 pp. 184–186
66. D) The primary mechanism whereby solutes move across a capillary
wall is simple diffusion.
TMP14 p. 196
67. E) Movement of the leg muscles causes blood to flow toward the vena
cava, which reduces venous hydrostatic pressure. A decrease in right
atrial pressure would increase venous return and decrease venous
hydrostatic pressure. Pregnancy and the presence of abdominal tumor
in the abdomen would tend to compress the vena cava and increase
venous hydrostatic pressure in the legs.
TMP14 pp. 188–190
68. A) Nitric oxide is a vasodilator that is believed to play a role in
regulating blood flow. Infusion of a nitric oxide donor into the brachial
artery would increase arteriolar diameter and decrease arteriolar
resistance. The decrease in arteriolar resistance would also result in an
increase in capillary hydrostatic pressure and filtration rate. The
increase in filtration rate leads to an increase in interstitial hydrostatic
pressure and lymph flow.
TMP14 pp. 173–174, 197–200, 210
69. D) In persons with decompensated heart failure, the kidneys retain
sodium and water, which causes a weight gain and an increase in blood
volume. This effect increases the mean systemic filling pressure, which
also stretches the heart. Therefore, a decreased mean systemic filling
pressure does not occur in decompensated heart failure. The excess
blood volume often will overstretch the sarcomeres of the heart, which
will prevent them from achieving their maximal tension. An excess
central fluid volume also results in orthopnea, which is the inability to
breathe properly except in the upright position.
TMP14 pp. 273–274
70. A) During progressive hemorrhagic shock, the vasomotor center often
fails, thus reducing sympathetic output. Decreases in arterial pressure
will reduce urine output. Decreased blood flow throughout the body
causes acidosis because of decreased removal of carbon dioxide. In
progressive shock due to hemorrhage, capillary permeability increases
and mean systemic filling pressure decreases.
TMP14 p. 296
71. C) With an overdose of furosemide there is a large loss of sodium and
water from the body, resulting in dehydration and sometimes shock.
The optimal therapy is to replenish the electrolytes that were lost as a
result of the overdose of the furosemide. Therefore, infusion of a
balanced electrolyte solution is the therapy of choice.
TMP14 pp. 301–302
72. C) Severe vomiting can lead to a large loss of sodium and water from
the body, resulting in dehydration and sometimes shock. The best
therapy is to replenish the depleted sodium and water lost by vomiting.
Therefore, infusion of a balanced electrolyte solution is the therapy of
choice.
TMP14 pp. 301–302
73. C) The formula for resistance to venous return is (mean systemic filling
pressure − right atrial pressure)/cardiac output. In this example, the
mean systemic filling pressure is 7 mm Hg, and the right atrial pressure
is 0 mm Hg. The cardiac output is 5 l/min. Using these values in the
previous formula indicates that the resistance to venous return is 1.4
mm Hg/l/min. Note that this formula only applies to the linear portion
of the venous return curve.
TMP14 pp. 251–253
74. A) During increases in sympathetic output to maximal values, several
changes occur. First, the mean systemic filling pressure increases
markedly, but at the same time the resistance to venous return increases.
Venous return is determined by the following formula: (mean systemic
filling pressure − right atrial pressure)/resistance to venous return.
During maximal sympathetic output, the increase in systemic filling
pressure is greater than the increase in resistance to venous return.
Therefore, in this formula, the numerator has a much greater increase
than the denominator, which results in an increase in the venous return.
TMP14 p. 255
75. C) This problem concerns the Fick principle for determining cardiac
output. The formula for cardiac output is oxygen absorbed per minute
by the lungs divided by the arterial-venous oxygen difference. In this
problem, oxygen consumption of the body is 240 ml/min, and in a
steady-state condition, this would exactly equal the oxygen absorbed by
the lungs. Therefore, by inserting these values into the equation, we see
that the cardiac output will equal 12 l/min.
TMP14 p. 256
76. A) A shift to the right in the cardiac output curve involves an increase
in the normal intrapleural pressure of −4 mm Hg. Changing the
intrapleural pressure to −1 mm Hg will shift the curve to the right.
Changing the mean systemic filling pressure does not change the
cardiac output curve. Taking a patient off of a ventilator, decreasing
intrapleural pressure to −7 mm Hg, and breathing against a negative
pressure shifts the cardiac output curve to the left.
TMP14 p. 250
77. C) Several factors can cause the cardiac output to shift to the right or to
the left. Among those are surgically opening the chest, which makes the
cardiac output curve shift 4 mm Hg to the right, and severe cardiac
tamponade, which increases the pressure inside the pericardium, thus
tending to collapse the heart, particularly the atria. Playing a trumpet or
positive pressure breathing tremendously increases the intrapleural
pressure, thus collapsing the atria and shifting the cardiac output curve
to the right. Breathing against a negative pressure shifts the cardiac
output curve to the left.
TMP14 p. 250
78. E) The plateau level of the cardiac output curve, which is one measure
of cardiac contractility, decreases in several circumstances. Some of
these circumstances include severe cardiac tamponade, which increases
the pressure in the pericardial space, and increasing parasympathetic
stimulation of the heart. Increased sympathetic stimulation of the heart
increases the level of the cardiac output curve by increasing heart rate
and contractility.
TMP14 p. 247
79. B) Cardiac output increases in several conditions because of increased
venous return. A-V fistulae also cause a decreased resistance to venous
return, thus increasing cardiac output. Cardiac output decreases in
patients with hypovolemia, severe aortic regurgitation, and
polycythemia. The hematocrit level is high in polycythemia, which
increases resistance to venous return.
TMP14 pp. 248–250
80. D) Mean systemic filling pressure is a measure of the tightness of fit of
the blood in the circulation. Mean systemic filling pressure is increased
by factors that increase blood volume and decrease the vascular
compliance. Therefore, decreased venous compliance, not increased
compliance, would cause an increase in mean systemic filling pressure.
Norepinephrine administration and sympathetic stimulation cause
arteriolar vasoconstriction and decreased vascular compliance, resulting
in an increase in mean systemic filling pressure. Increased blood volume
and skeletal muscle contraction, which cause a contraction of the
vasculature, also increase this filling pressure.
TMP14 pp. 252–253
81. C) Resistance to venous return (which is equal to cardiac output) is
equal to the mean systemic filling pressure minus the right atrial
pressure divided by venous return. Thus, resistance to venous return is
equal to 12 minus 2 divided by 10 or 10/10 or 1 mm Hg/l per min.
TMP14 pp. 253–254
82. A) Anemia decreases resistance to venous return because of arteriolar
dilation. The following mechanisms increase resistance to venous
return: increased venous resistance, increased arteriolar resistance,
increased sympathetic output, and obstruction of veins.
TMP14 pp. 253–254
83. D) Venous return (or cardiac output) is equal to the mean systemic
filling pressure minus the right atrial pressure divided by resistance to
venous return. Thus, an increase in resistance to venous return
decreases venous return and cardiac output.
TMP14 p. 253
84. E) Cardiac output decreases in several conditions because of decreased
venous return. Cardiac output decreases in hypothyroidism because of
the decreased oxygen use by the peripheral tissues, resulting in
arteriolar vasoconstriction and thus decreased venous return.
Conversely, cardiac output increases with hyperthyroidism. Beriberi
causes increased cardiac output because a lack of the vitamin thiamine
results in peripheral vasodilation. A-V fistulae also cause decreased
resistance to venous return, thus increasing cardiac output. Increased
muscle mass is associated with increased tissue metabolism, decreased
arteriolar resistance and resistance to venous return, and increased
venous return to the heart and thus an increase in cardiac output.
TMP14 pp. 248–249
85. G) In response to increases in cardiac workload, there are increases in
cardiac tissue metabolism and cardiac tissue adenosine concentration.
Adenosine reduces arteriole resistance and increases vascular
conductance.
TMP14 p. 263
86. C) The plateau level of the cardiac output curve, which is one measure
of cardiac contractility, decreases in several circumstances. Some of
these include myocarditis, severe cardiac tamponade that increases the
pressure in the pericardial space, myocardial infarction, and various
valvular diseases such as mitral stenosis. Decreased parasympathetic
stimulation of the heart actually moderately increases the level of the
cardiac output curve by increasing the heart rate.
TMP14 p. 247
87. A) During increases in sympathetic output, the main two organs that
maintain their blood flow are the brain and the heart. During exercise
for 1 hour, the intestinal flow decreases significantly, as do the renal and
pancreatic blood flows. The skeletal muscle blood flow to non-exercising
muscles also decreases at this time. Cerebral blood flow remains close to
its control value.
TMP14 pp. 259–262
88. E) According to the Fick principle,
Cardiac output = Oxygen absorbed by lungs(ml/min) (400) divided by
Arterio-venous oxygen difference (200 – 150 ml/l)
Cardiac output = 400/50 or 8 l/min
TMP14 pp. 256–257
89. A) Although bradykinin, prostaglandins, carbon dioxide, and
potassium ions serve as vasodilators for the coronary artery system, the
major controller of coronary blood flow is adenosine. Adenosine is
formed as adenosine triphosphate degrades to adenosine
monophosphate. Small portions of the adenosine monophosphate are
then further degraded to release adenosine into the tissue fluids of the
heart muscle, and this adenosine vasodilates the coronary arteries.
TMP14 p. 263
90. E) Sympathetic stimulation directly increases the strength of cardiac
contraction and increases the heart rate. In this way, the plateau of the
Starling curve elevates. Surgically opening the chest and undergoing
mechanical ventilation shifts the cardiac output curve to the right.
Cardiac tamponade rotates the curve downward, and parasympathetic
stimulation depresses the curve.
TMP14 pp. 247–248
91. E) Ischemia causes the cardiac muscle to release lactic acid, which
stimulates pain nerve endings in cardiac muscle, sending impulses
through sensory afferent nerve fibers into the central nervous system.
TMP14 pp. 263–264, 268
92. D) Several factors cause arteriolar vasodilation during exercise,
including increases in potassium ion concentration, plasma nitric oxide
concentration, plasma adenosine concentration, and plasma osmolality.
Although histamine causes arteriolar vasodilation, histamine release
does not normally occur during exercise.
TMP14 pp. 259–260
93. A) At the beginning of exercise, increases in sympathetic stimulation of
the heart strengthens the heart and increases the heart rate. Coronary
and cerebral blood flow are spared from any decrease. Reverse stress
relaxation does not occur. Venous constriction, not dilation, occurs.
TMP14 p. 261
94. D) When a sudden occlusion occurs in one of the larger coronary
arteries, cardiac tissue adenosine concentration increases as a result of
ATP breakdown. The small anastomoses in cardiac tissue begin to dilate
immediately. Then over the next several days, collateral blood flow
increases to partially restore blood flow to the ischemic tissue.
TMP14 p. 265
95. E) Several drugs have proven to be helpful to patients with myocardial
ischemia. Beta receptor blockers (not stimulators) inhibit the
sympathetic effects on the heart and are very helpful. ACE inhibition
prevents the production of angiotensin II and thus decreases the
afterload effect on the heart. Nitroglycerin causes nitric oxide release,
resulting in coronary vasodilation. Isometric exercise increases blood
pressure markedly and can be harmful, and increased dietary calcium
would be of lile benefit.
TMP14 pp. 268–269
96. E) During exercise, the sympathetic output increases markedly, which
causes arteriolar constriction in many places of the body, including nonexercising muscle. The increased sympathetic output also causes
venoconstriction throughout the body. During exercise, there also is an
increased release of norepinephrine and epinephrine by the adrenal
glands. These changes all help maintain blood pressure during exercise.
Arterioles in the exercising muscle dilate to increase blood flow to meet
the metabolic needs of the tissue.
TMP14 pp. 260–261
97. D) When increased quantities of blood flows into the heart, the
resulting stretch in the heart wall results in an increase in force of
contraction (Frank-Starling law of the heart) and stroke volume. Stretch
of the sinus node in the wall of the right atrium has a direct effect on the
rhythmicity of the node to increase the heart rate. Stretch of right atrium
also initiates a nervous reflex called the Bainbridge reflex that increases
heart rate.
TMP14 p. 245
98. C) Increased sympathetic stimulation excites the cardiac myocytes and
makes them much more susceptible to fibrillation. High (not low)
potassium increases fibrillation tendency. An increase (not a decrease) in
ventricular diameter allows the cardiac muscle to be out of the
refractory period when the cardiac impulse next arrives and can
increase the tendency to fibrillate. A low adenosine level probably only
causes some coronary constriction. Decreased parasympathetics allow
the heart rate to increase and has lile to do with fibrillation.
TMP14 p. 268
99. A) In a patient with angina due to myocardial ischemia, oxygen use by
the heart must be minimized. Oxygen use can be minimized with ACE
inhibition, which decreases angiotensin II formation. This reduces the
arterial pressure and decreases myocardial tension and oxygen use. The
use of beta sympathetic blockers (not stimulation) inhibits the effects of
excess sympathetic output on the heart, thus reducing wall tension and
oxygen use. Isometric exercise should be avoided because of the large
increase in arterial pressure that occurs. Chelation therapy with EDTA
and increased dietary calcium have lile to do with cardiac function.
TMP14 pp. 268–269
100. C) The major causes of death after myocardial infarction include a
decrease in cardiac output that prevents tissues of the body from
receiving adequate nutrition and oxygen delivery and prevents removal
of waste materials. Other causes of death are pulmonary edema, which
reduces the oxygenation of the blood, fibrillation of the heart, and
rupture of the heart. Cardiac contractility decreases after a myocardial
infarction.
TMP14 p. 266
101. E) During sympathetic stimulation, venous reservoirs constrict,
venous vascular resistance also increases, arterioles constrict (which
increases their resistance), and the heart rate increases. The epicardial
coronary vessels have a large number of alpha receptors, but the
subendocardial vessels have more beta receptors. Therefore,
sympathetic stimulation causes at least a slight constriction of the
epicardial vessels. This results in a slight decrease in epicardial flow.
TMP14 pp. 262–264
102. E) Several factors change during compensated heart failure to stabilize
the circulatory system. Because of increased sympathetic output, the
heart rate increases during compensated heart failure. The kidneys
retain sodium and water, which increases blood volume and thus right
atrial pressure. The increased blood volume that results causes an
increase in mean systemic filling pressure, which will help to increase
the cardiac output. Dyspnea usually will occur only in the early stages
of compensated failure.
TMP14 pp. 272–273
103. E) In unilateral left heart failure, the kidneys retain sodium and water
and thus increase blood volume, and the pulmonary veins, in turn,
become congested. Therefore, mean pulmonary filling pressure,
pulmonary wedge pressure, and left atrial pressure increase. In contrast,
in right heart failure, right atrial pressure increases, and edema of the
lower extremities, including the feet and ankles, occurs.
TMP14 pp. 275–276
104. A) In compensated heart failure, an increased release of angiotensin II
also occurs, which causes direct renal sodium retention and also
stimulates aldosterone secretion that will, in turn, causes further
increases in sodium retention by the kidneys. Because of the low arterial
pressure that occurs in compensated heart failure, the sympathetic
output increases. One of the results is a sympathetic vasoconstriction
(not vasodilation) of the afferent arterioles of the kidney. This decreases
the glomerular hydrostatic pressure and the glomerular filtration rate,
resulting in an increase in sodium and water retention in the body. The
excess sodium in the body increases osmolality, which increases the
release of antidiuretic hormone, which causes renal water retention (but
not sodium retention).
TMP14 p. 276
105. C) During acute pulmonary edema, the increased fluid in the lungs
diminishes the oxygen content in the blood. This decreased oxygen
weakens the heart even further and causes arteriolar dilation in the
body. This results in increases in venous return of blood to the heart,
which cause further leakage of the fluid in the lungs and further
decreases in oxygen content in the blood. It is important to interrupt this
vicious circle to save a patient’s life. This can be interrupted by placing
tourniquets on all four limbs, which effectively removes blood volume
from the chest. The patient can also breathe oxygen, and a
bronchodilator can be administered. Furosemide can be administered to
reduce some of the fluid volume in the body and especially in the lungs.
One thing you do not want to do is infuse whole blood or an electrolyte
solution in this patient because it may exacerbate the pulmonary edema
that is already present.
TMP14 p. 277
106. D) Cardiogenic shock results from a weakening of the cardiac muscle
many times after coronary thrombosis, which can result in a vicious
circle because of low cardiac output resulting in a low diastolic pressure.
This causes a decrease in coronary flow, which decreases the cardiac
strength even more. Therefore, arterial pressure, particularly diastolic
pressure, must be increased in patients with cardiogenic shock with
either vasoconstrictors or volume expanders. In this patient, the best
answer is to infuse plasma. Placing tourniquets on all four limbs
decreases the central blood volume, which would worsen the condition
of the patient in shock.
TMP14 p. 275
107. B) This patient has a resting cardiac output of 4 l/min, and his cardiac
reserve is 300% of this resting cardiac output or 12 l/min. This gives a
total maximum cardiac output of 16 l/min. Therefore, the cardiac reserve
is the percentage increase that the cardiac output can be elevated over
the resting cardiac output.
TMP14 pp. 277–278
108. B) Several factors cause sodium retention during heart failure,
including aldosterone release, decreased glomerular filtration rate, and
increased angiotensin II release. A decrease in mean arterial pressure
also results in decreases in glomerular hydrostatic pressure and causes a
decrease in renal sodium excretion. During heart failure, blood volume
increases, resulting in an increased cardiac stretch. In particular, the
atrial pressure increases, causing a release of atrial natriuretic factor,
resulting in an increase in renal sodium excretion.
TMP14 pp. 276–277
109. D) There is a vicious circle of cardiac deterioration in cardiogenic
shock. A weakened heart causes a decreased cardiac output, which
decreases arterial pressure. The decreased arterial pressure, particularly
the decrease in diastolic pressure, decreases the coronary blood flow
and further weakens the heart and thus further decreases cardiac
output. The therapy of choice for a patient in cardiogenic shock is to
increase the arterial pressure either with a vasoconstrictor drug or with
a volume-expanding drug. Placing tourniquets on the four limbs,
withdrawing a moderate amount of blood, or administering furosemide
decreases the thoracic blood volume and thus worsens the condition of
the patient in cardiogenic shock.
TMP14 p. 275
110. A) In unilateral right heart failure, the right atrial pressure increases,
and the overall cardiac output decreases, which results in a decrease in
arterial pressure and urinary output. However, left atrial pressure does
not increase but in fact decreases.
TMP14 p. 275
111. B) During compensated heart failure, many factors combine to
increase cardiac output so it returns to normal. The kidneys decrease
their urinary output of sodium and water to increase the blood volume.
This action, when combined with a depressed cardiac output curve,
increases right atrial pressure. Mean systemic filling pressure increases
(not decreases), and the venous return of blood back toward the heart
thus increases right atrial pressure. Heart rate is normal, and sweating
and dyspnea are absent in the chronic stages of compensated failure.
TMP14 pp. 272–273, 278–279
112. A) Reduction of fluid in the lungs can prevent rapid deterioration in
patients with acute pulmonary edema. Furosemide causes venodilation,
which reduces thoracic blood volume and acts as a powerful diuretic.
These both reduce excess fluid in the lungs. Blood can actually be
removed in moderate quantities from the patient to decrease the volume
of blood in the chest. Patients should also breathe oxygen to increase the
oxygen levels in the blood. However, they should never be given a
volume expander, such as saline, plasma, whole blood, or dextran,
because it could worsen the pulmonary edema. Norepinephrine would
be of lile help in treating pulmonary edema.
TMP14 p. 277
113. B) In compensated heart failure, mean systemic filling pressure
increases because of hypervolemia, and cardiac output is often at
normal values. The patient has air hunger, called dyspnea, and excess
sweating occurs in the early phases of compensated heart failure.
However, right atrial pressure becomes elevated to very high values in
these patients and is a hallmark of this disease.
TMP14 pp. 272–273, 278–279
114. B) Mean systemic pressure is increased by factors that increase blood
volume or decrease vascular capacity. Sympathetic inhibition and
venous dilation both decrease the mean systemic filling pressure. In
congestive heart failure, the kidneys retain great quantities of sodium
and water, resulting in an increase in blood volume, which causes large
increases in mean systemic filling pressure.
TMP14 pp. 272–274
115. A) During compensated heart failure, release of angiotensin II and
aldosterone is increased, causing the kidneys retain sodium and water,
which increases the blood volume in the body and the venous return of
blood to the heart. This situation results in an increase in right atrial
pressure. Increased sympathetic output during compensated heart
failure increases heart rate. Air hunger, called dyspnea, occurs during
any type of exertion. The patient also has orthopnea, which is the air
hunger that occurs from being in a recumbent position.
TMP14 pp. 271–273
116. B) During decompensated heart failure, cardiac output decreases
because of weakness of the heart and edema of the cardiac muscle.
Pressures in the pulmonary capillary system increase, including the
pulmonary capillary pressure and the mean pulmonary filling pressure.
Depletion of norepinephrine in the endings of the cardiac sympathetic
nerves is another factor that causes weakness of the heart.
TMP14 pp. 273–274, 279–280
117. D) In decompensated heart failure, the kidneys retain sodium and
water, which causes weight gain and an increase in blood volume. This
situation increases the mean systemic filling pressure, which also
stretches the heart. Therefore, decreased mean systemic filling pressure
does not occur in decompensated heart failure. The excess blood volume
often overstretches the sarcomeres of the heart, which prevents them
from achieving their maximal tension. An excess central fluid volume
also results in orthopnea, which is the inability to breathe properly
except in the upright position.
TMP14 pp. 273–274, 279–280
118. C) The mean electrical axis of the QRS of this patient is shifted
rightward to 170 degrees, which indicates that the right side of the heart
is involved. Both aortic stenosis and mitral regurgitation cause a
leftward shift of the QRS axis. Mitral stenosis does not affect the left
ventricle, but in severe enough circumstances, it could cause an increase
in pulmonary artery pressure, which would cause an increase in
pulmonary capillary pressure at the same time. Tricuspid stenosis does
not affect the right ventricle. Therefore, pulmonary valve stenosis is the
only condition that fits this set of symptoms.
TMP14 pp. 286–287, 290
119. A) The fourth heart sound occurs at the end of diastole and is caused
by inrushing of blood into the ventricles due to atrial contraction. The
first heart sound is caused by closing of the A-V valves. The closing of
the aortic and pulmonary valves at the end of systole causes the second
heart sound. This initiates a vibration throughout the ventricles, aorta,
and pulmonary artery. The third heart sound is caused by inrushing of
blood into the ventricles in the early to middle part of diastole.
TMP14 p. 284
120. B) Blowing murmurs of relatively high pitch are usually murmurs
associated with valvular insufficiency. The key pieces of data to identify
this murmur are the systolic and diastolic pressures. Aortic valve
regurgitation typically has a high pulse pressure, which is the systolic –
diastolic pressure and in this case is 100 mm Hg. Also notice that the
diastolic pressure decreases to very low values of 40 mm Hg as the
blood leaks back into the left ventricle.
TMP14 pp. 286–287
121. E) Left ventricular hypertrophy occurs when the left ventricle either
has to produce high pressure or when it pumps extra volume with each
stroke. During aortic regurgitation, extra blood leaks back into the
ventricle during the diastolic period. This extra volume must be
expelled during the next heartbeat. During mitral regurgitation, some
blood gets pumped out into the aorta, while at the same time blood
leaks back into the left atrium. Therefore, the left ventricle is pumping
extra volume with each heartbeat. During aortic stenosis, the left
ventricle must contract very strongly, producing high wall tension to
increase the aortic pressure to the values high enough to expel blood
into the aorta. During mitral stenosis, the ventricle is normal because the
atrium produces the extra pressure to get blood through the stenotic
mitral valve.
TMP14 pp. 286–287
122. E) Several diastolic murmurs can be heard easily with a stethoscope.
During diastole, aortic and pulmonary valve regurgitation occur
through the insufficient valves causing the heart murmur at this time.
Tricuspid and mitral stenosis are diastolic murmurs because blood flows
through the restricted valves during the diastolic period. Patent ductus
arteriosus is heard in both systole and diastole.
TMP14. pp. 289–290
123. C) Aortic stenosis has a very high ventricular systolic pressure.
Diastolic filling of the ventricle requires a much higher left atrial
pressure. However, tricuspid stenosis and regurgitation, pulmonary
valve regurgitation, and pulmonary stenosis are associated with an
increase in right atrial pressure and should not affect pressure in the left
atrium.
TMP14 pp. 286–287
124. B) This patient has a QRS axis of −45 degrees, indicating a leftward
axis shift. In other words, the left side of the heart is enlarged. In aortic
valve stenosis, the left side of the heart is enlarged because of the extra
tension the left ventricular walls must exert to expel blood out the aorta.
Therefore, these symptoms fit with a patient with aortic stenosis. In
pulmonary valve stenosis, the right side of the heart hypertrophies, and
in mitral valve stenosis, there is no left ventricular hypertrophy. In
tricuspid valve regurgitation, the right side of the heart enlarges, and in
tricuspid valve stenosis, no ventricular hypertrophy occurs.
TMP14 pp. 286–287
125. C) This patient has a heart murmur heard maximally in the
“pulmonary area of cardiac auscultation.” The high pitch indicates
regurgitation. The rightward axis shift indicates that the right side of the
heart has hypertrophied. The two choices that have a rightward axis
shift are pulmonary valve regurgitation and tetralogy of Fallot. In
tetralogy of Fallot, the arterial blood oxygen content is low, which is not
the case with this patient. Therefore, pulmonary valve regurgitation is
the correct answer.
TMP14 pp. 283–288
126. A) Right ventricular hypertrophy occurs when the right heart has to
pump a higher volume of blood or pump it against a higher pressure.
Tetralogy of Fallot is associated with right ventricular hypertrophy
because of the increased pulmonary valvular resistance, and this also
occurs during pulmonary artery stenosis. Tricuspid insufficiency causes
an increased stroke volume by the right heart, which causes
hypertrophy. However, tricuspid stenosis does not affect the right
ventricle.
TMP14 pp. 290–291
127. E) Mitral stenosis is heard during diastole only. Aortic stenosis,
tricuspid valve regurgitation, interventricular septal effect, and patent
ductus arteriosus are clearly heard during systole. However, patent
ductus arteriosus is also heard during diastole.
TMP14 p. 286
128. A) In tetralogy of Fallot, there is an interventricular septal defect as
well as stenosis of either the pulmo nary artery or the pulmonary valve.
Therefore, it is very difficult for blood to pass into the pulmonary artery
and into the lungs to be oxygenated. Instead the blood partially shunts
to the left side of the heart, thus bypassing the lungs. This situation
results in low arterial oxygen content.
TMP14 pp. 290–291
129. B) The first heart sound by definition is always associated with the
closing of the A-V valves. The heart sounds are usually not associated
with opening of any of the valves but with the closing of the valves and
the associated vibration of the blood and the walls of the heart. One
exception is an opening snap in some mitral valves.
TMP14 p. 283
130. B) In tetralogy of Fallot, an interventricular septal defect and increased
resistance in the pulmonary valve or pulmonary artery cause partial
blood shunting toward the left side of the heart without going through
the lungs. This situation results in a severely decreased arterial oxygen
content. The interventricular septal defect causes equal systolic
pressures in both cardiac ventricles, which causes right ventricular
hypertrophy and a wall thickness very similar to that of the left
ventricle.
TMP14 p. 290
131. C) Mitral regurgitation and aortic stenosis are murmurs heard during
the systolic period. A ventricular septal defect murmur is normally
heard only during the systolic phase. Tricuspid valve stenosis and
patent ductus arteriosus murmurs are heard during diastole. However,
a patent ductus arteriosus murmur is also heard during systole.
TMP14 pp. 286, 289–290
132. E) The third heart sound is associated with inrushing of blood into the
ventricles in the early to middle part of diastole. The next heart sound,
the fourth heart sound, is caused by inrushing of blood in the ventricles
caused by atrial contraction. The first heart sound is caused by the
closing of the A-V valves, and the second heart sound is caused by the
closing of the pulmonary and aortic valves.
TMP14 pp. 283–284
133. A) A number of things occur in progressive shock, including increased
capillary permeability, which allows fluid to leak out of the vasculature,
thus decreasing the blood volume. Other deteriorating factors include
vasomotor center failure, peripheral circulatory failure, decreased
cellular mitochondrial activity, and acidosis throughout the body.
Usually, urine output strikingly decreases; therefore, the increased
urinary output answer is incorrect. Tissue pH decreases, and reverse
stress relaxation of the veins occurs.
TMP14 pp. 296–298
134. A) Sympathomimetic drugs are given to counteract hypotension
during a number of conditions. These conditions include spinal cord
injury in which the sympathetic output is interrupted.
Sympathomimetic drugs are also given during very deep anesthesia,
which decreases the sympathetic output, and during anaphylactic shock
that results from histamine release and the accompanying
vasodilatation. Sympathomimetic drugs, such as norepinephrine,
increase blood pressure by causing a vasoconstriction. Shock caused by
excess vomiting, hemorrhage, or excessive administration of diuretics
results in fluid volume depletion, resulting in decreased blood volume
and decreased mean systemic filling pressure. Administering a balanced
electrolyte solution best counteracts this condition.
TMP14 pp. 299–301
135. D) Too deep a level of anesthesia can decrease sympathetic tone and
reduce arterial pressure enough to induce shock. To replace the
sympathetic tone that was lost, the optimal therapy is infusion of a
sympathomimetic drug. Infusion of red blood cells, plasma, or
electrolytes would be of lile benefit.
TMP14 pp. 299–301
136. D) The patient received an influenza inoculation and quickly went
into shock, which leads one to believe that he may be in anaphylactic
shock. Anaphylactic shock is a state of extreme vasodilation because of
histamine release. Antihistamines would be somewhat helpful, but they
are very slow acting, and the patient could die in the meantime.
Therefore, a very rapid-acting agent must be used, such as a
sympathomimetic drug.
TMP14 pp. 299–301
137. E) In compensated hemorrhagic shock, a number of factors prevent
the progression of the shock, including increased heart rate. Also
occurring is reverse stress relaxation in which the vasculature,
particularly the veins, constrict around the available blood volume.
Increased ADH release also occurs, which causes water retention from
the kidney but also vasoconstriction of the arterioles. A CNS ischemic
response also occurs if the blood pressure drops to very low values,
causing an increase in sympathetic output. Increased absorption of
interstitial fluid through the capillaries also occurs, which increases the
volume in the vasculature.
TMP14 p. 295
138. E) Spinal anesthesia, especially when the anesthesia extends all the
way up the spinal cord, can block the sympathetic nervous outflow from
the spinal cord. This can be a very potent cause of neurogenic shock.
The therapy of choice is to replace the sympathetic tone that was lost in
the body. The best way to in crease the sympathetic tone is by infusing a
sympathomimetic drug.
TMP14 pp. 299–301
139. A) This patient has obviously lost a lot of blood because of the
motorcycle wreck. The most advantageous therapy is to replace what
was lost in the accident. This would be whole blood, which is much
superior to a plasma infusion, because the patient is also receiving red
blood cells that have a much superior oxygen-carrying capacity than the
plasma component of blood. Sympathetic nerves are firing very rapidly
in this condition, and an infusion of a sympathomimetic agent would be
of lile advantage.
TMP14 p. 301
140. C) In hemorrhagic shock, anaphylactic shock, and neurogenic shock,
the venous return of blood to the heart markedly decreases. However, in
septic shock, the cardiac output increases in many patients because of
vasodilation in affected tissues and a high metabolic rate causing
vasodilation in other parts of the body.
TMP14 p. 300
141. E) This patient has been hemorrhaging, and the optimal therapy is to
replace the blood he has lost. Unfortunately, no blood is available, and
therefore we must choose next best therapy, which is increasing the
volume of his blood. Thus, plasma infusion is the next best therapy
because its high colloid osmotic pressure will help the infused fluid stay
in the circulation much longer than would a balanced electrolyte
solution.
TMP14 p. 301
142. B) Intestinal obstruction often causes severe reduction in plasma
volume. Obstruction causes a distention of the intestine and partially
blocks the venous blood flow in the intestines. This partial blockage
results in an increased intestinal capillary pressure, which causes fluid
to leak from the capillary into the walls of the intestines and into the
intestinal lumen. The leaking fluid has a high protein content very
similar to that of the plasma, which reduces the total plasma protein and
the plasma volume. Therefore, the therapy of choice would be to replace
the fluid lost by infusing plasma.
TMP14 pp. 299, 301
143. A) In progressive shock, because of the poor blood flow, the pH in the
tissues throughout the body decreases. Many vessels become blocked
because of local blood agglutination, which is called “sludged blood.”
Patchy areas of necrosis also occur in the liver. Mitochondrial activity
decreases and capillary permeability increases. There is also an
increased release of hydrolases by the lysosomes and a decrease in
cellular metabolism of glucose.
TMP14 pp. 296–298
144. A) Anaphylaxis is an allergic condition that results from an antigenantibody reaction that takes place after exposure of an individual to an
antigenic substance. The basophils and mast cells in the pericapillary
tissues release histamine or histamine-like substances. The histamine
causes venous dilation, dilation of arterioles, and greatly increased
capillary permeability with rapid loss of fluid and protein into the tissue
spaces. This response reduces venous return and often results in
anaphylactic shock.
TMP14 pp. 300–301
145. B) This patient has a resting cardiac output of 4.8 l/min, and a cardiac
output of 19.2 l/min after maximum exercise. The cardiac reserve [(19.2 –
4.8)/4.8 =3.0)] is the percentage increase that the cardiac output can be
elevated over the resting cardiac output. Therefore, her cardiac reserve is
300% (3-fold) greater than her resting cardiac output.
TMP14 pp. 277–278
146. C) Immediately after heart damage, a compensatory activation of the
sympathetic nervous system develops, which helps aenuate the
decreased cardiac output (heart muscle is damaged) and improve
cardiovascular dynamics (e.g. mean systemic filling pressures, venous
return). Damming of blood develops in the failing heart due to
depressed contraction. Parasympathetics are not activated as a
compensatory mechanism. The renin-angiotensin system activates later.
TMP14 pp. 271–273
147. B) Cardiac reserve is always reduced in heart failure, even when
compensatory mechanisms and/or treatments have been effective in
restoring cardiovascular hemodynamics at rest. Cardiac failure is the
failure of the heart to pump enough blood to satisfy the needs of the body,
which may develop with low or high cardiac output. A low cardiac
output will result in low blood flow into the kidneys, reduced
glomerular filtration, and a low urinary output.
TMP14 pp. 271–273, 277–278
148. C) The pumping ability of the heart is compromised in low-output
heart failure. Moderate fluid retention will improve venous return and
“prime” the partially damaged heart in order to increase cardiac output.
Based on the Frank-Starling law of the heart (the student should be
familiar with this mechanism), this will result in an increased preload of
the heart. Afterload and isovolumetric contraction are unchanged.
Aortic pressure would be preserved. Peripheral edema may develop if
fluid retention progresses.
TMP14 p. 272
149. A) The severe damage of the heart and the failure to increase CO
enough to satisfy the needs of the body will lead to a progressive fluid
retention (constant low flow to the kidneys will continuously activate
mechanisms of fluid retention), increased mean filling pressures, and
increased right atrial pressure. The progressive fluid retention in an
already weak heart will lead to a deleterious cycle that will progresses
until the heart is so overstretched and/or edematous that it fails
completely.
TMP14 pp. 273–274, 279–280
150. C) Mitral regurgitation produces a systolic murmur. The history of
dilated cardiomyopathy suggests mitral regurgitation. The lack of
cyanosis and age of the patients eliminates tetralogy of Fallot as a likely
diagnosis. The characteristics of the murmur eliminates patent ductus
arteriosus (machinery murmur), mitral and tricuspid stenosis (diastolic
murmurs).
TMP14 pp. 286, 288
151. D) The diastolic murmur of mitral stenosis is due to the narrow
opening of the mitral valve, making it difficult for the blood to move
from the left atria to the left ventricle. A consequence of mitral stenosis is
an increase in pressure of pulmonary vessels, but this is not a cause of
the murmur.
TMP14 pp. 286, 288
152. C) Abnormalities of the aortic valve (outflow tract of the left ventricle)
associates with significant increases in LV pressure and as such,
increases in afterload (which does not occur in mitral
stenosis/regurgitation). Preload is unchanged in aortic stenosis.
Isovolumetric systolic period is lost in aortic and in mitral regurgitation.
Tricuspid stenosis does not associate with loss of left ventricular
isovolumetric diastolic period.
TMP14 pp. 286–287
153. B) A right-to-left shunt as observed in tetralogy of Fallot results in
almost 2/3 of the blood passing from the right ventricle into aorta
without oxygenation, bypassing the lungs (right-to-left shunt). As a
result, oxygen in the blood is low and the infant develops cyanosis (blue
baby) and rapidly shows a significant enlargement of the right ventricle.
Patent ductus arteriosus (a left-to-right shunt) will not show cyanosis at
this stage and the blood is usually hyper-oxygenated. An interatrial
septal defect associates with a left-to-right shunt. Tricuspid stenosis
does not cause low oxygen in blood or associate with enlargement of the
right ventricle.
TMP14 pp. 289–290
154. A) Without intervention (i.e. volume replacement) or after entering
into the progressive phase, cardiac function progressively declines,
which constitutes the most important factor for the progression towards
irreversibility of shock. Autoregulation cannot prevent (or reverse) the
irreversible phase of shock.
TMP14 pp. 295–297
155. C) Based on the information included that states a massive heart
aack, a significant delay to initiate treatment, and the lack of responses
to treatments, the patient is most likely at the irreversible stage of shock
from cardiogenic origin (not hemorrhagic). More fluids will not change
the course of shock at this stage. Diminished delivery of oxygen will
lead to tissue acidosis, and tissue ischemia facilitates release of toxins
that further contribute to the generalized cellular deterioration.
TMP14 pp. 296–299
156. C) The patient is in ventricular fibrillation, thus in cardiac/circulatory
arrest. There is no effective mechanical activity in the left ventricle.
TMP14 pp. 301–302
157. D) Dextran is a large polysaccharide that do not pass through the
capillary pores and serves as a colloid osmotic agent (as do plasma
proteins). Anti-histamine agents target histamine, which is massively
released in anaphylactic shock. Glucocorticoids stabilize lysosomes and
help to prevent their disruption and, consequently, the release of
enzymes to the cellular cytoplasm.
TMP14 p. 301
Unit V: The Body Fluids and
Kidneys
1. All of the following changes would tend to cause interstitial fluid
edema in a tissue EXCEPT one. Which one is the EXCEPTION?
A) Increased venous resistance
B) Increased venous pressure
C) Decreased arteriolar resistance
D) Increased capillary filtration coefficient
E) Increased plasma protein concentration
2. Calculate the approximate extracellular fluid osmolarity of a patient
after administration of 2.0 l of 5% glucose solution, assuming
complete metabolism of the glucose, osmotic equilibrium and no
excretion of water or electrolytes. Also assume the following initial
conditions prior to infusing the glucose solution:
Body weight = 50 kg
Plasma sodium concentration = 170 mmol/l
Plasma osmolarity = 360 mOsm/l
Intracellular fluid volume = 40% of body weight
Extracellular fluid volume = 20% of body weight
Molecular wt of glucose = 180 g/mole
A) 264 mOsm/l
B) 282 mOsm/l
C) 306 mOsm/l
D) 319 mOsm/l
E) 338 mOsm/l
F) 355 mOsm/l
G) 360 mOsm/l
3. Calculate the approximate extracellular fluid volume of a patient after
administration of 3.0 l of 5% glucose solution, assuming complete
metabolism of the glucose, osmotic equilibrium, and no excretion of
water or electrolytes. Also assume the following initial conditions
prior to infusing the glucose solution:
Body weight = 50 kg
Plasma sodium concentration = 170 mmol/l
Plasma osmolarity = 360 mOsm/l
Intracellular fluid volume = 40% of body weight
Extracellular fluid volume = 20% of body weight
Molecular wt of glucose = 180 g/mole
A) 8.0 l
B) 10.7 l
C) 11.7 l
D) 130 l
E) 20.3 l
F) 21.3 l
G) 30.0 l
4. A patient with cirrhosis experiences a doubling of his serum
creatinine over a 6-month period after sustained heavy ingestion of a
nonsteroidal antiinflammatory drug (NSAID) for his arthritis. Which
of the following is the best explanation for his increased serum
creatinine?
A) Increased efferent arteriolar resistance which reduced
glomerular filtration rate (GFR)
B) Decreased Bowman’s capsule pressure which reduced GFR
C) Increased afferent arteriolar resistance which reduced GFR
D) Increased glomerular capillary filtration coefficient which
reduced GFR
E) Increased renal prostaglandins due to the NSAID
F) Increased nitric oxide formation due to the NSAID
5. Administration of empagliflozin, an inhibitor of sodium-glucose cotransporter 2 (SGLT2), would be expected to cause which of the
following sets of changes compared with normal?
GFR Resistance Afferent Arteriole Renal Blood Flow
A)
B)
C)
D)
E)
F)
↔
↔
↓
↓
↓
↑
↔
↔
↑
↑
↓
↓
↔
↓
↓
↔
↓
↑
6. Given the following measurements, calculate the approximate
filtration fraction:
Glomerular capillary hydrostatic pressure = 60 mm Hg
Colloid osmotic pressure in the glomerular capillaries = 30 mm
Hg
Bowman’s space hydrostatic pressure = 20 mm Hg
Glomerular capillary filtration coefficient (Kf) = 10 ml/min/mm
Hg
Renal plasma flow = 600 ml/min
Hematocrit = 0.4
A) 10 mm Hg
B) 100 ml/min
C) 0.100
D) 0.167
E) 0.200
F) 0.333
7. Which of the following statements is incorrect?
A) Creatinine concentration in the urine is normally higher than in
the glomerular filtrate.
B) Urea concentration in the urine is normally higher than in the
glomerular filtrate.
C) The proximal tubules normally reabsorb almost all of the
glucose filtered by the glomerular capillaries.
D) concentration in the urine is normally higher than in the
glomerular filtrate.
E) Organic acids and bases are secreted mainly by the proximal
tubules.
F) Sodium concentration remains relatively constant as tubular
fluid flows along the proximal tubule.
8. A patient with diabetes mellitus has a glomerular filtration rate of 100
ml/min, a urine flow rate of 4.0 ml/min, and a urine glucose
concentration of 2 mg/ml. If he has a kidney transport maximum for
glucose of 200 mg/min, what would be his approximate rate of
glucose excretion?
A) 0 mg/min
B) 8 mg/min
C) 100 mg/min
D) 180 mg/min
E) 300 mg/min
F) Urinary excretion rate of glucose cannot be determined from
these data
9. As tubular fluid passes along a juxtamedullary nephron of a person
with severe central diabetes insipidus and essentially no antidiuretic
hormone, where is the osmolarity lowest?
A) Bowman’s capsule (glomerular filtrate)
B) Fluid leaving the proximal tubule and entering the loop of Henle
C) Fluid leaving the descending thin limb and entering the
ascending thin limb of the loop of Henle
D) Fluid leaving the thick ascending segment of the loop of Henle
and entering the early distal tubule
E) Fluid in the cortical collecting tubules
F) Fluid leaving the collecting ducts (urine)
10. If GFR = 60 ml/min, urine flow rate = 2.0 ml/min, plasma K+
concentration = 4.0 mmol/l, and urine K+
concentration = 80 mmol/l,
what is the approximate rate of K+
excretion?
A) 0.08 mmol/min
B) 0.16 mmol/min
C) 0.32 mmol/min
D) 16 mmol/min
E) 160 mmol/min
F) Excretion rate of K+
cannot be determined from these data
11. If the glomerular filtration rate (GFR) of a patient is reduced to 50% of
normal and sustained at that level, you would expect to find
__________ renal creatinine excretion rate, __________ renal
creatinine clearance, and __________ serum creatinine concentration
6 weeks after the decrease in GFR compared with normal. Assume
steady-state conditions and that the patient has maintained the same
diet.
A) Decreased, decreased, increased
B) Decreased, no change, increased
C) No change, increased, increased
D) No change, no change, increased
E) No change, decreased, increased
F) Decreased, no change, decreased
12. If glomerular filtration rate suddenly decreases by 50%, from 80
ml/min to 40 ml/min and tubular fluid reabsorption simultaneously
decreases from 78 ml/min to 40 ml/min, which of the following
changes in urinary excretion rate will occur (assuming that the
changes in GFR and tubular fluid reabsorption are maintained)?
A) Urine flow rate will decrease to zero
B) Urine flow rate will not change
C) Urine flow rate will decrease by 50%
D) Urine flow rate will increase by 50%
13. Calculate the approximate total renal plasma flow given the
following data:
Urine PAH concentration = 200 µg/ml
Urine flow rate = 2 ml/min
Arterial plasma paraaminohippuric acid (PAH) concentration =
1.0 µg/ml
Renal venous PAH concentration = 0.2 µg/ml
Hematocrit = 0.4
A) 120 ml/min
B) 200 ml/min
C) 400 ml/min
D) 500 ml/min
E) 667 ml/min
F) 833 ml/min
14. Which of the following occurs in type A intercalated cells of the
collecting tubules?
A) Secretion of H+
, reabsorption of , and reabsorption of K+
B) Secretion of H+
, reabsorption of , and secretion of K+
C) Secretion of K+
, reabsorption of Na+
, and reabsorption of
D) Reabsorption of H+
, secretion of , and secretion of K+
E) Reabsorption of H+
, secretion of , and reabsorption of K+
15. Which of the following statements is incorrect?
A) Beta-adrenergic stimulation would tend to cause hypokalemia
by shifting potassium from the extracellular fluid into the cells
B) A powerful diuretic that inhibits proximal tubule or loop of
Henle sodium reabsorption would tend to increase potassium
secretion by the collecting tubules
C) Injection of excess insulin into a patient would tend to cause
hypokalemia
D) Strenuous, sustained exercise may tend to cause significant
hyperkalemia
E) Increased extracellular fluid osmolarity would tend to cause
hypokalemia
16. Which of the following statements is incorrect?
A) Urea transporters UT-A1 and UTA-3 in the collecting ducts are
activated by antidiuretic hormone (ADH)
B) Urea reabsorption in the inner medullary collecting duct is
greater than in the distal tubule during dehydration
C) Increased ADH markedly increases urea reabsorption by the
cortical collecting tubule
D) The inner medullary collecting tubule reabsorbs more urea
during antidiuresis than the thick ascending limb of Henle’s loop
E) The cortical collecting tubule is less permeable to urea than is
the inner medullary collecting duct during antidiuresis
F) Passive diffusion of urea into the thin loops of Henle is
facilitated by the urea transporter UT-A2
17. In a dehydrated person with normal kidneys and high ADH levels,
which part of the nephron normally reabsorbs the smallest amount of
water (see figure below of a renal tubule)?
A) A
B) B
C) C
D) D
E) E
F) F
18. Acute metabolic alkalosis tends to ______K+
secretion by the cortical
collecting tubules and ______ plasma K+
concentration.
A) decrease, decrease
B) decrease, increase
C) increase, increase
D) increase, decrease
E) cause no change in, increase
F) cause no change in, cause no change in
19. Which of the following statements is incorrect?
A) Carbonic anhydrase inhibitors tend to cause metabolic acidosis.
B) Thiazide diuretics inhibit the Na-Cl co-transporter in the distal
tubules.
C) Osmotic diuretics tend to increase potassium secretion.
D) Aldosterone antagonists (e.g., spironolactone) tend to cause
hypokalemia.
E) Sodium channel blockers (e.g., amiloride) inhibit sodium
transport across the luminal membrane of the collecting tubules.
F) Loop diuretics (e.g., furosemide) tend to cause hypokalemia.
20. Glomerular filtration rate (creatinine clearance) in a patient with
uncontrolled type 2 diabetes has decreased from 100 ml/min to 50
ml/min over the past 4 years. She has poorly controlled hypertension,
and her plasma pH is 7.16. Which of the following changes,
compared with before she developed renal disease, would you expect
to find, assuming steady-state conditions and no changes in protein
or electrolyte intake?
Sodium
Excretion
Rate
Creati-nine
Excretion Rate
Plasma Creati-nine
Concen-tration
Plasma Bicarbonate
Concen-tration
Ammo-nium
Excre-tion Rate
A)
B)
C)
D)
E)
F)
↓
↓
↔
↔
↔
↓
↓
↓
↔
↔
↓
↓
↑
↓
↑
↑
↑
↓
↑
↓
↓
↓
↓
↓
↑
↑
↑
↔
↔
↓
21. A patient has the following laboratory values:
Arterial pH = 7.18
Plasma
Plasma chloride concentration = 100 mEq/l
Arterial = 28 mm Hg
Plasma Na+
concentration = 141 mEq/l
What is the most likely cause of his acid-base disturbance?
A) Emphysema
B) Renal tubular acidosis
C) Severe diarrhea
D) Methanol poisoning
E) Ingestion of excess sodium bicarbonate
22. Atrial natriuretic peptide causes which of the following effects?
A) Reduced renal tubular sodium reabsorption
B) Reduced renin secretion
C) Increased renal sodium excretion
D) Only A and C
E) A, B, and C
23. If creatinine clearance = 100 ml/min, urine flow rate = 1.0 ml/min,
plasma Na+
concentration = 140 mmol/l, and urine Na+
concentration
= 80 mmol/l, what is the approximate rate of Na+
excretion?
A) 0.08 mmol/min
B) 0.16 mmol/min
C) 16 mmol/min
D) 160 mmol/min
E) Excretion rate of Na+
cannot be calculated from these data
24. If a person maintains a high (150 mmol/day) potassium diet, which
part of the nephron would be expected to reabsorb the most
potassium? Choose the appropriate nephron site in the figure below.
A) A
B) B
C) C
D) D
E) E
F) F
25. Which of the following would tend to cause hypokalemia by shifting
potassium from the extracellular fluid into the intracellular fluid?
A) Metabolic alkalosis
B) Insulin deficiency
C) Aldosterone deficiency
D) Beta-adrenergic receptor blockade
E) Increased extracellular fluid osmolarity
26. What is the theoretical maximum clearance rate possible for a
substance X that is freely filtered, actively secreted by the renal
tubules, and completely cleared from the plasma given the following
data?
Glomerular filtration rate = 100 ml/min
Plasma concentration of a substance X = 2 mg/ml
Urine flow= 5 ml/min
Renal plasma flow = 800 ml/min
A) 5 ml/min
B) 100 ml/min
C) 200 ml/min
D) 500 ml/min
E) 800 ml/min
F) 1000 ml/min
27. Which of the following solutions, when infused intravenously,
would result in an increase in extracellular fluid volume, a decrease
in intracellular fluid volume, and an increase in total body water after
osmotic equilibrium?
A) 1 l of 0.9% sodium chloride (NaCl) solution
B) 1 l of 0.45% NaCl solution
C) 1 l of 3% NaCl solution
D) 1 l of 5% dextrose solution
E) 1 l of pure water
28. Partial obstruction of a major vein draining a tissue would tend to
__________ lymph flow rate, __________ interstitial fluid hydrostatic
pressure, and __________ interstitial fluid protein concentration in
the tissue drained by that vein.
A) increase, increase, increase
B) increase, increase, decrease
C) increase, decrease, decrease
D) decrease, decrease, decrease
E) decrease, increase, increase
F) decrease, increase, decrease
29. A 36-year-old woman reports headaches and frequent urination.
Laboratory values reveal the following information.
Urine specific gravity = 1.003
Urine protein = negative
Plasma sodium (Na+) = 165 mmol/l
Plasma potassium (K+) = 4.4 mmol/l
Plasma creatinine = 1.4 mg/dl
Blood pressure = 88/40 mm Hg
Heart rate = 115 beats/min
What is the most likely cause of her elevated plasma Na+
concentration?
A) Primary aldosteronism
B) Diabetes mellitus
C) Diabetes insipidus
D) Simple dehydration caused by insufficient water intake and heavy
exercise
E) Barer syndrome
F) Liddle syndrome
30. After receiving a kidney transplant, a patient has severe
hypertension (170/110 mm Hg). A renal arteriogram indicates severe
renal artery stenosis in his single remaining kidney, with a reduction
in glomerular filtration rate (GFR) to 25% of normal. Which of the
following changes, compared with normal, would be expected in this
patient, assuming steady-state conditions?
A) A large increase in plasma sodium concentration
B) A reduction in urinary sodium excretion to 25% of normal
C) A reduction in urinary creatinine excretion to 25% of normal
D) An increase in serum creatinine to about four times normal
E) Normal renal blood flow in the stenotic kidney due to
autoregulation
Questions 31–33
The figure above represents various states of abnormal hydration. In each
diagram, the normal state (orange and lavender) is superimposed on the
abnormal state (dashed lines) to illustrate the shifts in the volume (width of
rectangles) and total osmolarity (height of rectangles) of the extracellular and
intracellular fluid compartments. Use this figure to answer Questions 31–33.
31. Which diagram represents the changes (after osmotic equilibrium) in
extracellular and intracellular fluid volume and osmolarity after the
infusion of 1% dextrose?
A) A
B) B
C) C
D) D
32. Which diagram represents the changes (after osmotic equilibrium) in
extracellular and intracellular fluid volume and osmolarity in a
patient with the syndrome of inappropriate antidiuretic hormone
(ADH; i.e., excessive secretion of ADH)?
A) A
B) B
C) C
D) D
33. Which diagram represents the changes (after osmotic equilibrium) in
extracellular and intracellular fluid volumes and osmolarities after
the infusion of 3% NaCl?
A) A
B) B
C) C
D) D
34. Which of the following tends to decrease potassium secretion by the
cortical collecting tubule?
A) Increased plasma potassium concentration
B) A diuretic that decreases proximal tubule sodium reabsorption
C) A diuretic that inhibits the action of aldosterone (e.g.,
spironolactone)
D) Acute alkalosis
E) High sodium intake
35. Because the usual rate of phosphate filtration exceeds the transport
maximum for phosphate reabsorption, which statement is true?
A) All the phosphate that is filtered is reabsorbed
B) More phosphate is reabsorbed than is filtered
C) Phosphate in the tubules can contribute significantly to titratable
acid in the urine
D) The “threshold” for phosphate is usually not exceeded
E) Parathyroid hormone must be secreted for phosphate
reabsorption to occur
Questions 36 and 37
Use the following clinical laboratory test results to answer Questions 36
and 37.
Urine flow rate = 1 ml/min
Urine inulin concentration = 100 mg/ml
Plasma inulin concentration = 2 mg/ml
Urine urea concentration = 50 mg/ml
Plasma urea concentration = 2.5 mg/ml
36. What is the GFR?
A) 25 ml/min
B) 50 ml/min
C) 100 ml/min
D) 125 ml/min
E) None of the above
37. What is the net urea reabsorption rate?
A) 0 mg/min
B) 25 mg/min
C) 50 mg/min
D) 75 mg/min
E) 100 mg/min
38. In normal kidneys, which of the following is true of the osmolarity of
renal tubular fluid that flows through the early distal tubule in the
region of the macula densa?
A) Usually isotonic compared with plasma
B) Usually hypotonic compared with plasma
C) Usually hypertonic compared with plasma
D) Hypertonic, compared with plasma, in antidiuresis
39. Which of the following changes would be expected in a patient with
diabetes insipidus due to a lack of ADH secretion, assuming free
access to water and normal thirst mechanisms for controlling water
intake?
Plasma Osmolarity
Concentration
Plasma Sodium
Concentration
Plasma Renin
Concentration
Urine
Volume
A) ↔ ↔ ↓ ↑
B) ↔ ↔ ↑ ↑
C) ↑ ↑ ↑ ↑
D) ↑ ↑ ↔ ↔
E) ↓ ↓ ↓ ↔
40. A 26-year-old woman recently decided to adopt a healthier diet and
eat more fruits and vegetables. As a result, her potassium intake
increased from 80 to 160 mmol/day. Which of the following
conditions would you expect to find 2 weeks after she increased her
potassium intake, compared with before the increase?
Potassium Excretion
Rate
Sodium Excretion
Rate
Plasma Aldosterone Concentration
Plasma Potassium
Concentration
A) ↔ ↔ ↑ Large increase (>1 mmol/l)
B) ↔ ↓ ↑ Small increase (<1 mmol/l)
C) ↑ 2× ↔ ↑ Small increase (<1 mmol/l)
D) ↑ 2× ↑ ↓ Large increase (>1 mmol/l)
E) ↑ 2× ↑ ↔ Large increase (>1 mmol/l)
41. When the dietary intake of K+
increases, body K+
balance is
maintained by an increase in K+
excretion primarily by which of the
following?
A) Decreased glomerular filtration of K+
B) Decreased reabsorption of K+
by the proximal tubule
C) Decreased reabsorption of K+
by the thick ascending limb of the
loop of Henle
D) Increased K+
secretion by the late distal and collecting tubules
E) Shift of K+
into the intracellular compartment
42. Which of the following would cause the greatest decrease in GFR in
a person with otherwise normal kidneys?
A) Decrease in renal arterial pressure from 100 to 80 mm Hg in a
normal kidney
B) 50% increase in glomerular capillary filtration coefficient
C) 50% increase in proximal tubular sodium reabsorption
D) 50% decrease in afferent arteriolar resistance
E) 50% decrease in efferent arteriolar resistance
F) 5 mm Hg decrease in Bowman’s capsule pressure
43. An 8-year-old boy is brought to your office with extreme swelling of
the abdomen. His parents indicate that he had a very sore throat a
“month or so” ago and that he has been “swelling up” since that
time. He appears to be edematous, and when you check his urine,
you find that large amounts of protein are being excreted. Your
diagnosis is nephrotic syndrome subsequent to glomerulonephritis.
Which of the following changes would you expect to find, compared
with normal?
Thoracic Lymph
Flow
Interstitial Fluid Protein
Concen-tration
Interstitial Fluid Hydrostatic
Pressure
Plasma Renin
Concentration
A) ↑ ↓ ↑ ↑
B) ↑ ↓ ↑ ↔
C) ↑ ↓ ↔ ↑
D) ↓ ↑ ↔ ↔
E) ↓ ↓ ↓ ↓
44. A patient with severe hypertension (blood pressure 185/110 mm Hg)
is referred to you. A renal magnetic resonance imaging scan shows a
tumor in the kidney, and laboratory findings include a very high
plasma renin activity of 12 ng angiotensin I/ml/h (normal = 1). The
diagnosis is a renin-secreting tumor. Which of the following changes
would you expect to find in this patient, under steady-state
conditions, compared with normal?
Plasma Aldosterone Concentration
Sodium Excretion
Rate
Plasma Potassium
Concentration
Renal Blood
Flow
A) ↔ ↓ ↓ ↑
B) ↔ ↔ ↓ ↑
C) ↑ ↔ ↓ ↓
D) ↑ ↓ ↔ ↓
E) ↑ ↓ ↓ ↔
45. The clinical laboratory returned the following values for arterial
blood taken from a patient: plasma pH = 7.28, plasma ,
and plasma partial pressure of carbon dioxide ( ) = 70 mm Hg.
What is this patient’s acid-base disorder?
A) Acute respiratory acidosis without renal compensation
B) Respiratory acidosis with partial renal compensation
C) Acute metabolic acidosis without respiratory compensation
D) Metabolic acidosis with partial respiratory compensation
46. The following laboratory values were obtained in a 58-year-old man:
Urine volume = 4320 ml of urine collected during the preceding
24 hours
Plasma creatinine = 3 mg/100 ml
Urine creatinine = 50 mg/100 ml
Plasma potassium = 4.0 mmol/l
Urine potassium = 30 mmol/l
What is his approximate GFR, assuming that he collected all of his urine
in the 24-hour period?
A) 20 ml/min
B) 30 ml/min
C) 40 ml/min
D) 50 ml/min
E) 60 ml/min
F) 80 ml/min
G) 100 ml/min
Questions 47 and 48
A 65-year-old man had a heart aack and experiences cardiopulmonary
arrest while being transported to the emergency department. Use the
following laboratory values obtained from arterial blood to answer
Questions 47 and 48.
Plasma pH = 7.12
Plasma = 60 mm Hg
Plasma concentration = 19 mEq/l
47. Which of the following options best describes his acid-base disorder?
A) Respiratory acidosis with partial renal compensation
B) Metabolic acidosis with partial respiratory compensation
C) Mixed acidosis: combined metabolic and respiratory acidosis
D) Mixed alkalosis: combined respiratory and metabolic alkalosis
48. In this patient, which of the following laboratory results would be
expected, compared with normal?
A) Increased renal excretion of bicarbonate ( )
B) Decreased urinary titratable acid
C) Increased urine pH
D) Increased renal excretion of ammonia (NH4
+)
49. What would cause the greatest degree of hyperkalemia?
A) Increase in potassium intake from 60 to 180 mmol/day in a
person with normal kidneys and a normal aldosterone system
B) Chronic treatment with a diuretic that inhibits the action of
aldosterone
C) Decrease in sodium intake from 200 to 100 mmol/day
D) Chronic treatment with a diuretic that inhibits loop of Henle
Na+-2Cl − -K+
co-transport
E) Chronic treatment with a diuretic that inhibits sodium
reabsorption in the collecting ducts
50. Which of the following changes would be expected in a patient with
Liddle syndrome (i.e., excessive activity of amiloride-sensitive
sodium channel in the collecting tubule) under steady-state
conditions, assuming that the intake of electrolytes remained
constant?
Plasma Renin Concentration
Blood
Pressure
Sodium Excretion
Concentration
Plasma
AldosteroneConcentration
A) ↔ ↑ ↓ ↔
B) ↑ ↑ ↔ ↑
C) ↑ ↑ ↓ ↓
D) ↓ ↑ ↔ ↓
E) ↓ ↑ ↓ ↓
F) ↓ ↓ ↑ ↑
51. A patient is referred for treatment of hypertension. After testing, you
discover that he has a very high level of plasma aldosterone, and
your diagnosis is Conn’s syndrome. Assuming no change in
electrolyte intake, which of the following changes would you expect
to find, compared with normal?
Plasma
pH
Plasma K+ Concentration
Urine K+
Excretion
Urine Na
+
Excretion
Plasma Renin Concentration
A) ↑ ↓ ↔ ↔ ↓
B) ↓ ↓ ↔ ↔ ↓
C) ↑ ↓ ↑ ↓ ↓
D) ↑ ↑ ↔ ↓ ↑
E) ↑ ↑ ↑ ↑ ↑
52. Which change tends to increase GFR?
A) Increased afferent arteriolar resistance
B) Decreased efferent arteriolar resistance
C) Increased glomerular capillary filtration coefficient
D) Increased Bowman’s capsule hydrostatic pressure
E) Decreased glomerular capillary hydrostatic pressure
53. Which of the following changes, compared with normal, would you
expect to find 3 weeks after a patient ingested a toxin that caused
sustained impairment of proximal tubular NaCl reabsorption?
Assume that there has been no change in diet or ingestion of
electrolytes.
Glomerular Filtration Rate Afferent Arteriolar Resistance Sodium Excretion
A) ↔ ↔ ↑
B) ↔ ↔ ↑
C) ↓ ↑ ↑
D) ↓ ↑ ↔
E) ↑ ↓ ↔
54. A patient has the following laboratory values: arterial pH = 7.13,
plasma , plasma chloride concentration = 118 mEq/l,
arterial = 28 mm Hg, and plasma Na+
concentration = 141 mEq/l.
What is the most likely cause of his acidosis?
A) Salicylic acid poisoning
B) Diabetes mellitus
C) Diarrhea
D) Emphysema
55. The GFR of a 26-year-old man with glomerulonephritis decreases by
50% and remains at that level for one month. For which substance
would you expect to find the greatest increase in plasma
concentration?
A) Creatinine
B) K+
C) Glucose
D) Na+
E) Phosphate
F) H+
56. Which changes would you expect to find after administering a
vasodilator drug that caused a 50% decrease in afferent arteriolar
resistance and no change in arterial pressure?
A) Decreased renal blood flow, decreased GFR, and decreased
peritubular capillary hydrostatic pressure
B) Decreased renal blood flow, decreased GFR, and increased
peritubular capillary hydrostatic pressure
C) Increased renal blood flow, increased GFR, and increased
peritubular capillary hydrostatic pressure
D) Increased renal blood flow, increased GFR, and no change in
peritubular capillary hydrostatic pressure
E) Increased renal blood flow, increased GFR, and decreased
peritubular capillary hydrostatic pressure
57. If the average hydrostatic pressure in the glomerular capillaries is 50
mm Hg, the hydrostatic pressure in the Bowman’s space is 12 mm
Hg, the average colloid osmotic pressure in the glomerular capillaries
is 30 mm Hg, and there is no protein in the glomerular ultrafiltrate,
what is the net pressure driving glomerular filtration?
A) 8 mm Hg
B) 32 mm Hg
C) 48 mm Hg
D) 60 mm Hg
E) 92 mm Hg
58. In a patient who has chronic, uncontrolled diabetes mellitus, which
set of conditions would you expect to find, compared with normal?
Titratable Acid Excretion NH+ Excretion Excretion Plasma
A) ↔ ↑ ↓ ↔
B) ↓ ↑ ↔ ↓
C) ↑ ↑ ↔ ↑
D) ↑ ↑ ↓ ↓
E) ↓ ↓ ↓ ↓
F) ↔ ↑ ↓ ↔
59. Intravenous infusion of 1 l of 0.45% NaCl solution (molecular weight
of NaCl = 58.5) would cause which of the following changes, after
osmotic equilibrium?
Intra-cellular Fluid
Volume
Intra-cellular Fluid
Osmolarity
Extra-cellular Fluid
Volume
Extracellular Fluid
Osmolarity
A) ↑ ↑ ↑ ↑
B) ↑ ↓ ↑ ↓
C) ↔ ↑ ↑ ↑
D) ↓ ↑ ↑ ↑
E) ↓ ↓ ↓ ↓
60. Lines A, B, and C on the figure above show the relative filterability
by the glomerular capillaries of dextran molecules as a function of
their molecular radius and electrical charges. Which lines on the
graph best describe the electrical charges of the dextrans?
A) A = polycationic; B = neutral; C= polyanionic
B) A = polycationic; B = polyanionic; C = neutral
C) A = polyanionic; B = neutral; C = polycationic
D) A = polyanionic; B = polycationic; C = polycationic
E) A = neutral; B = polycationic; C = polyanionic
F) A = neutral; B = polyanionic; C = polycationic
61. If distal tubule fluid creatinine concentration is 5 mg/100 ml and
plasma creatinine concentration is 1.0 mg/100 ml, what is the
approximate percentage of the water filtered by the glomerular
capillaries that remains in the distal tubule?
A) 5%
B) 10%
C) 20%
D) 50%
E) 80%
F) 95%
62. Which change tends to increase peritubular capillary fluid
reabsorption?
A) Increased blood pressure
B) Decreased filtration fraction
C) Increased efferent arteriolar resistance
D) Decreased angiotensin II
E) Increased renal blood flow
63. A 32-year-old man reports frequent urination. He is overweight (280
lb [127 kg], 5 feet 10 inches [178 cm] tall). After measuring the 24-
hour creatinine clearance, you estimate his GFR to be 150 ml/min. His
plasma glucose level is 300 mg/dl. Assuming that his renal transport
maximum for glucose is normal, as shown in the figure above, what
would be this patient’s approximate rate of urinary glucose
excretion?
A) 0 mg/min
B) 100 mg/min
C) 150 mg/min
D) 225 mg/min
E) 300 mg/min
F) Information provided is inadequate to estimate the glucose
excretion rate
64. An adrenal tumor that causes excess aldosterone secretion would
tend to __________ plasma K+
concentration, __________ plasma pH,
__________ renin secretion, and __________ blood pressure.
A) decrease, decrease, decrease, decrease
B) decrease, increase, decrease, increase
C) decrease, decrease, decrease, increase
D) decrease, increase, increase, increase
E) increase, increase, decrease, increase
F) increase, decrease, decrease, increase
65. Which of the following tends to increase potassium secretion by the
cortical collecting tubule?
A) A diuretic that inhibits the action of aldosterone (e.g.,
spironolactone)
B) A diuretic that decreases loop of Henle sodium reabsorption
(e.g., furosemide)
C) Decreased plasma potassium concentration
D) Acute metabolic acidosis
E) Low sodium intake
66. A diabetic patient has chronic renal disease and is referred to your
nephrology clinic. According to his family physician, his creatinine
clearance has decreased from 100 ml/min to 40 ml/min during the
past 4 years. His glucose level has not been well controlled, and his
plasma pH is 7.14. Which changes, compared with before the
development of renal disease, would you expect to find, assuming
steady-state conditions and no change in electrolyte intake?
Sodium
Excretion Rate
Creati-nine
Excretion Rate
Plasma Creati-nine
Concen-tration
Plasma HCO3
−
Concen-tration
NH4
+
Excretion Rate
A) ↓ ↓ ↑ ↑ ↑
B) ↔ ↔ ↑ ↓ ↑
C) ↔ ↔ ↑ ↓ ↔
D) ↔ ↓ ↑ ↓ ↔
E) ↓ ↓ ↓ ↓ ↑
F) ↓ ↓ ↓ ↓ ↓
67. A 20-year-old woman comes to your office because of rapid weight
gain and marked fluid retention. Her blood pressure is 105/65 mm
Hg, her plasma protein concentration is 3.6 g/dl (normal = 7.0), and
she has no detectable protein in her urine. Which changes would you
expect to find, compared with normal?
Thoracic Lymph
Flow
Interstitial Fluid Protein
Concentration
Capillary
Filtration
Interstitial Fluid
Pressure
A) ↓ ↓ ↓ ↓
B) ↓ ↑ ↔ ↔
C) ↑ ↓ ↑ ↑
D) ↑ ↓ ↑ ↔
E) ↑ ↑ ↑ ↑
68. A 48-year-old woman reports severe polyuria (producing about 0.5 l
of urine each hour) and polydipsia (drinking two to three glasses of
water every hour). Her urine contains no glucose, and she is placed
on overnight water restriction for further evaluation. The next
morning, she is weak and confused, her sodium concentration is 160
mEq/l, and her urine osmolarity is 80 mOsm/l. Which of the
following is the most likely diagnosis?
A) Diabetes mellitus
B) Diabetes insipidus
C) Primary aldosteronism
D) Renin-secreting tumor
E) Syndrome of inappropriate ADH
69. Which substance is filtered most readily by the glomerular
capillaries?
A) Albumin in plasma
B) Neutral dextran with a molecular weight of 25,000
C) Polycationic dextran with a molecular weight of 25,000
D) Polyanionic dextran with a molecular weight of 25,000
E) Red blood cells
70. A 22-year-old woman runs a 10-km race on a hot day and becomes
dehydrated. Assuming that her ADH levels are very high and that
her kidneys are functioning normally, in which part of the renal
tubule is the most water reabsorbed?
A) Proximal tubule
B) Loop of Henle
C) Distal tubule
D) Cortical collecting tubule
E) Medullary collecting duct
71. Furosemide (Lasix) is a diuretic that also produces natriuresis.
Which of the following is an undesirable side effect of furosemide
due to its site of action on the renal tubule?
A) Edema
B) Hyperkalemia
C) Hypercalcemia
D) Decreased ability to concentrate the urine
E) Heart failure
72. A female patient has unexplained hypernatremia (plasma Na+
= 167
mmol/l) and reports frequent urination and large urine volumes. A
urine specimen reveals that the Na+
concentration is 15 mmol/l (very
low) and the osmolarity is 155 mOsm/l (very low). Laboratory tests
reveal the following data: plasma renin activity = 3 ng angiotensin
I/ml/h (normal = 1.0), plasma ADH = 30 pg/ml (normal = 3 pg/ml),
and plasma aldosterone = 20 ng/dl (normal = 6 ng/dl). Which of the
following is the most likely reason for her hypernatremia?
A) Simple dehydration caused by decreased water intake
B) Nephrogenic diabetes insipidus
C) Central diabetes insipidus
D) Syndrome of inappropriate ADH
E) Primary aldosteronism
F) Renin-secreting tumor
73. Which change would you expect to find in a dehydrated person
deprived of water for 24 hours?
A) Decreased plasma renin activity
B) Decreased plasma antidiuretic hormone concentration
C) Increased plasma atrial natriuretic peptide concentration
D) Increased water permeability of the collecting duct
74. Juvenile (type 1) diabetes mellitus is often diagnosed because of
polyuria (high urine flow) and polydipsia (frequent drinking) that
occur because of which of the following?
A) Increased delivery of glucose to the collecting duct interferes
with the action of antidiuretic hormone
B) Increased glomerular filtration of glucose increases Na+
reabsorption via the sodium-glucose co-transporter
C) When the filtered load of glucose exceeds the renal threshold, a
rising glucose concentration in the proximal tubule decreases the
osmotic driving force for water reabsorption
D) High plasma glucose concentration decreases thirst
E) High plasma glucose concentration stimulates ADH release
from the posterior pituitary
75. Which of the following would cause the most serious hypokalemia?
A) A decrease in potassium intake from 150 mEq/day to 60
mEq/day
D) An increase in sodium intake from 100 to 200 mEq/day
C) Excessive aldosterone secretion plus high sodium intake
D) Excessive aldosterone secretion plus low sodium intake
E) A patient with Addison’s disease
F) Treatment with a beta-adrenergic blocker
G) Treatment with spironolactone
76. A 26-year-old woman reports that she has had a severe migraine and
has taken six times more than the recommended dose of aspirin for
the past 3 days to relieve her headaches. Her plasma pH is 7.24.
Which of the following would you expect to find (compared with
normal)?
Plasma concentration
Plasma Urine
Excretion
Urine NH4
+
Excretion
Plasma Anion
Gap
A) ↑ ↓ ↑ ↑ ↑
B) ↑ ↑ ↑ ↓ ↑
C) ↓ ↓ ↓ ↓ ↓
D) ↓ ↓ ↓ ↑ ↑
E) ↓ ↓ ↓ ↑ ↓
F) ↓ ↔ ↓ ↓ ↔
77. Under conditions of normal renal function, which of the following
statements is true of the concentration of urea in tubular fluid at the
end of the proximal tubule?
A) It is higher than the concentration of urea in tubular fluid at the
tip of the loop of Henle
B) It is higher than the concentration of urea in the plasma
C) It is higher than the concentration of urea in the final urine in
antidiuresis
D) It is lower than plasma urea concentration because of active
urea reabsorption along the proximal tubule
78. You begin treating a hypertensive patient with a powerful loop
diuretic (e.g., furosemide). Which changes would you expect to find,
compared with pretreatment values, when he returns for a follow-up
examination 2 weeks later?
Urine Sodium Excretion Extracellular Fluid Volume Blood Pressure Plasma Potassium Concentration
A) ↑ ↓ ↓ ↓
B) ↑ ↓ ↔ ↔
C) ↔ ↓ ↓ ↓
D) ↔ ↓ ↔ ↔
E) ↑ ↔ ↓ ↑
79. Which change, compared with normal, would be expected to occur,
under steady-state conditions, in a patient whose severe renal disease
has reduced the number of functional nephrons to 25% of normal?
A) Increased GFR of the surviving nephrons
B) Decreased urinary creatinine excretion rate
C) Decreased urine flow rate in the surviving nephrons
D) Decreased urinary excretion of sodium
E) Increased urine-concentrating ability
80. Which statement is correct?
A) Urea reabsorption in the medullary collecting tubule is less than
in the distal convoluted tubule during antidiuresis
B) Urea concentration in the interstitial fluid of the renal cortex is
greater than in the interstitial fluid of the renal medulla during
antidiuresis
C) The thick ascending limb of the loop of Henle reabsorbs more
urea than the inner medullary collecting tubule during
antidiuresis
D) Urea reabsorption in the proximal tubule is greater than in the
cortical collecting tubule
81. A patient’s urine is collected for 2 hours, and the total volume is 600
ml during this time. Her urine osmolarity is 150 mOsm/l, and her
plasma osmolarity is 300 mOsm/l. What is her “free water clearance”?
A) +5.0 ml/min
B) +2.5 ml/min
C) 0.0 ml/min
D) −2.5 ml/min
E) −5.0 ml/min
Questions 82–85
For Questions 82–85, choose the appropriate nephron site in the above
figure.
82. In a patient with severe central diabetes insipidus caused by a lack of
ADH secretion, which part of the tubule would have the lowest
tubular fluid osmolarity?
A) A
B) B
C) C
D) D
E) E
83. In a person on a very low potassium diet, which part of the nephron
would be expected to reabsorb the most potassium?
A) A
B) B
C) C
D) D
E) E
84. Which part of the nephron normally reabsorbs the most water?
A) A
B) B
C) C
D) D
E) E
85. In a normally functioning kidney, which part of the tubule has the
lowest permeability to water during antidiuresis?
A) A
B) B
C) C
D) D
E) E
86. Which substances are best suited to measure interstitial fluid
volume?
A) Inulin and heavy water
B) Inulin and 22Na
C) Heavy water and 125I-albumin
D) Inulin and 125I-albumin
E) 51Cr red blood cells and 125I-albumin
87. Long-term administration of furosemide (Lasix) would do what?
A) Inhibit the Na+-Cl − co-transporter in the renal distal tubules
B) Inhibit the Na+-Cl − -K+
co-transporter in the renal tubules
C) Tend to reduce renal concentrating ability
D) Tend to cause hyperkalemia
E) A and C
F) B and C
G) B, C, and D
88. A patient with normal lungs who has uncontrolled type 1 diabetes
and a plasma glucose concentration of 400 mg/100 ml (normal ∼100
mg/100 ml) would be expected to have which set of blood values?
pH Na
+
(mmol/l) Cl
−
(mmol/l)
A) 7.66 22 20 143 111
B) 7.52 38 48 146 100
C) 7.29 14 30 143 117
D) 7.25 12 28 142 102
E) 7.07 14 50 144 102
89. Which of the following would be expected to cause a decrease in
extracellular fluid potassium concentration (hypokalemia) at least in
part by stimulating potassium uptake into the cells?
A) α-Adrenergic blockade
B) Insulin deficiency
C) Strenuous exercise
D) Aldosterone deficiency (Addison disease)
E) Metabolic alkalosis
90. If a person has a kidney transport maximum for glucose of 350
mg/min, a GFR of 100 ml/min, a plasma glucose level of 150 mg/dl, a
urine flow rate of 2 ml/min, and no detectable glucose in the urine,
what would be the approximate rate of glucose reabsorption,
assuming normal kidneys?
A) Glucose reabsorption cannot be estimated from these data
B) 0 mg/min
C) 50 mg/min
D) 150 mg/min
E) 350 mg/min
91. Which diuretic inhibits Na+-2Cl − -K+
co-transport in the loop of Henle
as its primary action?
A) Thiazide diuretic
B) Furosemide
C) Carbonic anhydrase inhibitor
D) Osmotic diuretic
E) Amiloride
F) Spironolactone
92. A selective decrease in efferent arteriolar resistance would __________
glomerular hydrostatic pressure, __________ GFR, and __________
renal blood flow.
A) increase, increase, increase
B) increase, decrease, increase
C) increase, decrease, decrease
D) decrease, increase, decrease
E) decrease, decrease, increase
F) decrease, increase, increase
93. A patient with renal tubular acidosis would be expected to have
which set of blood values?
pH Na+ (mmol/l) Cl
−
(mmol/l)
A) 7.66 22 20 143 111
B) 7.52 38 48 146 100
C) 7.07 14 50 144 102
D) 7.25 12 28 142 102
E) 7.29 14 30 143 117
94. A patient reports that he is always thirsty, and his breath has an
acetone smell. You suspect that he has diabetes mellitus, and the
diagnosis is confirmed by a urine sample that tests positive for
glucose and a blood sample that shows a fasting blood glucose
concentration of 400 mg/dl. Compared with normal, you would
expect to find which changes in his urine?
Urine pH NH4
+ Excretion Urine volume (ml/24 h) Renal Production
A) ↓ ↓ ↓ ↓
B) ↓ ↑ ↓ ↓
C) ↑ ↓ ↓ ↓
D) ↓ ↑ ↑ ↑
E) ↑ ↑ ↑ ↑
Questions 95–97
A person with normal body fluid volumes weighs 60 kg and has an
extracellular fluid volume of approximately 12.8 l, a blood volume of 4.3 l,
and a hematocrit of 0.4; 57% of his body weight is water. Use this
information to answer Questions 95–71.
95. What is the approximate intracellular fluid volume?
A) 17.1 l
B) 19.6 l
C) 21.4 l
D) 23.5 l
E) 25.6 l
96. What is the approximate plasma volume?
A) 2.0 l
B) 2.3 l
C) 2.6 l
D) 3.0 l
E) 3.3 l
97. What is the approximate interstitial fluid volume?
A) 6.4 l
B) 8.4 l
C) 10.2 l
D) 11.3 l
E) 12.0 l
98. Which nephron segment is the primary site of magnesium
reabsorption under normal conditions?
A) Proximal tubule
B) Descending limb of the loop of Henle
C) Ascending limb of the loop of Henle
D) Distal convoluted tubule
E) Collecting ducts
99. Which changes would you expect to find in a newly diagnosed 10-
year-old patient with type 1 diabetes and uncontrolled
hyperglycemia (plasma glucose = 300 mg/dl)?
Thirst (Water Intake) Urine Volume Glomerular Filtration Rate Afferent Arteriolar Resistance
A) ↑ ↓ ↑ ↓
B) ↑ ↑ ↓ ↑
C) ↑ ↑ ↑ ↓
D) ↓ ↑ ↑ ↑
E) ↓ ↓ ↓ ↓
Questions 100 and 101
To evaluate kidney function in a 45-year-old woman with type 2 diabetes,
you ask her to collect her urine for a 24-hour period. She collects 3600 ml of
urine in that period. The clinical laboratory returns the following results after
analyzing the patient’s urine and plasma samples: plasma creatinine = 4
mg/dl, urine creatinine = 32 mg/dl, plasma potassium = 5 mmol/l, and urine
potassium = 10 mmol/l.
100. What is this patient’s approximate GFR, assuming that she collected
all her urine in the 24-hour period?
A) 10 ml/min
B) 20 ml/min
C) 30 ml/min
D) 40 ml/min
E) 80 ml/min
101. What is the net renal tubular reabsorption rate of potassium in this
patient?
A) 1.050 mmol/min
B) 0.100 mmol/min
C) 0.037 mmol/min
D) 0.075 mmol/min
E) Potassium is not reabsorbed in this example
Questions 102–106
Match each of the patients described in Questions 102–106 with the correct
set of blood values in the following table (the same values may be used for
more than one patient).
pH PCO 2
(mm Hg) Na
+
(mEq/l) Cl
−
(mEq/l)
A) 7.66 22 20 143 111
B) 7.28 30 65 142 102
C) 7.24 12 29 144 102
D) 7.29 14 30 143 117
E) 7.52 38 48 146 100
F) 7.07 14 50 144 102
102. A patient with severe diarrhea
103. A patient with primary aldosteronism
104. A patient with proximal renal tubular acidosis
105. A patient with diabetic ketoacidosis and emphysema
106. A patient treated chronically with a carbonic anhydrase inhibitor
107. Which change would you expect to find in a patient who developed
acute renal failure after ingesting poisonous mushrooms that caused
renal tubular necrosis?
A) Increased plasma bicarbonate concentration
B) Metabolic acidosis
C) Decreased plasma potassium concentration
D) Decreased blood urea nitrogen concentration
E) Decreased hydrostatic pressure in Bowman’s capsule
108. Which of the following has similar values for both intracellular and
interstitial body fluids?
A) Potassium ion concentration
B) Colloid osmotic pressure
C) Sodium ion concentration
D) Chloride ion concentration
E) Total osmolarity
109. Which of the following is true of the tubular fluid that passes
through the lumen of the early distal tubule in the region of the
macula densa?
A) It is usually isotonic
B) It is usually hypotonic
C) It is usually hypertonic
D) It is hypertonic in antidiuresis
E) It is hypertonic when the filtration rate of its own nephron
decreases to 50% below normal
110. In a person with normal kidneys and normal lungs who has chronic
metabolic acidosis, you would expect to find all of the following,
compared with normal, EXCEPT:
A) Increased renal excretion of NH4Cl
B) Decreased urine pH
B) Decreased urine excretion
D) Increased plasma concentration
E) Decreased plasma
111. Assume that you have a patient who needs fluid therapy and you
decide to administer by intravenous infusion 2.0 l of 0.45% NaCl
solution (molecular weight NaCl = 58.5). After osmotic equilibrium,
which changes would you expect, compared with before infusion of
the NaCl?
INTRA-CELLULAR
VOLUME
INTRA-CELLULAR
OSMOLARITY
ExTRA-CELLULAR
VOLUME
ExTRA-CELLULAR
OSMOLARITY
A) ↑ ↑ ↑ ↑
B) ↑ ↓ ↑ ↓
C) ↔ ↑ ↑ ↑
D) ↓ ↑ ↑ ↑
E) ↓ ↓ ↓ ↓
112. If the renal clearance of substance X is 300 ml/min and the
glomerular filtration rate is 100 ml/min, it is most likely that
substance X is
A) Filtered freely but not secreted or reabsorbed
B) Bound to plasma proteins
C) Secreted
D) Reabsorbed
E) Bound to tubular proteins
F) Clearance of a substance cannot be greater than the GFR
113. Which change tends to increase urinary calcium (Ca2+) excretion?
A) Extracellular fluid volume expansion
B) Increased plasma parathyroid hormone concentration
C) Decreased blood pressure
D) Increased plasma phosphate concentration
E) Metabolic alkalosis
114. Which change would you expect to find in a patient consuming a
high-sodium diet (200 mEq/day) compared with the same patient on
a normal-sodium diet (100 mEq/day), assuming steady-state
conditions?
A) Increased plasma aldosterone concentration
B) Increased urinary potassium excretion
C) Decreased plasma renin activity
D) Decreased plasma atrial natriuretic peptide
E) An increase in plasma sodium concentration of at least 5 mmol/l
115. What would tend to decrease GFR by more than 20% in a normal
kidney?
A) Decrease in renal arterial pressure from 100 to 85 mm Hg
B) 50% decrease in afferent arteriolar resistance
C) 50% decrease in efferent arteriolar resistance
D) 50% increase in the glomerular capillary filtration coefficient
E) Decrease in plasma colloid osmotic pressure from 28 to 20 mm
Hg
116. Acute metabolic acidosis tends to _____ intracellular K+
concentration and _____ K+
secretion by the cortical collecting
tubules.
A) Increase, increase
B) Increase, decrease
C) Decrease, increase
D) Decrease, decrease
E) Cause no change in, increase
F) Cause no change in, cause no change in
117. Which statement is true?
A) ADH increases water reabsorption from the ascending loop of
Henle
B) Water reabsorption from the descending loop of Henle is
normally less than that from the ascending loop of Henle
C) Sodium reabsorption from the ascending loop of Henle is
normally less than that from the descending loop of Henle
D) Osmolarity of fluid in the early distal tubule would be less than
300 mOsm/l in a dehydrated person with normal kidneys and
increased ADH levels
E) ADH decreases the urea permeability in the medullary
collecting tubules
118. In a person on a high-potassium (200 mmol/day) diet, which part of
the nephron would be expected to secrete the most potassium?
A) Proximal tubule
B) Descending loop of Henle
C) Ascending loop of Henle
D) Early distal tubule
E) Collecting tubules
119. Which of the following would you expect to find in a patient who
has chronic diabetic ketoacidosis?
A) Decreased renal excretion, increased NH4
+
excretion,
increased plasma anion gap
B) Increased respiration rate, decreased arterial , decreased
plasma anion gap
C) Increased NH4
+
excretion, increased plasma anion gap,
increased urine pH
D) Increased renal production, increased NH4
+
excretion,
decreased plasma anion gap
E) Decreased urine pH, decreased renal excretion, increased
arterial
120. Using the indicator dilution method to assess body fluid volumes in
a 40-year-old man weighing 70 kg, the inulin space is calculated to be
16 l, and 125I-albumin space is 4 l. If 60% of his total body weight is
water, what is his approximate interstitial fluid volume?
A) 4 l
B) 12 l
C) 16 l
D) 26 l
E) 38 l
F) 42 l
121. What would tend to decrease plasma potassium concentration by
causing a shift of potassium from the extracellular fluid into the cells?
A) Strenuous exercise
B) Aldosterone deficiency
C) Acidosis
D) β-adrenergic blockade
E) Insulin excess
122. A 26-year-old construction worker is brought to the emergency
department with a change in mental status after working a 10-hour
shift on a hot summer day (average outside temperature was 97°F
[36°C]). The man had been sweating profusely during the day but did
not drink fluids. He has a fever of 102°F [39°C], a heart rate of 140
beats/min, and a blood pressure of 100/55 mm Hg in the supine
position. Upon examination, he has no perspiration, appears to have
dry mucous membranes, and is poorly oriented to person, place, and
time. Assuming that his kidneys were normal yesterday, which set of
hormone levels describes his condition, compared with normal?
A) High ADH, high renin, low angiotensin II, low aldosterone
B) Low ADH, low renin, low angiotensin II, low aldosterone
C) High ADH, low renin, high angiotensin II, low aldosterone
D) High ADH, high renin, high angiotensin II, high aldosterone
E) Low ADH, high renin, low angiotensin II, high aldosterone
123. A 23-year-old man runs a 10-km race in July and loses 2 l of fluid by
sweating. He also drinks 2 l of water during the race. Which changes
would you expect, compared with normal, after he absorbs the water
and assuming osmotic equilibrium and no excretion of water or
electrolytes?
INTRA-CELLULAR
VOLUME
INTRA-CELLULAR
OSMOLARITY
ExTRA-CELLULAR
VOLUME
ExTRA-CELLULAR
OSMO-LARITY
A) ↓ ↑ ↓ ↑
B) ↓ ↓ ↓ ↓
C) ↔ ↓ ↔ ↓
D) ↔ ↑ ↓ ↑
E) ↑ ↓ ↓ ↓
F) ↑ ↓ ↑ ↓
124. Which change would tend to increase Ca2+
reabsorption in the renal
tubule?
A) Extracellular fluid volume expansion
B) Increased plasma parathyroid hormone concentration
C) Increased blood pressure
D) Decreased plasma phosphate concentration
E) Metabolic acidosis
125. A young man is found comatose, having taken an unknown
number of sleeping pills an unknown time before. An arterial blood
sample yields the following values: pH = 7.02, = 14 mEq/l, and
= 68 mm Hg. Which of the following describes this patient’s acidbase status most accurately?
A) Uncompensated metabolic acidosis
B) Uncompensated respiratory acidosis
C) Simultaneous respiratory and metabolic acidosis
D) Respiratory acidosis with partial renal compensation
E) Respiratory acidosis with complete renal compensation
126. In a person with chronic respiratory acidosis who has partial renal
compensation, you would expect to find which changes, compared
with normal? ______ urinary excretion of NH4
+
; ______ plasma
concentration; and _____ urine pH.
A) Increased, increased, decreased
B) Increased, decreased, decreased
C) No change in, increased, decreased
D) No change in, no change in, decreased
E) Increased, no change in, increased
127. Increases in both renal blood flow and GFR are caused by which
mechanism?
A) Dilation of the afferent arterioles
B) Increased glomerular capillary filtration coefficient
C) Increased plasma colloid osmotic pressure
D) Dilation of the efferent arterioles
128. A 55-year-old male patient with hypertension has had his blood
pressure reasonably well controlled by administration of a thiazide
diuretic. At his last visit (6 months ago), his blood pressure was
130/75 mm Hg, and his serum creatinine was 1 mg/100 ml. He has
been exercising regularly for the past 2 years but recently has
reported knee pain and began taking large amounts of a nonsteroidal
antiinflammatory drug. When he arrives at your office, his blood
pressure is 155/85 mm Hg, and his serum creatinine is 2.5 mg/100 ml.
What best explains his increased serum creatinine level?
A) Increased efferent arteriolar resistance that reduced GFR
B) Increased afferent arteriolar resistance that reduced GFR
C) Increased glomerular capillary filtration coefficient that reduced
GFR
D) Increased angiotensin II formation that decreased GFR
E) Increased muscle mass due to the exercise
129. An older adult patient reports muscle weakness and lethargy. A
urine specimen reveals a Na+
concentration of 600 mmol/l and an
osmolarity of 1200 mOsm/l. Additional laboratory tests provide the
following information: plasma Na+
concentration = 167 mmol/l,
plasma renin activity = 4 ng angiotensin I/ml/h (normal = 1 ml/h),
plasma ADH = 60 pg/ml (normal = 3 pg/ml), and plasma aldosterone
= 15 ng/dl (normal = 6 ng/dl). What is the most likely reason for this
patient’s hypernatremia?
A) Dehydration caused by decreased fluid intake
B) Syndrome of inappropriate ADH
C) Nephrogenic diabetes insipidus
D) Primary aldosteronism
E) Renin-secreting tumor
Answers
1. E) Increased plasma protein concentration would reduce the net force
favoring capillary filtration and would oppose edema formation. All of the
other changes would increase capillary filtration rate and tend to cause
edema.
TMP14 pp. 316–318.
2. E) The total body water would initially be 60% of body weight or ∼30 l.
The initial total mOsm in the body fluids would be 30 l × 360 mOsm/l, or
10,800 mOsm. After adding the glucose solution, total body water would
be 30 l + 2 l or 32 l. If we assume that all of the glucose is metabolized, the
final mOsm would also be 10,800. Therefore, the extracellular and
intracellular osmolarity would be 10,800 mOsm/32 l, or approximately 338
mOsm/l.
TMP14 pp. 312–314
3. B) The initial total body water would be 60% of body weight or ∼30 L. The
initial total mOsm in the body fluids would be 30 l × 360 mOsm/l, or 10,800
mOsm. The initial extracellular fluid volume would be 20% of body
weight, or ∼10 l. The initial total mOsm in the extracellular fluid would be
360 mOsm/l × 10 l, or 3600 mOsm. After adding the glucose solution, total
body water would be 30 l + 2 l or 32 l. If we assume that all of the glucose is
metabolized, the final mOsm would also be 10,800. Therefore, the
extracellular and intracellular osmolarity would be 10,800 mOsm/32 l, or
approximately 338 mOsm/l. The extracellular fluid would continue to have
a total of 3600 mOsm. Therefore, the final extracellular fluid volume would
be 3600 mOsm/338 mOsm per l, or ∼10.7 l.
TMP14 pp. 312–314
4. C) A doubling of serum creatinine implies a reduction in glomerular
filtration rate (GFR). Non-steroidal anti-inflammatory drugs (NSAIDS)
inhibit prostaglandin synthesis, which would tend increase afferent
arteriolar and reduce GFR.
TMP14 pp. 337–342, 362.
5. C) Inhibition of sodium-glucose co-transporter 2 (SGLT2) would reduce
glucose and sodium chloride reabsorption in the proximal tubules, causing
increased sodium chloride delivery to the macula densa, which would, in
turn, cause a feedback-mediated vasoconstriction of afferent arterioles and
reductions in glomerular filtration rate and renal blood flow.
TMP14 pp. 339–340, 345–346
6. D) Filtration fraction (FF) = glomerular filtration rate (GFR)/renal plasma
flow. GFR = Kf x (PG - ΠC – PB
) where Kf is the glomerular capillary
filtration coefficient, PG is glomerular hydrostatic pressure, ΠC is
glomerular capillary colloid osmotic pressure, and PB
is Bowman’s space
hydrostatic pressure. Therefore, GFR= 10 × (60 − 20 − 10) = 100 ml/min.
Since renal plasma flow is 600 ml/min, FF= 100 ml/min/600 ml/min = 0.167.
TMP14 pp. 333–336.
7. D) Bicarbonate is more avidly reabsorbed in the proximal tubules than
water, and therefore concentration decreases along the proximal tubules
and has a lower concentration in the urine than in the glomerular filtrate.
Approximately 85% of the filtered load of is normally reabsorbed in
the proximal tubules. All of the other statements are correct.
TMP14 pp. 354, 410–412.
8. B) With a glomerular filtration rate of 100 ml/min and a plasma glucose
concentration of 4 mg/ml, the filtered load of glucose would be 400
mg/min. Since the transport maximum for glucose in this case is 200
mg/min, the maximum rate of glucose reabsorption is 200 mg/min.
Glucose excreted rate is therefore the difference between the filtered load
of glucose (400 mg/min) and the glucose reabsorption rate (200 mg/min),
or 200 mg/min.
TMP14 pp. 346–347
9. F) Fluid in the ascending loop of Henle becomes dilute as electrolytes are
reabsorbed and water remains in the tubule. When antidiuretic hormone
(ADH) levels are very low, as occurs in central diabetes insipidus, fluid in
the distal and collecting tubules, and the collecting ducts is further diluted
by the reabsorption of sodium chloride and the failure to reabsorb water.
This lead to a very dilute urine (see figure below).
TMP14 p. 366
10. B) Potassium excretion in this case is equal to urine concentration of K+
(80 mmol/l) multiplied by the urine flow rate (2.0 ml/min, or 0.002 l/min)
which is 0.16 mmol/min.
TMP14 pp. 363–364
11. E) A 50% reduction in glomerular filtration rate (GFR) would initially
cause a reduction in creatinine excretion rate. However, within a few days,
the filtered load and excretion of creatinine would return to normal as
serum creatinine concentration increased to approximately twice the
normal level under steady-state conditions (see figure in next column).
Creatinine clearance is approximately equal to GFR and would also be
reduced by approximately 50%.
TMP14 pp. 361–362
12. A) Urine excretion rate is equal to glomerular filtration rate (GFR) minus
tubular reabsorption rate. In this example, the final is 40 ml/min, and the
tubular reabsorption rate is 40 ml/min. Therefore, the urine excretion rate
is zero.
TMP14 p. 343
13. D) Total renal plasma flow (RPF) is equal to the clearance of
paraaminohippuric acid (PAH) divided by the renal PAH extraction ratio
(EPAH).
Clearance of PAH (CPAH) = (UPAH × V) / APAH = (200 µg/ml × 2 ml/min)/1.0
µg/ml = 400 ml/min (EPAH) = (APAH – VPAH)/APAH = (1.0 µg/ml − 0.2
µg/ml)/1.0 µg/ml = 0.8
RPF = 400 ml/min / 0.8 = 500 ml/min
Where UPAH is urine PAH concentration, APAH is arterial PAH concentration,
VPAH is renal venous PAH concentration, and V is urine flow rate.
TMP14 pp. 362–363
14. A) Type A intercalated cells of the collecting tubules secrete H+
by a
hydrogen-ATPase transporter and by a hydrogen-potassium-ATPase
transporter. They also reabsorb and K+
(see figure on next page).
TMP14 pp. 352–354
15. E) Increased extracellular fluid osmolarity would tend to cause
hyperkalemia, rather than hypokalemia due to cell dehydration, which raise
intracellular potassium concentration and promotes potassium diffusion
into the extracellular fluid. All of the other statements are correct.
TMP14 pp. 383–384
16. C) The cortical collecting tubule is relatively impermeable to urea, and
very lile reabsorption occurs in this tubular segment, even in the presence
of ADH. All of the other statements are correct.
TMP14 pp. 370–371
17. C) The thick ascending loop of Henle is relatively impermeable to water
even in the presence of high levels of ADH. The other tubular segments
reabsorb significant amounts of water.
TMP14 pp. 368–369, 372–373
18. D) Acute metabolic alkalosis tends to shift K+
from the extracellular fluid
into the cells, including the renal tubular cells, contributing to increased K+
secretion and decreased plasma K+
concentration (hypokalemia).
TMP14 pp. 384, 389
19. D) Aldosterone antagonists such as spironolactone tend to cause
hyperkalemia rather than hypokalemia by shifting potassium from the
intracellular to the extracellular fluid and by inhibiting potassium
secretion in the principal cells of collecting tubules. All of the other
statements are correct.
TMP14 pp. 384, 387
20. C) Under steady-state conditions and no changes in protein or electrolyte
intake, excretion rate of sodium would not change and would be equal to
sodium intake. Creatinine excretion rate would also be unchanged in the
steady-state due to increased plasma creatinine concentration, which
would return the filtered load of creatinine to normal despite a 50%
reduction in glomerular filtration rate (creatinine clearance). Uncontrolled
diabetes mellitus can also result in metabolic acidosis, which would reduce
plasma bicarbonate concentration and stimulate a compensatory increased
in renal ammonium production and increased ammonium excretion rate.
TMP14 pp. 420, 428–429.
21. D) The patient has a low pH (7.18) and a low plasma ,indicating
metabolic acidosis. The plasma anion gap is
This is far above normal (8-16 mEq/l), indicating unmeasured anions and
excess nonvolatile acids. Therefore, the most likely explanation for the
patient’s metabolic acidosis is methanol poisoning. Patients with
emphysema would have respiratory acidosis. Patients with diarrhea or
renal tubular acidosis would have metabolic acidosis due to bicarbonate
loss and normal anion gap with hyperchloremia. Ingestion of excess
sodium bicarbonate would cause metabolic alkalosis.
TMP14 pp. 418–420
22. E) Atrial natriuretic peptide decreases renal sodium reabsorption through
direct effects on the renal tubules as well as indirectly by inhibiting renin
secretion. Both of these effects contribute to increased renal sodium
excretion.
TMP14 p. 360
23. A) Urine excretion rate of Na+
is equal to urine flow rate (1.0 ml/min or
0.001 l/min) multiplied by urine Na+
concentration (80 mmol/l), or 0.08
mmol/min.
TMP14 pp. 361–363
24. A) Approximately 65% of the filtered load of potassium is reabsorbed in
the proximal tubule. Variations in renal excretion of potassium during
change in potassium intake are achieved mainly by changes in potassium
secretion in collecting tubules. With high potassium intake, the proximal
tubule still reabsorbs a high fraction of the filtered load of potassium.
TMP14 pp. 384–386
25. A) Metabolic alkalosis shifts potassium from the extracellular fluid into
the cells and contributes to hypokalemia. Insulin deficiency, aldosterone
deficiency, beta-adrenergic blockade, and increased extracellular fluid
osmolarity all cause a shift of potassium from the cells to the extracellular
fluid.
TMP14 pp. 383–384
26. E) Theoretically, if all of the plasma flowing through the kidneys was
cleared of a substance, the clearance rate would be equal to the total renal
plasma flow. In this example, renal plasma flow is equal to 800 ml/min.
TMP14 pp. 362–363
27. C) A 3% NaCl solution is hypertonic, and when infused intravenously, it
would increase extracellular fluid volume and osmolarity, thereby causing
water to flow out of the cell. This action would decrease intracellular fluid
volume and further increase extracellular fluid volume. The 0.9% NaCl
solution and 5% dextrose solution are isotonic and therefore would not
reduce intracellular fluid volume. Pure water and the 0.45% NaCl solution
are hypotonic, and when infused, they would increase both intracellular
and extracellular fluid volumes.
TMP14 pp. 311–314
28. B) Partial obstruction of a major vein draining a tissue would increase
capillary hydrostatic pressure in the tissue, which, in turn, would raise
capillary fluid filtration and cause increases interstitial fluid volume,
interstitial fluid hydrostatic pressure, and lymph flow. The increased
lymph flow would “wash out” proteins from the interstitial fluid,
decreasing interstitial fluid protein concentration.
TMP14 pp. 316–317
29. C) The hypernatremia (plasma Na+
= 165 mmol/l) associated with a low
blood pressure (88/44 mm Hg) suggests dehydration. The frequent
urination and low urine specific gravity (1.003, which implies a urine
osmolarity of about 100-120 mOsm/l) despite hypernatremia and
dehydration suggests diabetes insipidus due to either insufficient secretion
of ADH (central diabetes insipidus) or failure of the kidneys to respond to
ADH (nephrogenic diabetes insipidus).
TMP14 pp. 315–316, 374, 432
30. D) A severe renal artery stenosis that reduces GFR to 25% of normal
would also decrease renal blood flow but would cause only a transient
decrease in urinary creatinine excretion. The transient decrease in
creatinine excretion would increase serum creatinine (to about four times
normal), which would restore the filtered creatinine load to normal and
therefore return urinary creatinine excretion to normal levels under
steady-state conditions. Urinary sodium secretion would also decrease
transiently but would be restored to normal so that intake and excretion of
sodium are balanced. Plasma sodium concentration would not change
significantly because it is carefully regulated by the ADH–thirst
mechanism.
TMP14 pp. 362, 428–429
31. B) A 1% solution of dextrose is hypotonic, and when infused, it would
increase both intracellular and extracellular fluid volumes while
decreasing the osmolarity of these compartments.
TMP14 pp. 311–313
32. B) Excessive secretion of ADH would increase renal tubular reabsorption
of water, thereby increasing extracellular fluid volume and reducing
extracellular fluid osmolarity. The reduced osmolarity, in turn, would
cause water to flow into the cells and raise intracellular fluid volume. In
the steady state, both extracellular and intracellular fluid volumes would
increase, and osmolarity of both compartments would decrease.
TMP14 pp. 314, 375–376
33. C) A 3% solution of NaCl is hypertonic, and when infused into the
extracellular fluid, it would raise osmolarity, thereby causing water to flow
out of the cells into the extracellular fluid until osmotic equilibrium is
achieved. In the steady state, extracellular fluid volume would increase,
intracellular fluid volume would decrease, and osmolarity of both
compartments would increase.
TMP14 pp. 311–313
34. C) Aldosterone stimulates potassium secretion by the principal cells of
the collecting tubules. Therefore, blockade of the action of aldosterone
with spironolactone would inhibit potassium secretion. Other factors that
stimulate potassium secretion by the cortical collecting tubule include
increased potassium concentration, increased cortical collecting tubule
flow rate (as would occur with high sodium intake or a diuretic that
reduces proximal tubular sodium reabsorption), and acute alkalosis.
TMP14 pp. 386–389
35. C) Phosphate excretion by the kidneys is controlled by an overflow
mechanism. When the transport maximum for reabsorbing phosphate is
exceeded, the remaining phosphate in the renal tubules is excreted in the
urine and can be used to buffer hydrogen ions and form titratable acid.
Phosphate normally begins to spill into the urine when the concentration
of extracellular fluid rises above a threshold of 0.8 mmol/l, which is usually
exceeded.
TMP14 pp. 391–392
36. B) GFR is equal to inulin clearance, which is calculated as the urine inulin
concentration (100 mg/ml) × urine flow rate (1 ml/min)/plasma inulin
concentration (2 mg/ml), which is equal to 50 ml/min.
TMP14 pp. 360–361
37. D) The net urea reabsorption rate is equal to the filtered load of urea (GFR
[50 ml/min] × plasma urea concentration [2.5 mg/ml]) − urinary excretion
rate of urea (urine urea concentration [50 mg/ml] × urine flow rate [1
ml/min]). Therefore, net urea reabsorption = (50 ml/min × 2.5 mg/ml) − (50
mg/ml × 1 ml/min) = 75 mg/min.
TMP14 pp. 360–363
38. B) As water flows up the ascending limb of the loop of Henle, solutes are
reabsorbed, but this segment is relatively impermeable to water;
progressive dilution of the tubular fluid occurs so that the osmolarity
decreases to approximately 100 mOsm/l by the time the fluid reaches the
early distal tubule. Even during maximal antidiuresis, this portion of the
renal tubule is relatively impermeable to water and is therefore called the
diluting segment of the renal tubule.
TMP14 pp. 372–373
39. C) In the absence of ADH secretion, a marked increase in urine volume
occurs because the late distal and collecting tubules are relatively
impermeable to water. As a result of increased urine volume, there are
dehydration, increased plasma osmolarity and high plasma sodium
concentration, and increased thirst which leads to increased water intake.
The dehydration reduces extracellular fluid volume increases renin
secretion and plasma renin concentration.
TMP14 pp. 375–376, 378–379
40. C) When potassium intake is doubled (from 80 to 160 mmol/day),
potassium excretion also approximately doubles within a few days, and
the plasma potassium concentration increases only slightly. Increased
potassium excretion is achieved largely by increased secretion of
potassium in the cortical collecting tubule. Increased aldosterone
concentration plays a significant role in increasing potassium secretion and
in maintaining a relatively constant plasma potassium concentration
during increases in potassium intake. Sodium excretion does not change
markedly during chronic increases in potassium intake.
TMP14 pp. 386–389
41. D) Most of the daily variation in potassium excretion is caused by
changes in potassium secretion in the late distal tubules and collecting
tubules. Therefore, when the dietary intake of potassium increases, the
total body balance of potassium is maintained primarily by an increase in
potassium secretion in these tubular segments. Increased potassium intake
has lile effect on GFR or on reabsorption of potassium in the proximal
tubule and loop of Henle. Although high potassium intake may cause a
slight shift of potassium into the intracellular compartment, a balance
between intake and output must be achieved by increasing the excretion of
potassium during high potassium intake.
TMP14 pp. 384–386
42. E) A 50% decrease in efferent arteriolar resistance would cause a
substantial decrease in GFR. A decrease in renal arterial pressure from 100
to 80 mm Hg in a normal kidney would cause only a slight reduction in
GFR in a normal kidney because of autoregulation. All of the other
changes would tend to increase GFR.
TMP14 pp. 333–336
43. A) The patient described has protein in the urine (proteinuria) and
reduced plasma protein concentration as a result of glomerulonephritis
caused by an untreated streptococcal infection (“strep throat”). The
reduced plasma protein concentration, in turn, decreased the plasma
colloid osmotic pressure and resulted in leakage from the plasma to the
interstitium. The extracellular fluid edema raised interstitial fluid pressure
and interstitial fluid volume, causing increased lymph flow and decreased
interstitial fluid protein concentration. Increasing lymph flow causes a
“washout” of the interstitial fluid protein as a safety factor against edema.
The decreased blood volume would tend to lower blood pressure and
stimulate the secretion of renin by the kidneys, raising the plasma renin
concentration.
TMP14 pp. 316–319
44. C) In a patient with a very high rate of renin secretion, there would also
be increased formation of angiotensin II, which in turn would stimulate
aldosterone secretion. The increased levels of angiotensin II and
aldosterone would cause a transient decrease in sodium excretion, which
would cause expansion of the extracellular fluid volume and increased
arterial pressure. The increased arterial pressure, as well as other
compensations, would return sodium excretion to normal so that intake
and output are balanced. Therefore, under steady-state conditions, sodium
excretion would be normal and equal to sodium intake. The increased
aldosterone concentration would cause hypokalemia (decreased plasma
potassium concentration), whereas the high level of angiotensin II would
cause renal vasoconstriction and decreased renal blood flow.
TMP14 pp. 358, 387
45. B) This patient has respiratory acidosis because the plasma pH is lower
than the normal level of 7.4, and the plasma is higher than the normal
level of 40 mm Hg. The elevation in plasma bicarbonate concentration
above normal (∼24 mEq/l) is due to partial renal compensation for the
respiratory acidosis. Therefore, this patient has respiratory acidosis with
partial renal compensation.
TMP14 pp. 415–416
46. D) GFR is approximately equal to creatinine clearance, which is
calculated as the urine creatinine concentration (50 mg/100 ml) × urine
flow rate (3 ml/min)/plasma creatinine concentration (3 mg/100 ml), which
is equal to 50 ml/min. Urine flow rate = 4320 ml/24 h = 4320 ml/1440 min =
3 ml/min.
TMP14 pp. 360–361
47. C) Because the patient has a low plasma pH (normal = 7.4), he has
acidosis. The fact that his plasma bicarbonate concentration is also low
(normal = 24 mEq/l) indicates that he has metabolic acidosis. However, he
also appears to have respiratory acidosis because his plasma is high
(normal = 40 mm Hg). The rise in is due to his impaired breathing as a
result of cardiopulmonary arrest. Therefore, the patient has a mixed
acidosis with combined metabolic and respiratory acidosis.
TMP14 pp. 415–416
48. D) An important compensation for respiratory acidosis is increased renal
production of NH4
+
and increased NH4
+
excretion. In acidosis, urinary
excretion of would be reduced, as would urine pH, and urinary
titratable acid would be slightly increased as a compensatory response to
the acidosis.
TMP14 pp. 412–414
49. B) Inhibition of aldosterone causes hyperkalemia by two mechanisms: (1)
shifting potassium out of the cells into the extracellular fluid and (2)
decreasing cortical collecting tubular secretion of potassium. Increasing
potassium intake from 60 to 180 mmol/day would cause only a very small
increase in plasma potassium concentration in a person with normal
kidneys and normal aldosterone feedback mechanisms. A reduction in
sodium intake also has very lile effect on plasma potassium
concentration. Chronic treatment with a diuretic that inhibits loop of
Henle Na+
-2Cl −
-K+
co-transport would tend to cause potassium loss in the
urine and hypokalemia. However, chronic treatment with a diuretic that
inhibits sodium reabsorption in the collecting ducts, such as amiloride,
would have lile effect on plasma potassium concentration.
TMP14 pp. 383–384, 386–389
50. D) Excessive activity of the amiloride-sensitive sodium channel in the
collecting tubules would cause a transient decrease in sodium excretion
and expansion of extracellular fluid volume, which in turn would increase
arterial pressure and decrease renin secretion, leading to decreased
aldosterone secretion. Under steady-state conditions, sodium excretion
would return to normal so that intake and renal excretion of sodium are
balanced. One of the mechanisms that re-establishes this balance between
intake and output of sodium is the rise in arterial pressure that induces a
“pressure natriuresis.”
TMP14 pp. 394, 433
51. A) Primary excessive secretion of aldosterone (Conn syndrome) would be
associated with marked hypokalemia and metabolic alkalosis (increased
plasma pH). Because aldosterone stimulates sodium reabsorption and
potassium secretion by the cortical collecting tubule, there could be a
transient decrease in sodium excretion and an increase in potassium
excretion, but under steady-state conditions, both urinary sodium and
potassium excretion would return to normal to match the intake of these
electrolytes. However, the sodium retention and the hypertension
associated with aldosterone excess would tend to reduce renin secretion.
TMP14 pp. 384, 387, 398, 417
52. C) The glomerular capillary filtration coefficient is the product of the
hydraulic conductivity and surface area of the glomerular capillaries.
Therefore, increasing the glomerular capillary filtration coefficient tends to
increase GFR. Increased afferent arteriolar resistance, decreased efferent
arteriolar resistance, increased Bowman’s capsule hydrostatic pressure,
and decreased glomerular hydrostatic pressure tend to decrease GFR.
TMP14 pp. 333–336
53. D) Impairment of proximal tubular NaCl reabsorption would increase
NaCl delivery to the macula densa, which in turn would cause a
tubuloglomerular feedback–mediated increase in afferent arteriolar
resistance. The increased afferent arteriolar resistance would decrease the
GFR. Initially there would be a transient increase in sodium excretion, but
after 3 weeks, steady-state conditions would be achieved. Sodium
excretion would equal sodium intake, and no significant change would
occur in urinary sodium excretion.
TMP14 pp. 339–340
54. C) The patient has a lower than normal pH and is therefore acidotic.
Because the plasma bicarbonate concentration is also lower than normal,
the patient has metabolic acidosis with respiratory compensation (i.e.,
is lower than normal). The plasma anion gap is in the
normal range, suggesting that the metabolic acidosis is not caused by
excess nonvolatile acids such as salicylic acid or ketoacids caused by
diabetes mellitus. Therefore, the most likely cause of the metabolic acidosis
is diarrhea, which would cause a loss of in the feces and would be
associated with a normal anion gap and a hyperchloremic (increased
chloride concentration) metabolic acidosis.
TMP14 pp. 415–416, 419–420
55. A) A 50% reduction of GFR would approximately double the plasma
creatinine concentration because creatinine is not reabsorbed or secreted,
and its excretion depends largely on glomerular filtration. Therefore, when
GFR decreases, the plasma concentration of creatinine increases until the
renal excretion of creatinine returns to normal. Plasma concentrations of
glucose, potassium, sodium, and hydrogen ions are closely regulated by
multiple mechanisms that keep them relatively constant even when GFR
falls to very low levels. Plasma phosphate concentration is also maintained
near normal until GFR falls to below 20% to 30% of normal.
TMP14 pp. 429–430
56. C) A 50% reduction in afferent arteriolar resistance with no change in
arterial pressure would increase renal blood flow and glomerular
hydrostatic pressure, thereby increasing GFR. At the same time, the
reduction in afferent arteriolar resistance would raise peritubular capillary
hydrostatic pressure.
TMP14 pp. 333–336, 356–357
57. A) The net filtration pressure at the glomerular capillaries is equal to the
sum of the forces favoring filtration (glomerular capillary hydrostatic
pressure) minus the forces that oppose filtration (hydrostatic pressure in
Bowman’s space and glomerular colloid osmotic pressure). Therefore, the
net pressure driving glomerular filtration is 50 − 12 − 30 = 8 mm Hg.
TMP14 p. 333
58. D) Uncontrolled diabetes mellitus results in increased blood acetoacetic
acid levels, which in turn cause metabolic acidosis and decreased plasma
and pH. The acidosis causes several compensatory responses,
including increased respiratory rate, which reduces plasma ; increased
renal NH+
production, which leads to increased NH+
excretion; and
increased phosphate buffering of hydrogen ions secreted by the renal
tubules, which increases titratable acid excretion.
TMP14 pp. 415–417
59. B) Infusion of a hypotonic solution of NaCl would initially increase
extracellular fluid volume and decrease extracellular fluid osmolarity. The
reduction in extracellular fluid osmolarity would cause osmotic flow of
fluid into the cells, thereby increasing intracellular fluid volume and
decreasing intracellular fluid osmolarity after osmotic equilibrium.
TMP14 pp. 312–313
60. A) For any given molecular radius, positively charged molecules (cations)
are filtered more readily than negatively charged molecules (anions)
because negative charges on the proteins of the basement membrane and
podocytes of the glomerular capillaries tend to repel large negatively
changed molecules (e.g., polycationic dextrans, curve C). Large positively
charged molecules (curve A) are filtered more readily.
TMP14 pp. 332–333
61. C) Because water is reabsorbed by the renal tubules but creatinine is not
reabsorbed, the concentration of creatinine in the renal tubular fluid will
increase as fluid flows from the proximal to the distal tubule. An increase
in the concentration from 1.0 mg/100 ml in the proximal tubule to 5.0
mg/100 ml in the distal tubule means that only about one fifth (20%) of the
water that was in the proximal tubules remains in the distal tubule.
TMP14 pp. 349–350
62. C) Peritubular capillary fluid reabsorption is determined by the balance of
hydrostatic and colloid osmotic forces in the peritubular capillaries.
Increased efferent arteriolar resistance reduces peritubular capillary
hydrostatic pressure and therefore increases the net force favoring fluid
reabsorption. Increased blood pressure tends to raise peritubular capillary
hydrostatic pressure and reduce fluid reabsorption. Decreased filtration
fraction increases the peritubular capillary colloid osmotic pressure and
tends to reduce peritubular capillary reabsorption. Decreased angiotensin
II causes vasodilatation of efferent arterioles, raising peritubular capillary
hydrostatic pressure, decreasing reabsorption, and decreasing tubular
transport of water and electrolytes. Increased renal blood flow also tends
to raise peritubular capillary hydrostatic pressure and decrease fluid
reabsorption.
TMP14 pp. 355–357
63. C) The filtered load of glucose in this example is determined as follows:
GFR (150 ml/min) × plasma glucose (300 mg/dl) = 450 mg/min. The
transport maximum for glucose in this example is 300 mg/min. Therefore,
the maximum rate of glucose reabsorption is 300 mg/min. The urinary
glucose excretion is equal to the filtered load (450 mg/min) minus the
tubular reabsorption of glucose (300 mg/min), or 150 mg/min.
TMP14 pp. 346–347, 360–361
64. B) Excess aldosterone increases sodium reabsorption and potassium
secretion by the principal cells of the collecting tubules, causing sodium
retention, increased blood pressure, and decreased renin secretion while
increasing excretion of potassium and tending to decrease plasma
potassium concentration. Excess aldosterone also causes a shift of
potassium from the extracellular fluid into the cells, further reducing
plasma potassium concentration. Aldosterone excess also stimulates
hydrogen ion secretion and bicarbonate reabsorption by the intercalated
cells and tends to increase plasma pH (alkalosis). Therefore, the classic
manifestations of excess aldosterone secretion are hypokalemia,
hypertension, alkalosis, and low renin levels.
TMP14 pp. 351–352, 384
65. B) Potassium secretion by the cortical collecting ducts is stimulated by (1)
aldosterone, (2) increased plasma potassium concentration, (3) increased
flow rate in the cortical collecting tubules, and (4) alkalosis. Therefore, a
diuretic that inhibits aldosterone, decreased plasma potassium
concentration, acute acidosis, and low sodium intake would all tend to
decrease potassium secretion by the cortical collecting tubules. A diuretic
that decreases loop of Henle sodium reabsorption, however, would tend to
increase the flow rate in the cortical collecting tubule and therefore
stimulate potassium secretion.
TMP14 pp. 386–389
66. B) This patient with diabetes mellitus and chronic renal disease has a
reduction in creatinine clearance to 40% of normal, implying a marked
reduction in GFR. He also has acidosis, as evidenced by a plasma pH of
7.14. The decrease in creatinine clearance would cause only a transient
reduction in sodium excretion and creatinine excretion rate. As the plasma
creatinine concentration increased, the urinary creatinine excretion rate
would return to normal, despite the sustained decrease in creatinine
clearance (creatinine excretion rate/plasma concentration of creatinine).
Diabetes is associated with increased production of acetoacetic acid, which
would cause metabolic acidosis and decreased plasma concentration,
as well as a compensatory increase in renal NH4
+
production and
increased NH4
+
excretion rate.
TMP14 pp. 417, 428–429
67. C) A reduction in plasma protein concentration to 3.6 g/dl would increase
the capillary filtration rate, thereby raising interstitial fluid volume and
interstitial fluid hydrostatic pressure. The increased interstitial fluid
pressure would, in turn, increase the lymph flow rate and reduce the
interstitial fluid protein concentration (“washout” of interstitial fluid
protein).
TMP14 pp. 316–319
68. B) The most likely diagnosis for this patient is diabetes insipidus, which
can account for the polyuria and the fact that her urine osmolarity is very
low (80 mOsm/l) despite overnight water restriction. In many patients
with diabetes insipidus, the plasma sodium concentration can be
maintained relatively close to normal by increasing fluid intake
(polydipsia). When water intake is restricted, however, the high urine flow
rate leads to rapid depletion of extracellular fluid volume and severe
hypernatremia, as occurred in this patient. The fact that she has no glucose
in her urine rules out diabetes mellitus. Neither primary aldosteronism nor
a renin-secreting tumor would lead to an inability to concentrate the urine
after overnight water restriction. Syndrome of inappropriate ADH would
cause excessive fluid retention and increased urine osmolarity.
TMP14 pp. 374–376, 379–380
69. C) The filterability of solutes in the plasma is inversely related to the size
of the solute (molecular weight). Also, positively charged molecules are
filtered more readily than are neutral molecules or negatively charged
molecules of equal molecular weight. Therefore, the positively charged
polycationic dextran with a molecular weight of 25,000 would be the most
readily filtered substance of the choices provided. Red blood cells are not
filtered at all by the glomerular capillaries under normal conditions.
TMP14 pp. 332–333
70. A) In normally functioning kidneys, approximately two thirds of the
water filtered by the glomerular capillaries is reabsorbed in the proximal
tubule. Although dehydration increases ADH levels and water
reabsorption by the distal tubules, collecting tubules, and collecting ducts
and this action contributes importantly to decreased water excretion in
dehydration, the total amount of water that remains in these tubular
segments is small compared with the amount of water in the proximal
tubules (see the figure below).
TMP14 pp. 372–373
71. D) Furosemide (Lasix) inhibits the Na+
-2Cl −
-K+
co-transporter in the
ascending limb of the loop of Henle. This action not only causes marked
natriuresis and diuresis but also reduces the urine-concentrating ability.
Furosemide does not cause edema; in fact, it is often used to treat severe
edema and heart failure. Furosemide also increases the renal excretion of
potassium and calcium and therefore tends to cause hypokalemia and
hypocalcemia rather than increasing the plasma concentrations of
potassium and calcium.
TMP14 pp. 350–351, 388-389, 421–422
72. B) Hypernatremia can be caused by excessive sodium retention or water
loss. The fact that the patient has large volumes of dilute urine suggests
excessive urinary water excretion. Of the two possible disturbances listed
that could cause excessive urinary water excretion (nephrogenic diabetes
insipidus and central diabetes insipidus), nephrogenic diabetes insipidus
is the most likely cause. Central diabetes insipidus (decreased ADH
secretion) is not the correct answer because plasma ADH levels are
markedly elevated. Simple dehydration due to decreased water intake is
unlikely because the patient is excreting large volumes of dilute urine.
TMP14 pp. 315–316, 374
73. D) Dehydration due to water deprivation decreases extracellular fluid
volume, which in turn increases renin secretion and decreases plasma
atrial natriuretic peptide. Dehydration also increases the plasma sodium
concentration, which stimulates the secretion of ADH. The increased ADH
increases water permeability in the collecting ducts. The ascending limb of
the loop of Henle is relatively impermeable to water, and this low
permeability is not altered by water deprivation or increased levels of
ADH.
TMP14 pp. 374–376
74. C) High urine flow occurs in type 1 diabetes because the filtered load of
glucose exceeds the renal threshold, resulting in an increase in glucose
concentration in the tubule, which decreases the osmotic driving force for
water reabsorption. Increased urine flow reduces extracellular fluid
volume and stimulates the release of ADH.
TMP14 pp. 346–347, 377
75. C) Excess aldosterone and a high-salt diet could cause serious
hypokalemia because aldosterone stimulates potassium secretion by the
renal tubules (and therefore tends to increase potassium excretion), as well
as causing a shift of potassium from the extracellular fluid into the cells. A
high-salt diet would exacerbate the hypokalemia because this would
increase collecting tubular flow rate, which would tend to further increase
renal potassium secretion. Treatment with spironolactone or a betaadrenergic blocker or Addison disease (adrenal insufficiency) would tend
to increase plasma potassium concentration. Changes in sodium and
potassium intakes over the ranges indicated would have minimal effects
on plasma potassium concentration.
TMP14 pp. 384–388
76. D) Ingestion of excess aspirin (acetylsalicylic acid) would tend to cause
metabolic acidosis, which would lead to decreases in plasma ,
decreased (due to respiratory compensation), decreased urine
excretion and increased NH4
+
excretion (renal compensation), and
increased anion gap due to increased unmeasured anions.
TMP14 pp. 417, 419–420
77. B) Approximately 30% to 40% of the filtered urea is reabsorbed in the
proximal tubule. However, the tubular fluid urea concentration increases
because urea is not nearly as permeant as water in this nephron segment.
Urea concentration increases further in the tip of the loop of Henle because
water is reabsorbed in the descending limb of the loop of Henle. Under
conditions of antidiuresis, urea is further concentrated as water is
reabsorbed and as fluid flows along the collecting ducts. Therefore, the
final urine concentration of urea is substantially greater than the
concentration in the proximal tubule or in the plasma.
TMP14 pp. 370–371
78. C) Diuretics that inhibit loop of Henle sodium reabsorption are used to
treat conditions associated with excessive fluid volume (e.g., hypertension
and heart failure). These diuretics initially cause an increase in sodium
excretion that reduces extracellular fluid volume and blood pressure, but
under steady-state conditions, the urinary sodium excretion returns to
normal, due in part to the fall in blood pressure. One of the important
adverse effects of loop diuretics is hypokalemia that is caused by the
inhibition of Na+
-2Cl −
-K+
co-transport in the loop of Henle and by the
increased tubular flow rate in the cortical collecting tubules, which
stimulates potassium secretion.
TMP14 pp. 388–389, 421–422
79. A) A reduction in the number of functional nephrons to 25% of normal
would cause a compensatory increase in GFR and urine flow rate of the
surviving nephrons and decreased urine concentrating ability. Under
steady-state conditions, the urinary creatinine excretion rate and sodium
excretion rate would be maintained at normal levels. (For further
information, see TMP14, Table 32-6.)
TMP14 pp. 428–430
80. D) Approximately 40% to 50% of the filtered urea is reabsorbed in the
proximal tubule. The distal convoluted tubule and the cortical collecting
tubules are relatively impermeable to urea, even under conditions of
antidiuresis; therefore, lile urea reabsorption takes place in these
segments. Likewise, very lile urea reabsorption takes place in the thick
ascending limb of the loop of Henle. Under conditions of antidiuresis, the
concentration of urea in the renal medullary interstitial fluid is markedly
increased because of reabsorption of urea from the collecting ducts, which
contributes to the hyperosmotic renal medulla.
TMP14 pp. 370–371
81. B) Free water clearance is calculated as urine flow rate (600 ml/2 h, or 5
ml/min) − osmolar clearance (urine osmolarity × urine flow rate/plasma
osmolarity). Therefore, free water clearance is equal to +2.5 ml/min.
TMP14 p. 374
82. E) In the absence of ADH, the late distal tubule and collecting tubules are
not permeable to water. Therefore, the tubular fluid, which is already
dilute when it leaves the loop of Henle (∼ 100 mOsm/l), becomes further
diluted as it flows through the late distal tubule and collecting tubules as
electrolytes are reabsorbed. Therefore, the final urine osmolarity in the
complete absence of ADH is less than 100 mOsm/l.
TMP14 p. 372, Figure 29-8
83. A) About 65% of the filtered potassium is reabsorbed in the proximal
tubule, and another 20% to 30% is reabsorbed in the loop of Henle.
Although most of the daily variation in potassium excretion is caused by
changes in potassium secretion in the distal and collecting tubules, only a
small percentage of the filtered potassium load can be reabsorbed in these
nephron segments. (For further information, see TMP14, Figure 30-2.)
TMP14 pp. 384–385
84. A) The proximal tubule normally absorbs approximately 65% of the
filtered water, with much smaller percentages being reabsorbed in the
descending loop of Henle and in the distal and collecting tubules. The
ascending limb of the loop of Henle is relatively impermeable to water and
therefore reabsorbs very lile water.
TMP14 pp. 349–354
85. C) The thick ascending limb of the loop of Henle is relatively
impermeable to water even under conditions of maximal antidiuresis. The
proximal tubule and descending limb of the loop of Henle are highly
permeable to water under normal conditions, as well as during
antidiuresis. Water permeability of the late distal and collecting tubules
increases markedly during antidiuresis because of the effects of increased
levels of ADH.
TMP14 pp. 372–373
86. D) Interstitial fluid volume is equal to extracellular fluid volume minus
plasma volume. Extracellular fluid volume can be estimated from the
distribution of inulin or 22Na, whereas plasma volume can be estimated
from 125I-albumin distribution. Therefore, interstitial fluid volume is
calculated from the difference between the inulin distribution space and
the 125I-albumin distribution space.
TMP14 pp. 308–310, Table 25-3
87. F) Furosemide (Lasix) is a “loop” diuretic that inhibits the Na+
-Cl −
-K+
cotransporter in the thick ascending loop of Henle, thus reducing urineconcentrating ability; increasing renal excretion of Na+
, Cl −
, and K+
; and
tending to cause hypokalemia.
TMP14 pp. 421–422
88. D) Uncontrolled type 1 diabetes would tend to cause metabolic acidosis
(decreases in plasma pH and ) due to increased metabolism of fat and
production of acetoacetic acid, which, in turn, would be associated with
increased anion gap. The normal respiratory compensation would
decrease plasma .
TMP14 p. 420
89. E) Metabolic alkalosis is associated with hypokalemia due to a shift of
potassium from the extracellular fluid into the cells (see table below). Betaadrenergic blockade, insulin deficiency, strenuous exercise, and
aldosterone deficiency all cause hyperkalemia due to a shift of potassium
out of the cells into the extracellular fluid.
TMP14 pp. 383–384, Table 5-1
Table 5-1 Factors That Can Alter Potassium Distribution Between Intracellular
and Extracellular Fluids
Factors That Shift K+ Into Cells (Decrease
Extracellular[K+
])
Factors That Shift K- Out of Cells (Increase
Extracellular[K+
])
Insulin Insulin deficiency (diabetes mellitus)
Aldosterone Aldosterone deficiency(Addison disease)
β-Adrenergic stimulation β-Adrenergic blockade
Alkalosis Acidosis
Cell lysis
Strenuous exercise
Increased extracellular fluid
osmolarity
90. D) In this example, the filtered load of glucose is equal to GFR (100
ml/min) × plasma glucose (150 mg/dl), or 150 mg/min. If there is no
detectable glucose in the urine, the reabsorption rate is equal to the filtered
load of glucose, or 150 mg/min.
TMP14 pp. 346–347, 361
91. B) Furosemide is a powerful inhibitor of the Na+
-2Cl −
-K+
co-transporter
in the loop of Henle. Thiazide diuretics primarily inhibit NaCl
reabsorption into the distal tubule, whereas carbonic anhydrase inhibitors
decrease bicarbonate reabsorption in the tubules. Amiloride inhibits
sodium channel activity, whereas spironolactone inhibits the action of
mineralocorticoids in the renal tubules. Osmotic diuretics inhibit water
and solute reabsorption by increasing osmolarity of the tubular fluid.
TMP14 p. 422
92. E) Decreased efferent arteriolar resistance would increase renal blood
flow while reducing glomerular hydrostatic pressure, which, in turn,
would tend to decrease the GFR.
TMP14 pp. 335–336
93. E) Renal tubular acidosis results from a defect of renal secretion or H+
, a
defect in reabsorption of , or both. This defect causes metabolic
acidosis associated with decreases in plasma pH and and a normal
anion gap associated with hyperchloremia (increased plasma chloride
concentration). Plasma is reduced because of respiratory compensation
for the acidosis.
TMP14 p. 417
94. D) The patient has classic symptoms of diabetes mellitus: increased thirst,
breath smelling of acetone (due to increased acetoacetic acids in the blood),
high fasting blood glucose concentration, and glucose in the urine. The
acetoacetic acids in the blood cause metabolic acidosis that leads to a
compensatory decrease in renal excretion, decreased urine pH, and
increased renal production of ammonium and . The high level of
blood glucose increases the filtered load of glucose, which exceeds the
transport maximum for glucose, causing an osmotic diuresis (increased
urine volume) due to the unreabsorbed glucose in the renal tubules acts as
an osmotic diuretic.
TMP14 pp. 346–347, 417
95. C) Intracellular fluid volume is calculated as the difference between total
body fluid (0.57 × 60 g = 34.2 kg, or ∼34.2 l) and extracellular fluid volume
(12.8 l), which equals 21.4 l.
TMP14 pp. 309–310
96. C) Plasma volume is calculated as blood volume (4.3 l) × (1.0 −
hematocrit), which is 4.3 × 0.6 = 2.58 l (rounded up to 2.6).
TMP14 pp. 309–310
97. C) Interstitial fluid volume is calculated as the difference between
extracellular fluid volume (12.8 l) and plasma volume (2.6 l), which is
equal to 10.2 l.
TMP14 pp. 309-310
98. C) The primary site of reabsorption of magnesium is in the loop of Henle,
where about 65% of the filtered load of magnesium is reabsorbed. The
proximal tubule normally reabsorbs only about 25% of filtered
magnesium, and the distal and collecting tubules reabsorb less than 5%.
TMP14 p. 392
99. C) A plasma glucose concentration of 300 mg/dl would increase the
filtered load of glucose above the renal tubular transport maximum and
therefore increase urinary glucose excretion. The unreabsorbed glucose in
the renal tubules would also cause an osmotic diuresis, increased urine
volume, and decreased extracellular fluid volume, which would stimulate
thirst. Increased glucose also causes vasodilatation of afferent arterioles,
which increases GFR.
TMP14 pp. 341–342, 346–347, 378–379
100. B) GFR is approximately equal to the clearance of creatinine. Creatinine
clearance = urine creatinine concentration (32 mg/dl) × urine flow rate
(3600 ml/24 h, or 2.5 ml/min) ÷ plasma creatinine concentration (4 mg/dl) =
20 ml/min.
TMP14 pp. 361–362
101. D) The net renal tubular reabsorption rate is the difference between the
filtered load of potassium (GFR × plasma potassium concentration) and the
urinary excretion of potassium (urine potassium concentration × urine
flow rate). Therefore, the net tubular reabsorption of potassium is 0.075
mmol/min.
TMP14 pp. 361–362
102. D) Severe diarrhea would result in loss of in the stool, thereby
causing metabolic acidosis that is characterized by low plasma and
low pH. Respiratory compensation would reduce . The plasma anion
gap would be normal, and the plasma chloride concentration would be
elevated (hyperchloremic metabolic acidosis) in metabolic acidosis caused
by loss in the stool.
TMP14 pp. 415–420
103. E) Primary excessive secretion of aldosterone causes metabolic alkalosis
due to increased secretion of hydrogen ions and reabsorption by the
intercalated cells of the collecting tubules. Therefore, the metabolic
alkalosis would be associated with increases in plasma pH and , with
a compensatory reduction in respiration rate and increased . The
plasma anion gap would be normal, with a slight reduction in plasma
chloride concentration.
TMP14 pp. 417–420
104. D) Proximal tubular acidosis results from a defect of renal secretion of
hydrogen ions, reabsorption of bicarbonate, or both. This defect leads to
increased renal excretion of and metabolic acidosis characterized by
low plasma concentration, low plasma pH, a compensatory increase
in respiration rate and low , and a normal anion gap with an increased
plasma chloride concentration.
TMP14 pp. 415–420
105. F) A patient with diabetic ketoacidosis and emphysema would be
expected to have metabolic acidosis (due to excess ketoacids in the blood
caused by diabetes), as well as increased plasma due to impaired
pulmonary function. Therefore, the patient would be expected to have
decreased plasma pH, decreased , increased , and an increased
anion gap due to the addition of ketoacids to the
blood.
TMP14 pp. 416–420
106. D) Secretion of hydrogen ions and reabsorption of depend critically
on the presence of carbonic anhydrase in the renal tubules. After inhibition
of carbonic anhydrase, renal tubular secretion of hydrogen ions and
reabsorption of would decrease, leading to increased renal excretion
of , reduced plasma concentration, and metabolic acidosis. The
metabolic acidosis, in turn, would stimulate the respiration rate, leading to
decreased . The plasma anion gap would be within the normal range.
TMP14 pp. 410–411, 420, 422
107. B) Acute renal failure caused by tubular necrosis would cause the rapid
development of metabolic acidosis due to the kidneys’ failure to rid the
body of the acid waste products of metabolism. The metabolic acidosis
would lead to decreased plasma concentration. Acute renal failure
would also lead to a rapid increase in blood urea nitrogen concentration
and a significant increase in plasma potassium concentration due to the
kidneys’ failure to excrete electrolytes or nitrogenous waste products.
Necrosis of the renal epithelial cells causes them to slough away from the
basement membrane and plug up the renal tubules, thereby increasing
hydrostatic pressure in Bowman’s capsule and decreasing GFR.
TMP14 pp. 425, 430
108. E) Intracellular and extracellular body fluids have the same total
osmolarity under steady-state conditions because the cell membrane is
highly permeable to water. Therefore, water flows rapidly across the cell
membrane until osmotic equilibrium is achieved. The colloid osmotic
pressure is determined by the protein concentration, which is considerably
higher inside the cell. The cell membrane is also relatively impermeable to
potassium, sodium, and chloride, and active transport mechanisms
maintain low intracellular concentrations of sodium and chloride and a
high intracellular concentration of potassium.
TMP14 pp. 310–311
109. B) Fluid entering the early distal tubule is almost always hypotonic
because sodium and other ions are actively transported out of the thick
ascending loop of Henle, whereas this portion of the nephron is virtually
impermeable to water. For this reason, the thick ascending limb of the loop
of Henle and the early part of the distal tubule are often called the diluting
segment.
TMP14 pp. 350–351
110. D) Chronic metabolic acidosis is, by definition, associated with
decreased plasma concentration. Decreased excretion of NH4Cl and
occurs with renal compensation for the acidosis, and respiratory
compensation for the acidosis increases the ventilation rate, resulting in
decreased plasma .
TMP14 pp. 415-418
111. B) A 0.45% NaCl solution is hypotonic. Therefore, administration of 2.0 l
of this solution would reduce intracellular and extracellular fluid
osmolarity and cause increases in intracellular and extracellular volumes.
TMP14 pp. 312-313
112. C) If the renal clearance is greater than the GFR, this implies that there
must be secretion of that substance into the renal tubules. A substance that
is freely filtered and not secreted or reabsorbed would have a renal
clearance equal to the GFR.
TMP14 p. 364
113. A) In the proximal tubule, calcium reabsorption usually parallels sodium
and water reabsorption. With extracellular volume expansion or increased
blood pressure, proximal sodium and water reabsorption are reduced, and
a reduction in calcium reabsorption also occurs, causing increased urinary
excretion of calcium. Increased parathyroid hormone, increased plasma
phosphate concentration, and metabolic alkalosis all tend to decrease the
renal excretion of calcium.
TMP14 pp. 390–391
114. C) Increasing sodium intake would decrease renin secretion and plasma
renin activity, as well as reduce plasma aldosterone concentration and
increase plasma atrial natriuretic peptide because of a mod est expansion
of extracellular fluid volume. Although a high sodium intake would
initially increase distal NaCl delivery, which would tend to increase
potassium excretion, the decrease in aldosterone concentration would
offset this effect, resulting in no change in potassium excretion under
steady-state conditions. Even very large increases in sodium intake cause
only minimal changes in plasma sodium concentration as long as the
ADH–thirst mechanisms are fully operative.
TMP14 pp. 388–389
115. C) A 50% reduction in efferent arteriolar resistance would cause a large
decrease in GFR—greater than 10%. A decrease in renal artery pressure
from 100 to 85 mm Hg would cause only a slight decrease in GFR in a
normal, autoregulating kidney. A decrease in afferent arteriole resistance,
a decrease in plasma colloid osmotic pressure, or an increase in the
glomerular capillary filtration coefficient would all tend to increase GFR.
TMP14 pp. 333–336, 338–339
116. D) Acute metabolic acidosis reduces intracellular potassium
concentration, which, in turn, decreases potassium secretion by the
principal cells of the collecting tubules. The primary mechanism by which
increased hydrogen ion concentration inhibits potassium secretion is by
reducing the activity of the sodium-potassium adenosine triphosphatase
pump. This action then reduces intracellular potassium concentration,
which, in turn, decreases the rate of passive diffusion of potassium across
the luminal membrane into the tubule.
TMP14 p. 389
117. D) In a dehydrated person, osmolarity in the early distal tubule is
usually less than 300 mOsm/l because the ascending limb of the loop of
Henle and the early distal tubule are relatively impermeable to water, even
in the presence of ADH. Therefore, the tubular fluid becomes
progressively more dilute in these segments compared with plasma. ADH
does not influence water reabsorption in the ascending limb of the loop of
Henle. The ascending limb, however, reabsorbs sodium to a much greater
extent than does the descending limb. Another important action of ADH is
to increase the urea permeability in the medullary collecting ducts, which
contributes to the hyperosmotic renal medullary interstitium in
antidiuresis.
TMP14 pp. 372–373
118. E) Most potassium secretion occurs in the collecting tubules. A highpotassium diet stimulates potassium secretion by the collecting tubules
through multiple mechanisms, including small increases in extracellular
potassium concentration, as well as increased levels of aldosterone.
TMP14 pp. 386–387
119. A) Diabetic ketoacidosis results in a metabolic acidosis that is
characterized by a decrease in plasma bicarbonate concentration, increased
anion gap (due to the addition of unmeasured anions to the extracellular
fluid along with the ketoacids), and a renal compensatory response that
includes increased secretion of NH4
+
. There is also an increased
respiratory rate with a reduction in arterial , as well as decreased urine
pH and decreased renal excretion.
TMP14 pp. 415–420
120. B) Interstitial fluid volume cannot be measured directly, but it can be
calculated as the difference between extracellular fluid volume (inulin
space = 16 l) and plasma volume (125I-albumin space = 4 l). Therefore,
interstitial fluid volume is approximately 12 l.
TMP14 pp. 308–310
121. E) Increased levels of insulin cause a shift of potassium from the
extracellular fluid into the cells. All the other conditions have the reverse
effect of shifting potassium out of the cells into the extracellular fluid.
TMP14 pp. 383–384
122. D) This patient is severely dehydrated as a result of sweating and lack of
adequate fluid intake. The dehydration markedly stimulates the release of
ADH and renin secretion, which in turn stimulates the formation of
angiotensin II and aldosterone secretion.
TMP14 pp. 359, 375
123. E) After running the race and losing both fluid and electrolytes, this
person replaces his fluid volume by drinking 2 l of water. However, he did
not replace the electrolytes. Therefore, he would be expected to experience
a decrease in plasma sodium concentration, resulting in a decrease in both
intracellular and extracellular fluid osmolarity. The decrease in
extracellular fluid osmolarity would lead to an increase in intracellular
volume as fluid diffused into the cells from the extracellular compartment.
Therefore, after drinking the water and absorbing it, the total body volume
would be normal but intracellular volume would be increased and
extracellular volume would be reduced.
TMP14 pp. 312–314
124. B) Increased levels of parathyroid hormone stimulate calcium
reabsorption in the thick ascending loops of Henle and distal tubules.
Extracellular fluid volume expansion, increased blood pressure, decreased
plasma phosphate concentration, and metabolic acidosis are all associated
with decreased calcium reabsorption by the renal tubules.
TMP14 pp. 390–391
125. C) In this example, the acidosis is associated with a reduced plasma
bicarbonate concentration, signifying metabolic acidosis. In addition, the
patient also has an elevated , signifying respiratory acidosis. Therefore,
the patient has simultaneous respiratory and metabolic acidosis.
TMP14 pp. 415–420
126. A) Chronic respiratory acidosis is caused by insufficient pulmonary
ventilation, resulting in an increase in . Acidosis, in turn, stimulates the
secretion of hydrogen ions into the tubular fluid and increased renal
tubular production of NH4
+
, which further contributes to the excretion of
hydrogen ions and the renal production of , thereby increasing
plasma bicarbonate concentration. The increased tubular secretion of
hydrogen ions also reduces urine pH.
TMP14 pp. 415–416
127. A) Dilation of the afferent arterioles leads to an increase in the
glomerular hydrostatic pressure and therefore an increase in GFR, as well
as an increase in renal blood flow. Increased glomerular capillary filtration
coefficient would also raise the GFR but would not be expected to alter
renal blood flow. Increased plasma colloid osmotic pressure or dilation of
the efferent arterioles would both tend to reduce the GFR. Increased blood
viscosity would tend to reduce renal blood flow and GFR.
TMP14 pp. 333–337
128. B) Nonsteroidal antiinflammatory drugs inhibit the synthesis of
prostaglandins, which, in turn, causes constriction of afferent arterioles
that can reduce the GFR. The decrease in GFR, in turn, leads to an increase
in serum creatinine. Increased efferent arteriole resistance and increased
glomerular capillary filtration coefficient would both tend to increase
rather than reduce GFR. Increasing muscle mass due to exercise would
cause very lile change in serum creatinine.
TMP14 pp. 333–338
129. A) In this example, the plasma sodium concentration is markedly
increased but the urine sodium concentration is relatively normal, and
urine osmolarity is almost maximally increased to 1200 mOsm/l. In
addition, there are increases in plasma renin, ADH, and aldosterone,
which is consistent with dehydration caused by decreased fluid intake.
The syndrome of inappropriate ADH would result in a decrease in plasma
sodium concentration, as well as suppression of renin and aldosterone
secretion. Nephrogenic diabetes insipidus, caused by the kidneys’ failure
to respond to ADH, would also be associated with dehydration, but urine
osmolarity would be reduced rather than increased. Primary
aldosteronism would tend to cause sodium and water retention with only
a modest change in plasma sodium concentration and a marked reduction
in the secretion of renin. Likewise, a renin-secreting tumor would be
associated with increases in plasma aldosterone concentration and plasma
renin activity but only a modest change in plasma sodium concentration.
TMP14 pp. 375–376, 378–380
Unit VI: Blood Cells, Immunity,
and Blood Coagulation
The following table of normal test values can be referenced
throughout Unit VI.
Test Normal Values
Erythrocyte count Male: 4.3-5.9 million/µl
Female: 3.5-5.5 million/µl
Hematocrit Male: 41%-53%
Female: 36%-46%
Hemoglobin, blood Male: 13.5-17.5 g/dl
Female: 12.0-16.0 g/dl
Mean corpuscular hemoglobin 25.4-34.6 pg/cell
Mean corpuscular hemoglobin concentration 31%-36% hemoglobin
Mean corpuscular volume
80-100 fl
Reticulocyte count 0.5%-1.5% of red blood cells
Platelet count 150,000-400,000/µl
Leukocyte count and differential
Leukocyte count 4500-11,000/µl
Neutrophils 54%-62%
Eosinophils 1%-3%
Basophils 0-0.75%
Lymphocytes 25%-33%
Monocytes 3%-7%
Partial thromboplastin time (activated) 25-40 seconds
Prothrombin time
11-15 seconds
Bleeding time
2-7 minutes
1. A 40-year-old woman visits the clinic complaining of fatigue.
She had recently been treated for an infection. Her laboratory
values are as follows: red blood cell (RBC) count, 1.8 × 106
/µl;
hemoglobin (Hb), 5.2 g/dl; hematocrit (Hct), 15; white blood
cell (WBC) count, 7.6 × 103
/µl; platelet count, 320,000/µl;
mean corpuscular volume (MCV), 92 fL; and reticulocyte
count, 24%. What is the most likely explanation for this
presentation?
A) Aplastic anemia
B) Hemolytic anemia
C) Hereditary spherocytosis
D) B12
deficiency
2. What RBC enzyme facilitates transport of carbon dioxide
(CO2
)?
A) Myeloperoxidase
B) Carbonic anhydrase
C) Superoxide dismutase
D) Globin reductase
3. A patient presents with hemoglobin of 7.9 g/dl, hematocrit of
23%, and mean corpuscular volume of 89 fl. Erythropoietin
level is quite low. What is the most likely diagnosis?
A) Sickle cell anemia
B) Pernicious anemia
C) End-stage renal disease
D) Chronic blood loss
4. A patient has a hematocrit of 63% and a reduced
erythropoietin level. Based on this information alone, what is
the most likely diagnosis?
A) Prolonged and continued exposure to high altitude
B) Polycythemia vera
C) Chronic obstructive pulmonary disease with hypoxemia
D) Hemochromatosis
5. A 74-year-old man has a hemoglobin of 6.0 g/dl, hematocrit of
19%, mean corpuscular volume of 117, and large, bizarrely
shaped erythrocytes on peripheral blood smear. His serum
folic acid levels are normal. What is the likely cause of this
disorder?
A) Insufficient intrinsic factor
B) Vitamin K deficiency
C) Hepatic cirrhosis
D) Alpha thalassemia
6. When red blood cells break down, the porphyrin portion of
hemoglobin is converted by macrophages to what
product(s)?
A) Xanthine
B) Branched-chain amino acids
C) Homocysteine
D) Bilirubin
7. How many oxygen atoms can be transported by each
hemoglobin molecule?
A) 2
B) 4
C) 8
D) 16
8. During the second trimester of pregnancy, where is the
predominant site of RBC production in the embryo?
A) Yolk sac
B) Bone marrow
C) Lymph nodes
D) Liver
9. What function do vitamin B12
and folic acid perform that is
critical to hematopoiesis?
A) Support porphyrin production
B) Serve as cofactors for iron uptake
C) Support terminal differentiation of erythroid and
myeloid cells
D) Support production of thymidine triphosphate
10. A 62-year-old man complains of headaches, visual
difficulties, and chest pains. Physical examination reveals a
red complexion and a large spleen. His complete blood cell
count (CBC) is as follows: Hct, 58%; WBC, 13,300/µl; and
platelets, 600,000/µl. His arterial oxygen saturation is 97% on
room air. Which treatment would you recommend?
A) Chemotherapy
B) Phlebotomy
C) Iron supplement
D) Inhaled oxygen therapy
11. A 38-year-old healthy woman comes to you for a routine
visit. She has spent the past 2 months hiking through the
Himalayas and climbed to the base camp of Mount Everest.
Which results would you expect to see on her CBC?
Hematocrit RBC count WBC count MCV
A) ↑ ↑ ↑ ↑
B) ↑ ↑ ↔ ↑
C) ↑ ↑ ↔ ↔
D) ↑ ↔ ↔ ↔
E) ↔ ↑ ↑ ↔
F) ↑ ↔ ↑ ↑
G) ↔ ↑ ↔ ↑
12. A 24-year-old African American man comes to the
emergency department 3 hours after the onset of severe back
and chest pain. These problems started while he was walking
briskly through a hilly neighborhood. He has had similar
episodes numerous times in the past. He is in obvious pain.
Laboratory studies show the following values:
Hemoglobin = 11 g/dl
Leukocyte count = 22,000/µl3
Reticulocyte count = 3.5%
What is this patient’s diagnosis?
A) Acute blood loss
B) Sickle cell anemia
C) Anemia of chronic disease
D) End-stage renal disease
13. After a person is placed in an atmosphere with low oxygen,
how long does it take for increased numbers of reticulocytes
to develop?
A) 6 hours
B) 12 hours
C) 3 days
D) 5 days
E) 2 weeks
14. A patient presents to your office complaining of extreme
fatigue and shortness of breath on exertion that has gradually
worsened during the past 2 weeks. Physical examination
reveals a well-nourished woman who appears comfortable
but somewhat short of breath. Her vital signs include a pulse
of 120, a respiratory rate of 20, and blood pressure of 120/70.
When she stands up, her pulse increases to 150, and her
blood pressure falls to 80/50. Her hematologic values are as
follows: Hb, 7 g/dl; Hct, 20%; RBC count, 2 × 106
/µl; and
platelet count, 400,000/µl. On a peripheral smear, her RBCs
are microcytic and hypochromic. What is your diagnosis?
A) Aplastic anemia
B) Renal failure
C) Chronic and acute blood loss with iron-deficiency
anemia
D) Sickle cell anemia
E) Megaloblastic anemia
15. Which phagocytes can extrude digestion products and
continue to survive and function for many months?
A) Neutrophils
B) Basophils
C) Macrophages
D) Eosinophils
16. What cells ingest foreign material and micro-organisms
within the sinusoids of the liver?
A) Dendritic cells
B) Megakaryocytes
C) Basophils
D) Kupffer cells
17. A 45-year-old man presents to the emergency department
with a 2-week history of diarrhea that has goen
progressively worse during the past several days. He has
minimal urine output and is admied to the hospital for
dehydration. His stool specimen is positive for parasitic eggs.
Which type of WBC would have an elevated number?
A) Eosinophils
B) Neutrophils
C) T lymphocytes
D) B lymphocytes
E) Monocytes
18. A 24-year-old man came to the emergency department with
a broken leg. A blood test revealed his WBC count to be 22 ×
103
/µl. Five hours later, a second blood test revealed a WBC
count of 7 × 103
/µl. What is the cause of the increased WBC
count in the first test?
A) Increased production of WBCs by the bone marrow
B) Release of preformed, mature WBCs into the circulation
C) Decreased destruction of WBCs
D) Increased production of selectins
19. A 62-year-old man who was known to have a normal blood
cell count and differential count 3 months ago presents with
pallor, bone pain, bruising, and a WBC count of 42,000.
Eighty-five percent of cells in the circulation appear to be
immature granulocytes. What is the diagnosis?
A) Acute lymphocytic leukemia
B) Acute myelocytic leukemia
C) Chronic lymphocytic leukemia
D) Chronic myelocytic leukemia
20. What cell type can interact with IgE and release large
amounts of histamine, bradykinin, serotonin, heparin,
lysosomal enzymes, and other inflammatory mediators?
A) Neutrophils
B) Basophils and mast cells
C) Eosinophils
D) Monocytes
y
21. A 60-year-old man is noted on a routine physical
examination to have a significantly elevated white blood cell
(WBC) count of 22,000/µl3
, and 80% of cells are matureappearing cells with large, round nuclei and scant cytoplasm.
A review of the record shows that the WBC count has been
moderately elevated for at least 18 months. What is the likely
diagnosis?
A) Acute lymphocytic leukemia
B) Chronic lymphocytic leukemia
C) Acute myelocytic leukemia
D) Chronic myelocytic leukemia
22. Where does the transmigration of WBCs occur in response
to infectious agents?
A) Arterioles
B) Lymphatic ducts
C) Capillaries and venules
D) Inflamed arteries
23. An 8-year-old boy frequently comes to the clinic for
persistent skin infections that do not heal within a normal
time frame. He had a normal recovery from the measles. A
check of his antibodies after immunizations yielded normal
antibody responses. A defect in which of the following cells
would most likely be the cause of the frequent and prolonged
infections?
A) B lymphocytes
B) Plasma cells
C) Neutrophils
D) Macrophages
E) CD4 T lymphocytes
24. Which cell type migrates into inflammatory sites to clean up
necrotic tissue and direct tissue remodeling?
A) Neutrophil
B) Macrophage
C) Dendritic cell
D) Eosinophil
25. A 3-year-old child who has had frequent ear infections is
found to have reduced immunoglobulin levels and is
unresponsive to vaccination with tetanus toxoid. However,
the child has normal skin test reactivity (delayed redness and
induration) to a common environmental antigen. Which cell
lineage is not functioning normally?
A) Macrophages
B) Helper T cells
C) Cytotoxic T cells
D) B cells
26. Patients with human immunodeficiency virus (HIV) exhibit
abnormal functioning of which of the following mechanisms?
A) Antibody production only
B) T cell–mediated cytotoxicity only
C) Degranulation of appropriately stimulated mast cells
D) Both antibody production and T cell–mediated
cytotoxicity
27. What is the term for binding of IgG and complement to an
invading microbe to facilitate recognition?
A) Chemokinesis
B) Opsonization
C) Phagolysosome fusion
D) Signal transduction
28. Presentation by a cell of a foreign antigen bound to major
histocompatibility complex (MHC)-I will result in which of
the following?
A) Generation of antibodies
B) Activation of cytotoxic T cells
C) Increase in phagocytosis
D) Release of histamine by mast cells
29. The variable and hypervariable regions of immunoglobulins
that are responsible for antigen binding and specificity reside
in which part(s) of the immunoglobulin molecule?
A) The heavy chains
B) The light chains
C) The hinge regions
g g
D) Both the heavy chains and the light chains
E) The FC region
30. Fluid exudation into the tissue in an acute inflammatory
reaction is due to which of the following?
A) Decreased blood pressure
B) Decreased protein in the interstitium
C) Obstruction of the lymph vessels
D) Increased cloing factors
E) Increased vascular permeability
31. Which immune molecules can bind intact antigen?
A) T cell receptors
B) MHC class I
C) MHC class II
D) Immunoglobulins
32. What mechanisms are primarily responsible for producing a
lip ulcer from herpes simplex virus?
A) Activation of bradykinin by antibodies directed against
herpes antigens
B) Rapid influx of neutrophils and dendritic cells in
response to virally-derived chemotactic factors
C) Presentation of viral antigens by MHC class I and
engagement by cytotoxic T cells
D) Presentation of viral antigens by MHC class II and
engagement by cytotoxic T cells
33. What is the function of IL-2 in the immune response?
A) Binds to and presents antigen
B) Stimulates proliferation of T cells
C) Kills virus-infected cells
D) Is required for an anaphylactic response
34. Which of the following is true about helper T cells?
A) They are activated by the presentation of antigen by an
infected cell
B) They require the presence of a competent B-cell system
C) They destroy bacteria by phagocytosis
D) They are activated by the presentation of antigen by
macrophages or dendritic cells
p g
35. Which of the following applies to cytotoxic T cells?
A) They require the presence of a competent B-lymphocyte
system
B) They require the presence of a competent suppressor Tlymphocyte system
C) They are activated by the presentation of foreign
antigen by an infected cell
D) They destroy bacteria by initiating macrophage
phagocytosis
36. A 9-year-old girl has nasal discharge and itching of the eyes
in the spring every year. An allergist performs a skin test
using a mixture of grass pollens. Within a few minutes, the
girl exhibits a focal redness and swelling at the test site. This
response is most likely due to
A) Antigen–antibody complexes being formed in blood
vessels in the skin
B) Activation of neutrophils due to injected antigens
C) Activation of CD4 helper cells and the resultant
generation of specific antibodies
D) Activation of cytotoxic T lymphocytes to destroy
antigens
37. Activation of the complement system results in which
action?
A) Binding of the invading microbe with IgG
B) Inactivation of eosinophils
C) Decreased tissue levels of complement
D) Generation of chemotactic substances
38. Which statement is true concerning erythroblastosis fetalis
(hemolytic disease of the newborn [HDN])?
A) HDN occurs when an Rh-positive mother has an Rhnegative child
B) HDN is prevented by giving the mother a blood
transfusion
C) A complete blood transfusion after the first birth will
prevent HDN
D) The father of the child must be Rh positive
p
39. A couple requests blood typing of a newborn male (man is
AB, Rh-positive; woman is A, Rh-negative).
Hemagglutination assays show that the child is type O, Rhpositive. Which of the following conclusions regarding the
child’s parentage is valid?
A) The child could be the natural offspring of this couple
B) The mother could be the natural mother, but the father
could not be the natural father
C) The father could be the natural father, but the mother
could not be the natural mother
D) Neither the father nor the mother could be the natural
parents
40. A woman whose blood type is A, Rh positive, and a man
whose blood type is B, Rh positive, come to the clinic with a
3-year-old girl whose blood type is O, Rh negative. What can
be said about the relationship of these two adults to this
child?
A) The woman could be the child’s natural mother, but the
man could not be the natural father
B) The man could be the child’s natural father, but the
woman could not be the natural mother
C) Neither adult could be the natural parent of this child
D) This couple could be the natural parents of this child
41. What is the appropriate treatment for an infant born with
severe HDN (erythroblastosis fetalis)?
A) Passive immunization with anti-Rh(D) immunoglobulin
B) Immunization with Rh(D) antigen
C) Exchange transfusion with Rh(D)-positive blood
D) Exchange transfusion with Rh(D)-negative blood
42. Chronic allograft rejection results primarily from the actions
of what effector cell type?
A) Activated macrophages
B) Helper T lymphocytes
C) Cytotoxic T lymphocytes
D) Dendritic cells
43. Which of the following transfusions will result in an
immediate transfusion reaction?
A) O Rh-negative whole blood to an O Rh-positive patient
B) A Rh-negative whole blood to a B Rh-negative patient
C) AB Rh-negative whole blood to an AB Rh-positive
patient
D) B Rh-negative whole blood to a B Rh-negative patient
44. Given optimal transplantation technique, the following
types of grafts may be expected to succeed, with long-term
graft acceptance.
A) Allografts and xenografts
B) Autografts and allografts
C) Isografts and autografts
D) Allografts and isografts
E) Grafts with at least a 3/6 HLA match
45. After trauma, a person whose blood type is AB, Rh negative
and who has never had a transfusion before receives four
units of appropriately cross-matched type A, Rh positive
blood. What is the likely outcome?
A) Fever, chills, shortness of breath, circulatory shock, and
renal shutdown
B) Rapid hemolysis of the transfused cells
C) Moderate, intracellular hemolysis over a period of a few
weeks
D) Anaphylactic shock
46. Which transfusion will result in an immediate transfusion
reaction? Assume that the patient has never had a
transfusion.
A) Type O Rh-negative packed cells to an AB Rh-positive
patient
B) Type A Rh-positive packed cells to an A Rh-negative
patient
C) Type AB Rh-positive packed cells to an AB Rh-positive
patient
D) Type A Rh-positive packed cells to an O Rh-positive
patient
p
47. Which antigens must be matched optimally between donors
and recipients of solid organ transplants?
A) Class I human leukocyte antigen (HLA) antigens only
B) Class II HLA antigens only
C) Class I and Class II HLA antigens only
D) Class I and Class II HLA antigens and ABO antigens
48. A woman whose blood type is A positive and who has
always been healthy just delivered her second child. The
father’s blood type is O negative. Because the child’s blood
type is O negative (O, Rh negative), what would you expect
to find in this child?
A) Erythroblastosis fetalis due to rhesus incompatibility
B) Erythroblastosis fetalis due to ABO blood group
incompatibility
C) Both A and B
D) The child would not be expected to have HDN
49. A 55-year-old man who has been undergoing stable and
successful anticoagulation with warfarin for recurrent deep
vein thrombosis is treated for pneumonia, and 8 days later he
presents with lower intestinal bleeding. His prothrombin
time is quite prolonged. What is the appropriate therapy?
A) Treatment with tissue plasminogen activator
B) Infusion of calcium citrate
C) Treatment with fresh-frozen plasma and vitamin K
D) Rapid infusion of protamine
50. A 2-year-old boy bleeds excessively from minor injuries and
has previously had bleeding gums. The maternal grandfather
has a bleeding disorder. The child’s physical examination
shows slight tenderness of his knee with fluid accumulation
in the knee joint. You suspect this patient is deficient in
which coagulation factor(s)?
A) Prothrombin activator
B) Factor II
C) Factor VIII or Factor IX
D) Factor X
51. A Viet Nam veteran with chronic, severe hepatitis C has
developed an intractable nosebleed. His platelet count is
180,000, and his international normalized ratio (INR) is
markedly increased at 7.0. In addition to local compression at
the bleeding site what is the appropriate therapy?
A) Coumarin
B) Fresh-frozen plasma
C) Platelet transfusion
D) Fibrinogen infusion
52. Which agent is not effective as an in vitro anticoagulant?
A) Heparin
B) Warfarin (Coumadin)
C) Ethylenediamine tetraacetic acid (EDTA)
D) Sodium citrate
53. Hemophilia A affects which pathway of blood coagulation?
A) Intrinsic pathway only
B) Extrinsic pathway only
C) Intrinsic and extrinsic pathways only
D) Intrinsic, extrinsic, and common pathways
54. A homeless person is brought in from a vacant lot with signs
of sepsis and with persistent bleeding from his gums,
phlebotomy sites, and around IV catheters. His white blood
cell count is 15,000 with normal cellular morphology and a
few immature neutrophils (band cells), and his platelet count
is somewhat reduced at 95,000. What is the most likely
diagnosis?
A) Reduced fibrinogen concentration due to liver disease
B) Inhibition of platelet function by methyl alcohol
ingestion
C) Dietary deficiency of vitamin B12 and folic acid
D) Disseminated intravascular coagulation
55. Which coagulation pathway begins with tissue
thromboplastin?
A) Extrinsic pathway
B) Intrinsic pathway
C) Common pathway
p y
D) Fibrin stabilization
56. A 56-year-old electrician has severe but undiagnosed gluten
sensitivity with persistent diarrhea and weight loss. He has
excessive bleeding after a dental extraction and is found to
have a markedly prolonged prothrombin time of 37 seconds.
What is the most likely cause of his excessive bleeding?
A) Hepatocellular disease
B) Vitamin K deficiency
C) Protein malnutrition with inadequate fibrinogen
D) Loss of cloing factors through the gastrointestinal
mucosa
57. Which of the following would best explain a prolonged
bleeding time test?
A) Hemophilia A
B) Hemophilia B
C) Thrombocytopenia
D) Coumadin use
58. Which of the following is appropriate therapy for a massive
pulmonary embolism?
A) Citrated plasma
B) Warfarin
C) Aspirin
D) Tissue plasminogen activator
59. What is the primary mechanism by which heparin prevents
blood coagulation?
A) Antithrombin III activation
B) Binding and inhibition of tissue factor
C) Binding available calcium
D) Inhibition of platelet-activating factor
60. A 14-year-old boy has a life long history of excessive
bleeding, often involving bleeding into his joints
(hemarthrosis). There is a positive history of a similar
disorder in several male relatives. You may assume that he
inherited this disorder from which parent(s)?
A) His father
B) His mother
C) Both parents
D) Penetrance of this disorder is variable and difficult to
interpret
Answers
1. B) This patient has increased production of RBCs as indicated by a
markedly increased reticulocyte count in the seing of significant
anemia (low red blood cell number, hemoglobin, and hematocrit).
The RBCs being produced have a normal size (MCV = 90), and
thus the patient does not have spherocytosis (small RBCs) or
vitamin B12 deficiency (large RBCs). The normal WBC count and
the increased reticulocyte count suggest that the bone marrow is
functioning. The increased reticulocyte count means that a large
number of RBCs are being produced. These laboratory values
support an anemia due to some type of blood loss—in this case an
anemia due to hemolysis.
TMP14 pp. 440–443, 446
2. B) Carbonic anhydrase catalyzes the reaction of CO2
with water to
allow large amounts of CO2
to be transported in blood as soluble
bicarbonate ion.
TMP14 p. 439
3. C) In end-stage renal disease, the kidneys cease to make
erythropoietin, leading to severe anemia. The types of anemia in
the other three possible answers lead to red blood cell loss or
destruction and increased erythropoietin due to tissue hypoxemia.
TMP14 p. 442
4. B) In polycythemia vera autonomous, clonal production of red
blood cells results in increased hemoglobin, hematocrit, and red
blood cell count and decreased erythropoietin, because hypoxemic
drive for “epo” production is reduced. High-altitude exposure and
chronic obstructive pulmonary disease are both associated with
tissue hypoxemia that drives increased erythropoietin production.
Hemochromatosis is a disorder of systemic iron overload with
normal red blood cell production.
TMP14 pp. 442, 447
5. A) The clinical presentation is that of a megaloblastic anemia,
which can be caused by a deficiency of either vitamin B12 or folic
acid. Because this patient’s folic acid levels are normal, the cause
must be vitamin B12 deficiency caused by atrophy of the gastric
mucosa resulting in insufficient production of intrinsic factor,
needed for normal gastrointestinal absorption of vitamin B12.
TMP14 p. 443
6. D) Macrophages convert the porphyrin portion of hemoglobin to
bilirubin.
TMP14 p. 445
7. C) Each hemoglobin molecule has four globin chains (in
hemoglobin A, the predominant form in adults, the hemoglobin
molecule includes two alpha and two beta chains). Each globin
chain is associated with one heme group, containing one atom of
iron. Each of the four iron atoms can bind loosely with one
molecule (two atoms) of oxygen. Thus, each hemoglobin molecule
can transport eight oxygen atoms.
TMP14 p. 444
8. D) RBC production begins in the yolk sac for the first trimester.
Production in the yolk sac decreases at the beginning of the
second trimester, and the liver becomes the predominant source of
RBC production. RBC production from the bone marrow increases
during the third trimester and continues throughout life.
TMP14 pp. 439–440
9. D) Cell proliferation requires DNA replication, which requires an
adequate supply of thymidine triphosphate. Both vitamin B12 and
folate are needed to make thymidine triphosphate.
TMP14 p. 443
10. B) This patient has polycythemia vera: increased RBCs, WBCs,
and platelets. Normal arterial oxygen saturation shows that there
is no reason for a compensatory polycythemia. His increased
hematocrit also increases the viscosity of the blood, resulting in
increased afterload for the heart, which is probably the reason for
his chest pain. Thus, phlebotomy (bleeding) is needed to decrease
his elevated blood cell count and blood viscosity.
TMP14 p. 447
11. C) Secondary polycythemia has developed because of exposure
to low oxygen levels. She will have an increased hematocrit level
and RBC count but a normal WBC count. The cells are normal, so
the MCV will be normal.
TMP14 p. 447
12. B) This African American man has sickle cell anemia, as
demonstrated by his decreased hemoglobin concentration,
moderately elevated reticulocyte count, and history of painful
crises. His elevated WBC count suggests a response to stress
hormones, with recruitment of mature leukocytes into the
circulation. Increased oxygen demand from walking in hilly
terrain likely produced tissue ischemia with resultant sickling of
his RBCs.
TMP14 p. 446
13. C) EPO levels increase in response to decreased arterial oxygen
level, with the maximum EPO production occurring within 24
hours. It takes 3 days for new reticulocytes to appear in the
circulation, and after a total of 5 days from the beginning of
hypoxemia, these reticulocytes will be circulating as mature
erythrocytes. Because it takes 1 to 2 days for a reticulocyte to
become an erythrocyte, the correct answer is 3 days until the
person has an increased number of reticulocytes.
TMP14 pp. 441–442
14. C) The blood cell count shows that the patient is anemic. Her
bone marrow is functioning and she has a normal platelet count,
but she is generating a decreased number of abnormal RBCs. The
microcytic (small), hypochromic (decreased intracellular
hemoglobin) RBCs are a classic finding of iron-deficiency anemia,
which suggests that blood loss has been sufficiently sustained to
lead to iron depletion. A decrease in her blood pressure on
standing (postural hypotension) suggests decreased intravascular
volume, indicating that there may also be a component of acute
blood loss. If she had renal failure, she would be anemic with
normal RBCs. People with sickle cell anemia have misshapen
RBCs. Megaloblastic anemia is characterized by macrocytic (large)
RBCs.
TMP14 p. 446
15. C) Macrophages become activated and enlarged at sites of
inflammation and can ingest up to 100 bacteria per macrophage.
They can extrude digested material and remain viable and active
for many months. Basophils are not phagocytic, and eosinophils
are weak phagocytes. Neutrophils respond rapidly to infection or
inflammation and ingest from 3 to 20 bacteria or other particles
before dying.
TMP14 pp. 451–452, 456–457
16. D) Kupffer cells are tissue-specific macrophages that reside in the
hepatic sinusoids.
TMP14 p. 453
17. A) Eosinophils normally constitute about 2% of the total WBC
count, but they are produced in large numbers in people with
parasitic infections.
TMP14 p. 456
18. B) The majority of WBCs are stored in the bone marrow, waiting
for an increased level of cytokines or stress hormones (e.g.,
epinephrine, corticosteroids) to stimulate their release into the
circulation. Trauma to bone can result in a release of WBCs into
the circulation. This increase in WBC count is not primarily due to
any inflammatory response but instead is aributed to mechanical
trauma and associated stress responses.
TMP14 p. 455
19. B) The WBC count of 42,000 is higher than the range usually seen
as a response to infection and suggests leukemia. The patient’s
florid clinical presentation suggests an acute process, and findings
of a normal CBC 3 months previously confirm that this patient has
an acute leukemia. Granulocytes are myeloid cells, and the fact
that they are in the circulation while still being immature is wholly
compatible with leukemia. Thus, the patient has acute myelocytic
(also referred to as “myelogenous” or “myeloid”) leukemia.
TMP14 p. 457
20. B) Cross-linking of IgE receptors on the surface of basophils and
mast cells results in their degranulation, releasing an array of
inflammatory mediators.
TMP14 p. 457
21. B) The white blood cell count has been elevated for more than a
year and the cells that are in excess are mature-appearing cells that
clearly are lymphocytes. Thus, this is chronic lymphocytic
leukemia (CLL). Patients with CLL frequently remain
asymptomatic for long periods, often many years.
TMP14 p. 457
22. C) Transmigration of WBCs occurs through parts of the
vasculature that have very thin walls and minimal vascular
smooth muscle layers. This includes capillaries and venules.
TMP14 p. 451
23. C) Bacterial killing in cutaneous infections is largely mediated by
neutrophils, which produce bactericidal agents, including reactive
oxygen species. For the acquired immune response to function
normally, T and B lymphocytes and plasma cells, along with
macrophages, are needed.
TMP14 pp. 452, 454–455
24. B) Macrophages follow the initial influx of neutrophils into an
inflammatory site. Whereas neutrophils ingest a modest number
of bacteria per cell before dying, macrophages persist at the site,
ingesting and digesting infectious organisms and necrotic material
and producing cytokines that direct tissue remodeling by
fibroblasts and other cell types. Dendritic cells are resident
antigen-presenting cells, whereas eosinophils are weakly
phagocytic cells whose products (e.g., major basic protein) can kill
parasites without the eosinophils ingesting them.
TMP14 p. 455
25. D) The presence of normal skin test reactivity, which is T cell
mediated, indicates normal function of macrophages and other
antigen-presenting cells, helper T cells, and cytotoxic T cells. This
information, and the reduction in antibody production, localizes
the defect to the B-cell lineage.
TMP14 pp. 462, 467–468
26. D) Patients with HIV have specific loss of T-helper cells, resulting
in a loss of T-cell help for both antibody production and activation
and proliferation of cytotoxic T cells. Assuming that mast cells can
be appropriately stimulated (i.e., bear sufficient residual surfacebound IgE and are exposed to relevant antigen), their processes
for degranulation are intact.
TMP14 p. 467
27. B) Phagocytosis of bacteria is enhanced by the presence on their
surfaces of both immunoglobulin and products of the complement
cascade, which in turn bind to surface receptors on phagocytes.
This “tagging” of bacteria and other particles for enhanced
phagocytosis is called opsonization.
TMP14 p. 465
28. B) Presentation of a foreign antigen bound to MHC I on an
infected cell results in activation of the cytotoxic T cells to kill the
infected cell. Presentation of an antigen by MHC II on
macrophages activates helper T cells, which promote antibody
production and support proliferation of both helper and cytotoxic
T cells.
TMP14 p. 466
29. D) Variable and hypervariable regions are present at the ends of
both the heavy and light chains of immunoglobulin, allowing both
chains to participate in antigen binding.
TMP14 pp. 463–464
30. E) Fluid leaks into the tissue because of an increase in capillary
permeability.
TMP14 pp. 465–466
31. D) Only immunoglobulins bind intact antigen. T-cell receptors
and MHC class I and II molecules bind processed peptides but do
not bind intact antigen.
TMP14 pp. 464, 466
32. C) Infected labial cells process viral antigens and present them,
bound to MHC class I molecules. Viral antigens in association
with MHC class I molecules are recognized by antigen-specific
cytotoxic T cells, which then destroy the virally infected cells,
producing a lip ulcer.
TMP14 pp. 466–468
33. B) IL-2 is secreted by helper T cells when the T cells are activated
by specific antigens. IL-2 plays an important role in the
proliferation of helper, cytotoxic, and suppressor T cells.
TMP14 p. 467
34. D) Helper T cells are activated by the presentation of processed
antigens by MHC class II molecules on the surface of antigenpresenting cells (primarily macrophages and dendritic cells).
p g p y p g
Helper T cells activate B cells to form antibodies, but B cells are
not required for activation of helper T cells. Helper T cells help to
enhance phagocytosis by macrophages but they do not have the
ability to phagocytize bacteria themselves.
TMP14 pp. 466–467
35. C) Cytotoxic cells act on infected cells when the cells have the
appropriate antigen located on their surfaces. Cytotoxic T cells are
stimulated by lymphokines generated by activation of helper T
cells. Cytotoxic T cells destroy an infected cell by expressing
proteins that punch large holes in the membrane of the infected
cells. There is no interaction between cytotoxic T cells and normal
B cells.
TMP14 pp. 467–468
36. A) Because the person has previously demonstrated allergic
reactions, the initial cutaneous response reflects the binding of
antigen by specific IgE on the surfaces of basophils and mast cells,
resulting in the release of an array of inflammatory mediators. In
addition, activation of the complement system results in the
production of C3a and C4a, which also activate basophils and
mast cells. The influx of neutrophils activation of T-helper cells
and the accumulation of antigen-specific T lymphocytes would
take some time.
TMP14 pp. 469–470
37. D) Activation of the complement system results in a series of
actions, including opsonization and phagocytosis by neutrophils,
lysis of bacteria, agglutination of organisms, activation of
basophils and mast cells, and chemotaxis. Fragment C5a of the
complement system causes chemotaxis of neutrophils and
macrophages.
TMP14 p. 465
38. D) HDN occurs when an Rh-negative mother gives birth to a
second or subsequent Rh-positive child. Therefore, the father must
be Rh positive. The mother becomes sensitized to the Rh antigens
during the events surrounding the birth of an Rh-positive child.
HDN in future pregnancies can be prevented by treating the
mother with antibodies against Rh antigen around the birth of
each Rh-positive child. This treatment destroys fetal RBCs in the
mother and prevents the mother from being sensitized to the Rh
antigen. A transfusion of the child after birth does not prevent
HDN in future pregnancies because the mother has already been
exposed to the Rh antigen during the birth process.
TMP14 pp. 473–474
39. B) The mother could be heterozygous for blood types A and O
and thus could have a type O child. Rh positivity could have been
determined by an allele from the actual father. The man in this
couple is type AB and thus could not have contributed a blood
type O allele (persons who are type O must receive an O allele
from each parent).
TMP14 p. 472
40. D) Each parent needs only a single allele for either the A or B
antigen or the Rh(D) antigen to express these antigens on their
blood cells and other cell types. Thus, if each parent also carries an
allele for blood type O, as well as a null allele for the Rh(D)
antigen, then the child can be homozygous for the recessive O
allele and the Rh(D)-negative allele.
TMP14 pp. 472–473
41. D) The appropriate treatment is repetitive removal of Rh-positive
blood, replacing it with Rh-negative blood (an exchange of about
400 ml over 90 minutes). This treatment may be performed several
times over a few weeks. Maternal antibodies disappear over 1 to 2
months, so the newborn’s endogenous Rh-positive cells cease to be
a target. Exchange transfusions can actually be initiated in utero
when there is evidence of an active immune reaction against the
fetus’s blood cells.
TMP14 p. 474
42. C) Allograft rejection occurs primarily through the actions of
cytotoxic T cells. T-helper cells promote this reaction but are not
the effector cells. Both macrophages and dendritic cells may
present antigen that promotes the immune response, but the key
effector cells are cytotoxic T cells.
TMP14 pp. 475–476
43. B) Type A blood has A antigen on the red blood cell surface and
serum antibodies to the B antigen. Type B blood has B surface
antigens and anti-A antibodies. Therefore, transfusing type A
blood into a person with type B blood causes the A antibodies in
the type B recipient to react with the donor blood. Transfusion of
Rh-negative blood into an Rh-positive person with the same ABO
type will not result in any reaction.
TMP14 pp. 472–473
44. C) Isografts and autografts are derived from genetically identical
donors or from the recipient’s own tissues, respectively. Therefore,
they can be expected to be readily accepted because they present
no foreign antigens to the recipient. Allografts and xenografts do
result in the presentation of foreign antigens. Any HLA mismatch
indicates that the transplanted tissue or organ is an allograft that
carries known foreign antigens.
TMP14 p. 475
45. C) A person who has never been exposed to the Rh antigen
before would not have anti-Rh antibodies and thus would not
have an immediate transfusion reaction. Rather, such antibodies
would develop over a few weeks, resulting in gradual
opsonization and phagocytosis of the transfused cells with
moderate, intracellular hemolysis.
TMP14 p. 473
46. D) Type A, Rh-positive cells express both the A and Rh antigens.
Persons with type O blood develop antibodies to type A antigen
through dietary and environmental exposure. Thus, transfusion of
type A blood into a type O recipient results in an immediate
transfusion reaction. Reaction to Rh antigens requires prior
exposure through an Rh-incompatible blood transfusion.
Reactions occur between the recipient’s antibodies and donor
antigens as shown in the following table.
TMP14 pp. 472–473
Donor Donor Antigen Recipient Recipient Antibody Reaction
O negative None AB positive None None
A positive A, Rh A negative B None
AB positive A, B, Rh AB positive None None
A positive A, Rh O positive A, B A (antigen) and A (antibody)
47. D) Unmatched donor HLA antigens of both classes are
recognized as foreign by recipient T cells. In addition, blood group
(ABO) antigens are expressed on the cells of solid organs and can
lead to strong, immediate organ rejection.
TMP14 p. 475
48. D) HDN occurs when the mother is Rh negative and the father is
Rh positive, resulting in an Rh-positive child. Because the child is
O, Rh negative, HDN would not be expected to develop.
TMP14 p. 473
49. C) Antibiotic treatment for pneumonia can kill flora in the
gastrointestinal tract that are critical for the production of vitamin
K. Production of several active cloing factors (prothrombin and
factors VII, IX, and X) has been suppressed in this patient by
warfarin inhibition of VKOR c1, which normally chemically
reduces vitamin K so that it can activate the listed cloing factors.
Further quantitative reduction of vitamin K by the death of critical
gut flora has produced excessive anticoagulation and resulted in
bleeding in this patient. Fresh-frozen plasma is infused to provide
active cloing factors immediately, and vitamin K is provided to
promote endogenous production of active cloing factors. Both
are needed in the seing of acute bleeding.
TMP14 pp. 484, 486–487
50. C) Inheritance of a bleeding disorder from the maternal
grandfather is classic for an X-linked disorder—the grandfather
transmits the affected X allele to his daughter, who transmits it to
her son. Hemophilia A and B, resulting in deficiency of cloing
factors VIII and IX, respectively, are both X-linked disorders that
present in the manner described for this 2-year-old boy.
TMP14 p. 485
51. B) In this patient with chronic, severe viral hepatitis, liver
damage has led to hepatic insufficiency, so that he is no longer
able to manufacture sufficient quantities of cloing factors. These
must be provided in this emergent situation by treatment with
fresh-frozen plasma. Fibrinogen production generally remains
adequate in patients with liver failure.
TMP14 p. 484
52. B) Warfarin functions by interfering with the production of active
cloing factors. It is not effective in inhibiting cloing factors
in vitro.
TMP14 pp. 486–487
53. A) Hemophilia A is caused by a deficiency of cloing factor VIII,
which functions in the intrinsic cloing pathway only.
TMP14 pp. 482, 485
54. D) In sepsis and particularly septic shock, bacteria and bacterial
toxins such as endotoxin, plus the release of large amounts of
tissue factor from traumatized or dying tissue, cause wide spread
activation of the cloing system. This produces innumerable small
clots that occlude small vessels and exacerbate tissue damage.
Paradoxically, this consumes cloing factors to an extent that the
clinical result is bleeding from multiple sites. Even if a toxin that
could inhibit platelet function had been ingested, this usually is
manifested by cutaneous petechiae (small, punctate hemorrhages)
and bruising rather than widespread bleeding.
TMP14 p. 486
55. A) The extrinsic pathway begins with the release of tissue
thromboplastin in response to vascular injury or contact between
traumatized extravascular tissue and blood. Tissue
thromboplastin is composed of phospholipids from the
membranes of the damaged tissues.
TMP14 p. 481
56. B) Severe diarrhea can lead to fat malabsorption. Because vitamin
K is a fat-soluble vitamin, this can result in vitamin K deficiency.
Vitamin K is required for the production of active cloing factors.
Thus, vitamin K deficiency can cause defective blood coagulation
and excessive bleeding.
TMP14 p. 484
57. C) Bleeding time, measured by determining how long it takes for
bleeding to stop from a small, standardized cut, reflects platelet
aggregation and activation and the formation of platelet plugs in
the small vessels that have been damaged. Blood coagulation
involving the activation of thrombin and the accumulation of
fibrin fibrils takes significantly longer.
TMP14 p. 487
58. D) Treating a massive pulmonary embolus is an emergency that
requires lysis of an existing clot. This can be accomplished by
infusing tissue plasminogen activator, which converts
plasminogen to plasmin, which then actively lyses the clot. In this
seing measures that inhibit clot formation are less relevant
because a life-threatening clot already exists.
TMP14 p. 486
59. A) The primary function of heparin is to bind to and activate
antithrombin III.
TMP14 p. 484
60. B) A familial disorder that occurs in males only suggests an Xlinked disorder because males have a single X chromosome, and if
it is defective, then that defect is expressed clinically (rather than
being masked by a normally functioning X chromosome, as occurs
in females). Males receive their X chromosome from their mothers
and their Y chromosome from their fathers. The most common Xlinked bleeding disorders are hemophilia A and hemophilia B,
which are highly penetrant.
TMP14 p. 485
Unit VII: Respiration
1. A healthy 25-year-old medical student participates in a 10-km
charity run for the American Heart Association. Which muscles
does the student use (contract) during expiration?
A) Diaphragm and external intercostals
B) Diaphragm and internal intercostals
C) Diaphragm only
D) Internal intercostals and abdominal recti
E) Scaleni
F) Sternocleidomastoid muscles
2. Several students are trying to see who can generate the highest
expiratory flow. Which muscle is most effective at producing a
maximal effort?
A) Diaphragm
B) Internal intercostals
C) External intercostals
D) Rectus abdominis
E) Sternocleidomastoid
3. The above figure shows three different compliance curves (S, T, and
U) for isolated lungs subjected to various transpulmonary
pressures. Which of the following best describes the relative
compliances for the three curves?
A) S < T < U
B) S < T > U
C) S − T − U
D) S > T < U
E) S > T > U
Questions 4 and 5
Use the figure below to answer Questions 4 and 5.
4. Assuming a respiratory rate of 12 breaths/min, calculate the minute
ventilation.
A) 1 l/min
B) 2 l/min
C) 4 l/min
D) 5 l/min
E) 6 l/min
5. A 22-year-old woman inhales as much air as possible and exhales
as much air as she can, producing the spirogram shown in the
figure. A residual volume of 1.0 liter is determined using the
helium dilution technique. What is her FRC (in liters)?
A) 2.0
B) 2.5
C) 3.0
D) 3.5
E) 4.0
F) 5.0
6. A 22-year-old woman has a pulmonary compliance of 0.2 l/cm H2O
and a pleural pressure of −4 cm H2O. What is the pleural pressure
(in cm H2O) when the woman inhales 1.0 l of air?
A) −6
B) −7
C) −8
D) −9
E) −10
7. A preterm infant has a surfactant deficiency. Without surfactant,
many of the alveoli collapse at the end of each expiration, which in
turn leads to pulmonary failure. Which set of changes is present in
the preterm infant compared with a normal infant?
Alveolar Surface Tension Pulmonary Compliance
A) Decreased Decreased
B) Decreased Increased
C) Decreased No change
D)
Increased Decreased
E)
Increased Increased
F)
Increased No change
G) No change No change
8. A patient has a dead space of 150 ml, FRC of 3 liters, tidal volume
(VT) of 650 ml, expiratory reserve volume (ERV) of 1.5 l, total lung
capacity (TLC) of 8 l, and respiratory rate of 15 breaths/min. What
is the residual volume (RV)?
A) 500 ml
B) 1000 ml
C) 1500 ml
D) 2500 ml
E) 6500 ml
9. The various lung volumes and capacities include the total lung
capacity (TLC), vital capacity (VC), inspiratory capacity (IC), tidal
volume (VT), expiratory capacity (EC), expiratory reserve volume
(ERV), inspiratory reserve volume (IRV), functional residual
capacity (FRC), and residual volume (RV). Which of the following
lung volumes and capacities can be measured using direct
spirometry without additional methods?
TLC VC IC VT EC ERV IRV FRC RV
A) No No Yes No Yes No Yes No No
B) No Yes Yes Yes Yes Yes Yes No No
C) No Yes Yes Yes Yes Yes Yes Yes No
D) Yes Yes Yes Yes Yes Yes Yes No Yes
E) Yes Yes Yes Yes Yes Yes Yes Yes Yes
10. A 34-year-old man sustains a bullet wound to the chest that causes
a pneumothorax. What best describes the changes in lung volume
and thoracic volume in this man compared with normal?
Lung Volume Thoracic Volume
A) Decreased Decreased
B) Decreased Increased
C) Decreased No change
D)
Increased Decreased
E)
Increased Increased
F) No change Decreased
11. A healthy 10-year-old boy breathes quietly under resting
conditions. His tidal volume is 400 ml, and his ventilation
frequency is 12/min. Which of the following best describes the
ventilation of the upper, middle, and lower lung zones in this boy?
Upper Zone Middle Zone Lower Zone
A) Highest Lowest Intermediate
B) Highest Intermediate Lowest
C)
Intermediate Lowest Highest
D) Lowest Intermediate Highest
E) Same Same Same
12. An experiment is conducted in two persons (subjects T and V)
with identical VTs (1000 ml), dead space volumes (200 ml), and
ventilation frequencies (20 breaths/min). Subject T doubles his VT
and reduces his ventilation frequency by 50%. Subject V doubles
his ventilation frequency and reduces his VT by 50%. What best
describes the total ventilation (also called minute ventilation) and
VA of subjects T and V?
Total Ventilation VA
A) T < V T − V
B) T < V T > V
C) T − V T < V
D) T − V T − V
E) T − V T > V
F) T > V T < V
G) T > V T − V
13. A person with normal lungs has an oxygen (O2
) consumption of
750 ml O2
/min. The hemoglobin (Hb) concentration is 15 g/dl. The
mixed venous saturation is 25%. What is the cardiac output?
A) 2500 ml/min
B) 5000 ml/min
C) 7500 ml/min
D) 10,000 ml/min
E) 20,000 ml/min
14. A cardiac catheterization is performed in a healthy adult. The
blood sample withdrawn from the catheter shows 60% O2
saturation, and the pressure recording shows oscillations from a
maximum of 27 mm Hg to a minimum of 12 mm Hg. Where was
the catheter tip located?
A) Ductus arteriosus
B) Foramen ovale
C) Left atrium
D) Pulmonary artery
E) Right atrium
15. A 67-year-old man is admied emergently to the hospital because
of severe chest pain. A Swan-Ganz catheter is floated into the
pulmonary artery, the balloon is inflated, and the pulmonary
wedge pressure is measured. The pulmonary wedge pressure is
used clinically to monitor which pressure?
A) Left atrial pressure
B) Left ventricular pressure
C) Pulmonary artery diastolic pressure
D) Pulmonary artery systolic pressure
E) Pulmonary capillary pressure
16. Which diagram in the above figure best illustrates the pulmonary
vasculature when the cardiac output has increased to a maximum
extent?
A) A
B) B
C) C
D) D
E) E
17. A 19-year-old man sustains a full-thickness burn over 60% of his
body surface area. A systemic Pseudomonas aeruginosa infection
occurs, and severe pulmonary edema follows 7 days later. The
following data are collected from the patient: plasma colloid
osmotic pressure, 19 mm Hg; pulmonary capillary hydrostatic
pressure, 7 mm Hg; and interstitial fluid hydrostatic pressure, 1
mm Hg. Which set of changes has occurred in the lungs of this
patient as a result of the burn and subsequent infection?
Lymph Flow Plasma Colloid Osmotic Pressure Pulmonary Capillary Permeability
A) Decrease Decrease Decrease
B)
Increase Decrease Decrease
C)
Increase Decrease Increase
D)
Increase Increase Decrease
E)
Increase Increase Increase
18. Blood gas measurements are obtained in a resting patient who is
breathing room air. The patient has an arterial content of 19 ml
O2
/min with a PO 2
of 95. The mixed venous O2
content is 4 ml
O2
/100 ml blood. Which condition does the patient have?
A) An increase in physiological dead space
B) Pulmonary edema
C) A low Hb concentration
D) A low cardiac output
19. A normal male subject has the following initial conditions (in the
steady state):
Arterial PO 2
= 92 mm Hg
Arterial O2
saturation = 97%
Venous O2
saturation = 20%
Venous PO 2
= 30 mm Hg
Cardiac output = 5600 ml/min
O2
consumption = 256 ml/min
Hb concentration = 12 g/dl
If you ignore the contribution of dissolved O2
to the O2
content, what
is the venous O2
content?
A) 2.2 ml O2
/100 ml blood
B) 3.2 ml O2
/100 ml blood
C) 4 ml O2
/100 ml blood
D) 4.6 ml O2
/100 ml blood
E) 6.2 ml O2
/100 ml blood
F) 10.8 ml O2
/100 ml blood
G) 16 ml O2
/100 ml blood
20. A 30-year-old woman performs a Valsalva maneuver about 30
minutes after eating lunch. Which option best describes the
changes in pulmonary and systemic blood volumes that occur in
this woman?
Pulmonary Volume Systemic Volume
A) Decreases Decreases
B) Decreases Increases
C) Decreases No change
D)
Increases Decreases
E)
Increases Increases
F)
Increases No change
G) No change Decreases
H) No change Increases
I) No change No change
21. A child who is eating round candies approximately 1.5 cm in
diameter inhales one down his airway, blocking his left bronchiole.
Which of the following describes the changes that occur?
Left Lung Alveolar PCO 2 Left Lung Alveolar PO 2 Systemic Arterial PO 2
A) ↑ ↑ ↔
B) ↑ ↔ ↑
C) ↓ ↓ ↓
D) ↑ ↑ ↑
E) ↑ ↓ ↓
22. The forces governing the diffusion of a gas through a biological
membrane include the pressure difference across the membrane
(ΔP), the cross-sectional area of the membrane (A), the solubility of
the gas (S), the distance of diffusion (d), and the molecular weight
of the gas (MW). Which changes increase the diffusion of a gas
through a biological membrane?
ΔP A S D MW
A)
Increase Increase Increase Increase Increase
B)
Increase Increase Increase Increase Decrease
C)
Increase Decrease Increase Decrease Decrease
D)
Increase Increase Increase Decrease Increase
E)
Increase Increase Increase Decrease Decrease
23. A person’s normal VT is 400 ml with a dead space of 100 ml. The
respiratory rate is 12 breaths/min. The person undergoes
ventilation during surgery, and the VT is 700 with a rate of 12.
What is the approximate alveolar PCO 2
for this person?
A) 10
B) 20
C) 30
D) 40
E) 45
24. A 45-year-old man at sea level has an inspired O2
tension of 149
mm Hg, nitrogen tension of 563 mm Hg, and water vapor pressure
of 47 mm Hg. A small tumor pushes against a pulmonary blood
vessel, completely blocking the blood flow to a small group of
alveoli. What are the O2
and carbon dioxide (CO2
) tensions of the
alveoli that are not perfused (in mm Hg)?
CO2 O2
A) 0 0
B) 0 149
C) 40 104
D) 47 149
E) 45 149
No comments:
Post a Comment
اكتب تعليق حول الموضوع